You are on page 1of 78

NEET Compendium

PHYSICS
Motion of System of Particles and Rigid Body
CONTENTS
Motion of System of Particles and Rigid Body
Theory
Centre of Mass .................................................................................................................................................................... 1
Centre of Mass of a Discrete mass System .......................................................................................................................... 1
Centre of mass of Continuous mass system ........................................................................................................................ 3
Centre of mass of combination of masses ........................................................................................................................... 6
Motion of Center of Mass ................................................................................................................................................... 6
Momentum .......................................................................................................................................................................... 8
Force of a System ................................................................................................................................................................ 9
Collision of Bodies ............................................................................................................................................................. 12
Rigid Body .......................................................................................................................................................................... 17
Moment of Inertia ............................................................................................................................................................... 8
Moment of Inertia of Rigid Bodies ..................................................................................................................................... 20
Theorems of Moment of Inertia .......................................................................................................................................... 23
Moment of Inertia of Some Regular Bodies ........................................................................................................................ 23
Torque ............................................................................................................................................................................... 32
Body is in equilibrium ......................................................................................................................................................... 33
Rotation about a fixed axis .................................................................................................................................................. 35
Angular Momentum (Moment of Linear Momentum ) ....................................................................................................... 36
Direction of angular momentum due to linear motion ........................................................................................................ 37
Angular Impulse .................................................................................................................................................................. 37
Conservation of angular momentum ................................................................................................................................... 38
Work and power in rotational motion .................................................................................................................................. 40
Rolling ............................................................................................................................................................................... 41
Pure Rolling ........................................................................................................................................................................ 42
Rolling Motion on an inclined plane ................................................................................................................................... 43
Instantaneous Axis of Rotation ........................................................................................................................................... 46

Exercises
Exercise-I .......................................................................................................................................................... 47
Exercise-II ......................................................................................................................................................... 58
Exercise-III
KVPY Previous Year’s Questions ....................................................................................................................................... 66
AIPMT/NEET Previous Year’s Questions .......................................................................................................................... 70
Answer Key .......................................................................................................................................... 75
Motion of System of Particles and Rigid Body

Motion of System of
Particles and Rigid Body 1
CENTRE OF MASS : 
and r COM  x COM ˆi  yCOM ˆj  z COM kˆ
Every physical system has associated with it a certain
So, the cartesian co-ordinates of the COM will be
point whose motion characterises the motion of the
n
whole system. When the system moves under some  mi x i
external forces, then this point moves as if the entire m1 x1  m 2 x 2  ......  m n x n i 1
xCOM =  n
mass of the system is assumed to be concentrated at m1  m 2  .......m n  mi
this point and also the external force is applied at this i 1

point for translational motion. This point is called the n

centre of mass of the system.  mi x i


or xCOM = i 1
M
CENTRE OF MASS OF A DISCRETE MASS SYSTEM n

(i) Centre of Mass of a System of ‘N’ Discrete


n
 m i yi  mi zi
i 1
Particles : Similarly, yCOM = i 1 and z COM 
M
M
Consider a system of N point masses m1, m2, m3,
.................... mn whose position vectors from origin O Note
    • If the origin is taken at the centre of mass then
are given by r1, r2 , r3 .............. rn respectively. Then

n
the position vector of the centre of mass C of the
m r
i 1
i i = 0. hence, the COM is the point about
system is given by.
Y which the sum of “mass moments” of the system is
. . . .. . .. . . . zero.
. . . ..m...1 .. ..C. ..m..2   
.. .. . . .. . . . . .. .. . • If we change the origin then r1 , r2 , r3 ....... changes.
. . .. .. . . . r.2. . 
 .. . . . . . .. . .m.
 n So rcm also changes but distance of centre of mass
r1 .. . .. . ... r..n .
m

rc

from its constituent masses does not change.

(ii) Position of COM of two particles : -


O X Consider two particles of masses m1 and m2 separated
   by a distance l as shown in figure.
 m r  m 2 r2  ...........  m n rn
rcm  1 1 ; m1 C m2
m1  m 2  .........  m n l
 Let us assume that m1 is placed at origin and m2 is
n


m r i i
 n

placed at position (l, 0) and the distance of centre of
m r
i 1 1
rcm  n rcm  i i
mass from m1 & m2 is r1 & r2 respectively.

m
M i 1 m1x1  m 2 x 2
i
i 1 So xCOM = m1  m 2

where, mi ri is called the moment of mass of particle
0  m2l m 2l
with respect to origin. r1 = m  m = m  m ...(1)
1 2 1 2
 n 
M 


 m  is the total mass of the system.
i 1
i
r1 C r2
 (0,0) m1
Further, r i  x i ˆi  yi ˆj  z i kˆ m2
l

1
Motion of System of Particles and Rigid Body

So these system is like two particle system of mass 4


m2l
r2 = l – = m1l ...(2) kg and 2kg are located (0, 0) and (4, 3) respectively.
m1  m 2 m1  m 2 then
From the above discussion, we see that 4kg 2kg
(0,0) r C (4,3)
l
r1 = r2 = if m1 = m2, i.e., COM lies midway between the
2 m1x1  m 2 x 2 0  2 4 8 4
xcom = m1  m 2 = = =
42 6 3
two particles of equal masses.
Similarly, r1 > r2 if m1 < m2 and r1 < r2 if m2 < m1 i.e., COM m1 y1  m 2 y 2 0  3 2
ycom = m1  m 2 = 4  2 = 1 m
is nearer to the particle having larger mass.
From equation (1) & (2) 4 
So position of C.O.M is  , 1
m1r1 = m2r2 ...(3) 3 
Centre of mass of two particle system lie on the line
Example-3
joining the centre of mass of two particle system.
The position vector of three particles of masses m1 = 1kg,

SOLVED EXAMPLE m2 = 2 kg and m3 = 3 kg are r1  (iˆ  4ˆj  k) ˆ m,
 ˆ m and r  (2iˆ  ˆj  2k)
Example-1 r2  (iˆ  ˆj  k) 3
ˆ m respectively..
Two particle of mass 1 kg and 2 kg are located at Find the position vector of their centre of mass.
x = 0 and x = 3 m. Find the position of their centre of Sol. The position vector of COM of the three particles will
mass. be given by
  
 m1 r1  m 2 r2  m3 r3
m1 =1kg COM m2 =2kg rCOM 
m1  m 2  m3

Sol. x=0 x=x x=3 Substituting the values, we get


r1=x r2=(3–x)  (1) (iˆ  4jˆ  k)
ˆ  (2) (iˆ  ˆj  k)
ˆ  (3)(2i–j–2k)
ˆ ˆ ˆ
rCOM 
1 2  3
Since, both the particles lie on x-axis, the COM will
1 ˆ ˆ ˆ
also lie on x-axis. Let the COM is located at x = x, then  (3i  j  k) m Ans.
2
r1 = distance of COM from the particle of mass 1 kg = x
and r2 = distance of COM from the particle of mass Example-4
2 kg = (3 – x) Four particles of mass 1 kg, 2 kg, 3 kg and 4 kg are
r1 m 2 x 2 placed at the four vertices A, B, C and D of a square of
Using r  m or  or x = 2 m side 1 m. Find the position of centre of mass of the
2 1 3– x 1
particles.
thus, the COM of the two particles is located at x = 2m.
Sol. Assuming D as the origin, DC as x -axis and DA as y-
Example-2 axis, we have
m1 = 1 kg, (x1, y1) = (0, 1m)
Two particle of mass 4 kg & 2kg are located as shown
in figure then find out the position of centre of mass.

y 2kg
5m

4kg 37°
(0,0) x m2 = 2 kg, (x2, y2) = (1m, 1m)
Sol. First find out the position of 2 kg mass m3 = 3 kg, (x3, y3) = (1m, 0)
x2kg = 5 cos 37° = 4 m and m4 = 4 kg, (x4, y4) = (0, 0)
y2kg = 5 sin 37° = 3 m Co-ordinates of their COM are

2
Motion of System of Particles and Rigid Body

m1x1  m 2 x 2  m3 m3  m 4 x 4 Similarly, zCOM = 0


xCOM = m1  m 2  m3  m 4 L 
i.e., the coordinates of COM of the rod are  , 0, 0 ,
2 
(1)(0)  2(1)  3(1)  4(0) 5 1 i.e. it lies at the centre of the rod.
= = = m = 0.5 m
1 2  3  4 10 2
SOLVED EXAMPLE
m1 y1  m 2 y 2  m3 y3  m 4 y 4
Similarly, yCOM = m1  m 2  m3  m 4 Example-5
(1)(1)  2(1)  3(0)  4(0) 3 A rod of length L is placed along the x-axis between
=
1 2  3  4
=
10
= 0.3 m x = 0 and x = L. The linear density (mass/length)  of
the rod varies with the distance x from the origin as
 = Rx. Here, R is a positive constant. Find the position
of centre of mass of this rod.
Sol. Mass of element dx situated at x = x is
dm =  dx = Rx dx
The COM of the element has coordinates (x, 0, 0).
Therefore, x-coordinate of COM of the rod will be
 (xCOM, yCOM) = (0.5 m, 0.3 m) Ans.
Thus, position of COM of the four particles is as shown y
in figure.

dx
CENTRE OF MASS OF CONTINUOUS MASS SYSTEM
x=0 x=x x=L x
(i) Centre of Mass of a Uniform Rod
Suppose a rod of mass M and length L is lying along L

L
the x-axis with its one end at x = 0 and the other at
xCOM =
 0
x dm
=
0
(x)(Rx) dx
L

x = L. Mass per unit length of the rod =


M
 dm  0
(Rx) dx
L
Hence, dm, (the mass of the element dx situated at L
 x3 
L

M 
R x 2 dx  
 3 0 2L
x = x is) = dx =
0
= 
L L
 x2 
L 3
The coordinates of the element dx are (x, 0, 0). Therefore,
R x dx
0
 
 2 0
x-coordinate of COM of the rod will be
dx The y-coordinate of COM of the rod is


x=0 x=x x=L
y dm
L
 x dm yCOM = =0 (as y = 0)

xCOM =
0
 dm

 dm Similarly, zCOM = 0
Hence, the centre of mass of the rod lies at
L M 
=
 0
(x)  dx 
L   2L 
 3 , 0, 0  Ans.
M  
1 L

L
= x dx  (ii) Centre of mass of a Semicircular Ring
L 0 2
The y-coordinate of COM is Figure shows the object (semi circular ring). By
observation we can say that the x-coordinate of the
 y dm centre of mass of the ring is zero as the half ring is
symmetrical about y-axis on both sides of the origin.
 dm
yCOM = =0
Only we are required to find the y-coordinate of the
centre of mass.
3
Motion of System of Particles and Rigid Body

Y R R
1 2x 1
  R
4M
ycm = dm = x 2 dx
Rd M  M 2
0 0
y=Rsin
ycm d
 X 4R
ycm =
3
To find ycm we use
(iv) Centre of mass of a solid Hemisphere
The hemisphere is of mass M and radius R. To find its
ycm =
1
M
 dm y ...(i) centre of mass (only y-coordinate), we consider an
element disc of width dy, mass dm at a distance y from
Here for dm we consider an elemental arc of the ring at
the centre of the hemisphere. The radius of this
an angle  from the x-direction of angular width d. If elemental disc will be given as
radius of the ring is R then its y coordinate will be R
sin, here dm is given as
r= R 2  y2

M
dm = × R d
R

So from equation ---(i), we have

  The mass dm of this disc can be given as



1 R sin  d

M
ycm = Rd (R sin) = 3M 3M
M R  0 dm = ×  r2 dy = (R2 – y2) dy
0 2R 3
2R 3
ycm of the hemisphere is given as
2R
ycm = ...(ii) R
 R
ycm =
1
M
 dm y = 1
M  2R
3M
3
(R2 – y2) dy y
(iii) Centre of mass of Semicircular Disc 0 0
Figure shows the half disc of mass M and radius R. R

 (R
3
Here, we are only required to find the y-coordinate of = 3
2
 y 2 ) y dy
2R 0
the centre of mass of this disc as centre of mass will be
located on its half vertical diameter. Here to find ycm, 3R
we consider a small elemental ring of mass dm of radius ycm =
8
x on the disc (disc can be considered to be made up (v) Centre of mass of a Hollow Hemisphere
such thin rings of increasing radii) which will be A hollow hemisphere of mass M and radius R. Now we
integrated from 0 to R. Here dm is given as consider an elemental circular strip of angular width
d at an angular distance  from the base of the
2M
dm = ( x)dx hemisphere. This strip will have an area.
R 2
dS = 2R cos  Rd
Y
ycm

dx

x
X
R

2x Its mass dm is given as


Now the y-coordinate of the element is taken as ,

M
as in previous section, we have derived that the centre dm = 2R cos  Rd
2R 2
2R Here y-coordinate of this strip of mass dm can be taken
of mass of a semi circular ring is concentrated at
 as R sin. Now we can obtain the centre of mass of the
Here ycm is given as system as.

4
Motion of System of Particles and Rigid Body

 
2 2
1 1  M 
ycm =
M 
0
dm R sin  =
M   2R
0
2
2 R 2 cos  d  R sin


2
R

= R sin  cos  d ycm =
2
0 h
(vi) Centre of mass of a Solid Cone at the centroid : yc =
3
A solid cone has mass M, height H and base radius R.  A semi-circular ring
Obviously the centre of mass of this cone will lie
somewhere on its axis, at a height less than H/2. To
locate the centre of mass we consider an elemental
disc of width dy and radius r, at a distance y from the
apex of the cone. Let the mass of this disc be dm,
which can be given as
2R
3M yc = xc = 0
dm = × r dy 2 
R H 2
 A semi-circular disc
here ycm can be given as
H

 y dm
1
ycm =
M
0

1 H
 3M  Ry 2 

=  2  dy  y 4R
M  R H  H   yc = xc = 0
0   3
 A hemispherical shell
H
3 3H
y
3
= 3 dy =
H 4
0

(vii) Centre of mass of some common systems


 A system of two point masses m1 r1 = m2 r2
R
yc = xc = 0
2
 A solid hemisphere

The centre of mass lies closer to the heavier mass.


 Rectangular plate (By symmetry)

3R
yc = xc = 0
8
 A circular cone (solid)

b
xc =
2
L h
yc = yc =
2 4
 A triangular plate (By qualitative argument)  A circular cone (hollow)
5
Motion of System of Particles and Rigid Body

We divide the above problem in two parts


(i) First find out position of centre of mass of both
semicircular plate and rectangular plate
separately.
(ii) Then find the position of centre of mass of given
structure .
4R
h Centre of mass of semicircular disc lie at
yc = 3
3
4R
 AB 
CENTRE OF MASS OF COMBINATION OF MASSES 3
Centre of mass of rectangular plate lie at the centre of
SOLVED EXAMPLE plate at point C
 BC = R
Example-6
m
Two circular discs having radius R and mass density  SC
mR
and 2 respectively are placed as shown in figure. Then
find out the position of COM of the system.
R 2
T 2 mSC  ; mR   4 R 2
2
R R
O O
C
m r1 mR
A B sc

Sol. Mass of disc A mA = R2  Let us assume COM is at r1 distance from mR


Mass of disc B mB = 2R2
Due to symmetry the COM of disc A lie at point O and R 2  4R 
.R   R (3  4)
COM of disc B lie at point O. So we realize the above 2  3 
 r1   r1  3(  8) Ans.
problem in a following way R 2
.   4R 2
O C 2
O'
mA mB
x MOTION OF CENTRE OF MASS :
Velocity of centre of mass of system
2R
Centre of mass due to both the disc lie at point C    
dr1 dr2 dr3 drn
(assume), having distance x from mA  m1  m2  m3 ..............  m n
v cm = dt dt dt dt
m B (2R) 2R 2 (2R) 4R M
 x  m  m ; x  ; x
A B (R  2R )
2 2
3    
m1 v1  m 2 v 2  m3 v3 ..........  m n v n
So the centre of mass lie in the disc B having distance =
M
4R
from O. Here numerator of the right hand side term is the total
3
momentum of the system i.e., summation of momentum
of the individual component (particle) of the system
Example-7
Hence velocity of centre of mass of the system is the
Find out the position of centre of mass of the figure
ratio of momentum of the system to the mass of the
shown below.
system.
2R  
 PSystem = M vcm
R
2R Acceleration of centre of mass of system
B rectangular    
A C dv1 dv2 dv3 dv n
Sol. plate
 m  m2  m3 ..............  m n
a cm = 1 dt dt dt dt
Plate M
2R

6
Motion of System of Particles and Rigid Body

   
m1a1  m 2 a 2  m3a 3 ..........  m n a n u 2y
= (20)2
M h2    20 m
2g 2  10
Net force on system
=  Total height h = h1 + h2 = 35 m
M
uy = 20 m/s just after collision.
Net External Force  Net internal Force
=
M MOTION OF CENTRE MASS :
Net External Force As for a system of particles, position of centre of mass
=    
M m1 r1  m 2 r2  m3 r3  ...
R
is CM 
(action and reaction both of an internal force must m1  m 2  m 3  ...
be within the system. Vector summation will cancel all
  
internal forces and hence net internal force on system d r1 dr dr
 m 2 3  m 3 3  ...
d 
m1
is zero) dt dt dt
so ( R CM ) 
  dt m1  m 2  m3  ....
 Fext = M a cm
   
 m1v1  m 2 v 2  .... d r  
where Fext is the sum of the 'external' forces acting on v    v
Velocity of CM CM m1  m 2  ....  dt 
the system. The internal forces which the particles exert
on one another play absolutely no role in the motion  d 
similarly a CM  v CM
of the centre of mass. dt
If no external force is acting on a system of particles, acceleration of CM
the acceleration of centre of mass of the system will be   
 m1a1  m 2a 2  ..   dv 
zero. If ac = 0, it implies that vc must be a constant and a CM   a 
if vcm is a constant, it implies that the total momentum m1  m 2 _ .... dt 
of the system must remain constant. It leads to the we can write
  
principal of conservation of momentum in absence of Mv CM  m1v1  m 2 v 2  ...
external forces.      
  or MvCM  p1  p 2  p 3  .... [ p  mv]
If Fext  0 then vcm = constant    
or MvCM  p CM [ p1  pCM ]
“If resultant external force is zero on the system,
then the net momentum of the system must remain Linear momentum of a system of particles is equal to
the product of mass of the system with velocity of its
constant”.
centre of mass.
 
d (Mv CM )
from Newtons second law Fext 
Example-8
dt

A particle of mass 2 m is projected at an angle of 45°

with horizontal with a velocity of 20 2m / s. After If Fext .  0 then vCM  cons tan t
1 s explosion takes place and the particle is broken If no external force acts on a system the velocity of its
into two equal pieces. centre of mass remains constant, i.e., velocity of centre
As a result of explosion one part comes to rest. Find of mass is unaffected by internal forces.
the maximum height attained by the other part.
(Take g = 10 m/s2) SOLVED EXAMPLE
Sol. Applying conservation of linear momentum at the time
of collision, or at t = 1 s, Example-9

mv  m(0)  2m(20iˆ  10ˆj)
Two blocks of masses m1 and m2 are connected by a
light inextensible string passing over a smooth fixed

 v  40jˆ  20ˆj pulley of negligible mass. Find the acceleration of the
centre of mass of the sytem when blocks move under
At 1 sec, masses will be at height : gravity.
1 1 Sol.  m1 > m2 so m1 will move downwards and m2 upwards
h1  u y t  v y t 2  (20)(1)  (–10)(1)2  15m
2 2 Magnitude of acc of each block
After explosion other mass will further rise to a height: net pulling force (m1  m 2 )g
a 
mass to be pulled m1  m 2
7
Motion of System of Particles and Rigid Body

 
 m1a1  m 2a 2 m1a  m 2a  m1  m 2 
a CM     a
m1  m 2 m1  m 2  m1  m 2 

a m1g a
m2g

(+ ve downwards and – ve upwards)


   
2 x1 and the shell x 2 then m. x1  m . x 2  0 and
 m  m 2   m1  m 2   m1  m 2 
a cm   1    g     g
 m1  m 2   m1  m 2   m1  m 2    3R  3R
x1  x 2   | x 2 | Ans.
In the direction of acceleration of m1, downwards 4 8

Example-10 Example-12
If man walks from A to B find displacement of man and A balloon of mass M with a light rope
plank. and monkey of mass m are at rest in
mid air. If the monkey climbs up the
rope and reaches the top of the rope,
the distance by which the balloon
descends will be-
(The length of the rope = L )
m mL
(A) (B)
 m1 (m  M) 2 mM
Ans. S2 = 
m1  m 2 m M L M  m
(C) (D)
Sol. Initial momentum of system man and plank is zero. Net m mL
ext. force on this system is zero.

Ans. (B)
Sol. Here, the upthrust on the balloon and the weights of
Thus Psys = 0
the balloon and the monkey are the external forces.
 
 v cm = 0  scm =0 Since the balloon-monkey system is in equilibrium,
  then  Fext  0 . When the monkey climbs the rope,
 m1s1  m 2 s2 = 0 only internal forces of centre of mass remain unaltered.
NOTE: CM will not change it’s horizontal position since If the distance of centre of mass from the balloon is a
there is no external force. and from the monkey is b, then Ma = mb and
(a + b) = L
Example-11 m
Inside a smooth spherical shell of the radius R a ball of Therefore a = L. When the monkey moves
mM
the same mass is released from the shown position
up, balloon comes down and meets at the the centre
(Fig.) Find the distance travelled by the shell on the
of mass of the system. Hence the balloon will descend
horizontal floor when the ball comes to the lowest point
of the shell. mL
by a distance
mM

m R MOMENTUM
 
3R/4 Momentum of a single particle P  mv
Momentum depends on reference frame as velocity is
frame dependent.
Sol. In horizontal direction on system there is no net force Momentum of system : For a system consisting of
on the b all + shell system, hence centre of mass will particles of mass m1, m2, m3....
remain at rest hence if the ball moves
8
Motion of System of Particles and Rigid Body

    
i.e. p = constant
Psys  m1v1  m 2 v 2  m 3 v3  ......
This leads to law of conservation of momentum which
While adding momentum we should remember it is is " In the absence of external forces, the total
vector sum. momentum of the system is conserved."
For eg. : Momentum of rotating fan as a system is
zero. Since all three equidistant pts. of blades will be Note:
having momentum at 120° and will add up to zero. (i) For an isolated system, the initial momentum of the
   system is equal to the final momentum of the system.
 m1r1  m 2 r2  m 3 r3  ....... If the system consists of n bodies having
rC.M. 
m1  m 2  m 3  ........ momentum.
   
differentiate above equation wrt time. p1 , p 2 , p3 ........p n then
      
 m1v1  m 2 v 2  m 3 v3  ...... p1  p 2  p3  ........  p n = constant
Vcm 
m1  m 2  m 3  ...... (ii) As linear momentum depends on frame of reference,
observers in different frames would find different
If we multiply both sides by total mass
 
values of linear momentum of a given system but
Psys  MVcm each would agree that his own values of linear
momentum does not change with time provided the
FORCE ON A SYSTEM : system is isolated and closed, i.e., law of
  conservation of linear momentum is independent
Fext = Ma cm of frame of reference though linear momentum
which can be written in terms of its components as depends on frame of reference.
(iii) Conservation of linear momentum is equivalent to
Fext , x = Ma cm , x Newton's III law of motion for a system of two par-
ticles in absence of external force by law of conser-
Fext , y = Ma cm , y and Fext ,z = Ma cm,z vation of linear momentum.
 
The overall translational motion of a system of particles p1  p2  cons tan t
can be analyzed using Newton’s laws as if all the mass  
and the total external force were applied at that point. i.e., m1v1  m 2 v2  cons tan t
Differentiating above with respect to time
SOLVED EXAMPLE  
dv1 dv
m1  m2 2  0 [as m is constant]
dt dt
Example-13 
Two blocks of masses M1 = 1kg and M2 = 2kg kept on   dv 2 
or m1a1  m2a 2  0 [as  a]
smooth surface, are connected to each other through a dt
  
[as F  ma ]
light spring (k = 100 N/m) as shown in the figure. When
or F1  F2  0
we push mass M1 with a force F = 10N and M1 is seen  
to move with an acceleration a1 = 2 m/s2 , what will be or F1  F2
the acceleration of M2? i.e., for every action there is equal and opposite reac-
a1 tion which is Newton's III law of motion.
F M M2 This law is universal, i.e., it applies to body macro-
1 Smooth
scopic as well as microscopic systems.
Sol. From FBD of M1
Examples of conservation of linear momentum
F – kx = M1a1and from FBD of M2, kx = M2a2
(a) When a bullet fired from the gun :
F  M1a1 F  M1a1 M – Mass of gun with man
 a2 = [Ans. 0004, ] m – mass of bullet
M2 M2
V M m
a2 = 4 v
Bullet
Gun
Law of conservation of linear momentum :
According to Newton's Second law of motion the rate 
V – Velocity of gun (with respect to ground)
of change of momentum is equal to the applied force. 
 dp  
dp V – Velocity of bullet (with respect to ground)
F if F  0 then we have 0
The initial momentum of system = 0
dt dt
9
Motion of System of Particles and Rigid Body

 
The final momentum = m V + M V
 
0 = mV+ MV
 m 
 
V= – V
M
m
 V= v
M
 
The direction V is opposite to that of V

(b) (i) When a bullet of mass m with velocity v


pierces into a wooden block of mass M and gets
embedded in it :
mv = (m + M) V
The block is free to move on frictionless surface. Now
both, the bullet and block have same velocity V, then mv
Momentum before collision = Momentum after  V = (m  M)
collision
mv As bullet gets embedded in the block, so the
mv = (m + M ) V  V 
mM collision is not elastic. There is loss of energy. Now
1 if the block rises to a height h, then
Initial Kinetic Energy = mv2 = Ei
2 (m + M)gh = (1/2) (m + M)V2
m m2v2
 h
2(m  M) 2 g
m
v (ii) If bullet moves out of the block
conserving mometum
v mv + 0 = mv1 + Mv2
m (v – v1) = Mv2 ....(1)
1 m2 v2
Final Kinetic Energy = = Ef
2 mM conserving energy
1
Mv 22  Mgh
2
Ef m

E i m  M < 1  Ef < Ei Some part of energy
gets dissipated.

(ii) When the bullet of mass m comes out after


penetrating the block of mass M :

In this case, mv = MV + mv1 ...(1) v2


M
M m v v1
m
m m
v v1 v2  2gh ...(2)
 From eq . (1) & eqn. (2)
n

V
m(v – v1) = M 2gh
1 1 1
Loss of energy = mv2 – ( MV2 + mv12 ) m 2 (v  v1 )2
2 2 2 or h 
2gM 2
(c) (i)When a block of mass M is tied to a string Rocket propulsion :
of length l and a bullet of mass m strikes to it with Rocket is an example of variable mass problem. In a
velocity v and gets embedded in it : rocket the fuel burns and produces gases at high
According to conservation of linear momentum temperatures. These gases are ejected out of the rocket
from the nozzle at the back of the rocket and hence

10
Motion of System of Particles and Rigid Body


dP 
forward force is exerted on the rocket this is the
consequence of conservation of momentum = Fext
 dt
Lets consider a rocket experiencing external force F . 
 MdV  dM
From time t to t + t a mass of fuel m is turned and Fext = – u
 dt dt
expelled as gas with velocity u rel. to the rocket. 
(generally u is constant & is independent of velocity MdV
here is the net force. In rocket propulsion Fext
of rocket). dt
is the weight of rocket.
m

M
SOLVED EXAMPLE
t V

  Example-14

 
V+u A rocket burns 50 g of fuel per second ejecting it
m M as a gas with velocity of 5 × 10 5 cm/s. What force
V + V
is exerted by gas on rocket.
t + t

dm
 Sol. Upward thrust  u
P( t ) = (M + m) V dt
      dm
P( t  t ) = M ( V  V ) + m (V  V  u ) (where u = 5 × 105 cm/s = 5 × 103 m/sec &
dt
= 50
  
P = P (t + t) – P ( t ) gm/s = 0.05 kg/sec)
= 5 × 103 × 0.05 = 250 N
  
P = MV  mu
 Example-15
here we have neglected ( m.V ) as both are small A rocket with mass 20000 kg is blasted upward with
so there product will be very small initial acceleration of 5 m/s2 . Calculate initial force
  of the blast.
dP Lim M V  u  m  Sol. Net force = upthrust – weight of rocket
=   
dt t  0  t  t   dm 
or upthrust  u  = net force (ma) + weight (mg)
  dt 
dV  dm = m (a + g) = 20000 (5 + 10) = 3 × 105 N
=M +u
dt dt
  Example-16
Note : 1. u is defined +ve in direction of V , but In the figure block A has mass m1 (constant) and B has
generally it is opposite. initial mass m0. B is filled with sand which is thrown out
dm by some internal mechanism at constant rate  kg/sec
2. represents rate of releasing mass (r). at velocity v relative to B in downward direction.
dt
Assuming m1 > m0, light string and pulley, no friction
dm in pulley and motion in vertical plane find acceleration
so = r of A.
dt
m + M = constant
dm dM
r 
dt dt
dM
so =–r
dt
dM
where represents rate of change of mass of
dt (m1  m 2 ) g  v
Ans. m1  m 2 ; m2 = m0 –  t
rocket.

11
Motion of System of Particles and Rigid Body

COLLISION OF BODIES Coefficeint of restitution (e) :


A collision is said to take place when either two bodies The coefficient of restitution is defined as the ratio of
physically collide against each other or when the path the impulses of recovery and deformation of either
of one body is changed by the influence of the other body. Mathematically it can be expressed as velocities
body. of colliding bodies as :
As a result of collision, the momentum and kinetic
velocity of separation along line of impact
energy of the interacting bodies change. e
velocity of approach along line of impact
Forces involved in a collision are action-reaction forces,
i.e., the internal forces of the system. Value of e is 1 for elastic collision, 0 for perfectly inelastic
collision, 0 < e < 1 for inelastic collision .
Type of collision according to the direction of collision:
(a) Head on collision : direction of velocities of bodies Head on Elastic collisions :
is similar to the direction of collision. The elastic collision in which the colliding bodies move
along the same straight line path before and after the
   
u1 u2 v2 v1 collision.
A B A B
m1 m2 m2 m1
u1 u2 v1 v2
Before collision After collision
A B A B A B
m1 m2 m1 m2
(b) Oblique collision : direction of velcoties of bodies Before collision During collision After collision
is not similar to the direction of collision.
Assuming initial direction of motion to be positive and
u1 > u2 (so that collision may take place) and applying
law of conservation of linear momentum, we get
m1u1 + m2u2 = m1v1 + m2v2
no force : they collide due to difference of their velocities
only
i.e. m1 (u1 – v1) = m2 (v2 – v2) …(1)

Types of collision according to the conservation law of For collision is elastic, kinetic energy before collision
kinetic energy : must be equal to KE after collision, i.e.
(a) Elastic collision : kinetic energy is conserved.
1 1 1 1
KEbefore collision = KEafter collision (It is not conserved m1u12  m 2 u 22  m1v12  m 2 v 22 ...(2)
2 2 2 2
during collision)

or   
m1 u12  v12  m 2 v 22  v 22 
(b) Inelastic collision : kinetic energy is not conserved.
Some energy is lost in collision Dividing equation (2) by (1)
KEbefore collision > KEafter collision u1 + v1 = v2 + u2
or (u1 – u2) = (v2 – v1) ...(3)
(c) Perfect inelastic collision : Two bodies stick In 1-D elastic collision 'velocity of approach' before
together after the collision. collision is equal to the 'velocity of recession' after
Note : Momentum remains conserved in all types of collision, no matter what the masses of the colliding
collisions
particles be.

12
Motion of System of Particles and Rigid Body

The law is called Newton's law for elastic collision. When light body A collides against, a heavy body B at
Now if we multiply equation (3) by m2 and substacting rest, the body A should start moving with same
it from (1) velocity just in opposite direction while the body B
(m1 – m2)u1 + 2m2u2 = (m1 + m2)v1 should practically remains at rest.

m1  m 2 2m 2
or v1  u1  u2 ...(4)
m1  m 2 m1  m 2 SOLVED EXAMPLE

Similarly multiplying equation (3) by m1 and adding it Example-17


to equation (1) Two ball bearing mass 5 kg each is moving in opposite
2m1u1 + (m2 – m1)u2 = (m2 + m1)v2 directions with equal speed 5m/s. collides head on
with each other. Find out the fnal velocities of the ball
2m1 m  m1 if collision is elastic.
v2  u1  2 u2 ...(5)
m1  m 2 m1  m 2 Sol. Here m1 = m2 = 5kg

Note : u1 = 5 m/s u2 = –5 m/s

Special cases : In such type of condition velocity get interchange so


(a) If the two bodies are of equal masses : v2 = u1 = 5m/s
m1 = m2 = m
v1 = u2 = –5m/s
v1 = u2 and v2 = u1
Thus, if two bodies of equal masses undergo elastic Example-18
collision in one dimension, then after the collision, the A ball of 0.1 kg makes an elastic head on collision with
bodies will exchange their velocities. a ball of unknown mass that is initially at rest. If the 0.1
kg ball rebounds at one third of its original speed. What
(b) If two bodies are of equal masses and second body is is the mass of other ball.
at rest. Sol. Here m1 = 0.1 kg m2 = ?
m1 = m2 and inital velocity of second body u2 = 0
u
v1 = 0, v2 = u1 u2 = 0 u1 = u v1 = –
3
When body A collides against body B of equal mass at
rest, the body Acomes to rest and the body B moves As
on with the velocity of the body A. In this case transfer
 m1  m 2  2m2 u 2 u  0.1  m 2 
v1 =  m  m  u  m  m  3   0.1  m  u
of energy is hundred percent.
e.g. Billiard's Ball, Nuclear moderation.  1 2  1 2  2 

m2 = 0.2 kg
(c) If the mass of a body is negligible as comparaed to
other
m1 >> m2 and u2 = 0 Head on inelastic collision of two particles :
v1 = u1 v2 = 2u1 Let the coefficient of restitution for collision is e
When a heavy body A collides against a light body B (i) Momentum is conserved hence
at rest, the body A should keep on moving with same m1u1 + m2u2 = m1v1 + m2v2 ……(1)
velocity and the body B will move with velocity double (ii) Kinetic energy is not conserved
that of A. (iii) According to Newton's law
if m2 >> m1 and u2 = 0
v 2  v1
then v2 = 0 = –e ……(2)
u 2  u1

13
Motion of System of Particles and Rigid Body

By solving equation (1) and (2)


K m2  1 2
(m1  em 2 )u1 m 2 (1  e)u 2 or 
(m1  m 2 )  K  2 m1u1 
v1 = m  m + m m , K  
1 2 1 2
(ii) If target is massive, i.e. , m2 >> m1
(m 2  em1 )u 2 m1 (1  e)u1
v2 = m  m + m m K
1 2 1 2 = 1 in percentage loss in KE = 100%
K
i.e., if light moving body strikes a heavy target at rest
Perfect Inelastic collision :
and sticks to it.
In case of inelastic collision, after collision two bodies
eg. Collision with a wall
move with same velocity (or stick together)
If two particles of masses m1 and m2, moving with Elastic Collision in two dimension :
velocity u1 and u2 (u2 < u1) respectively along the same Two perfectly elastic bodies A and B of masses M1 and
line collide 'head on' and after collision they have same M2 moving along the same straight line (x-axis) with
common velocity v, then by conservation of linear velocities u1 and u2. After the collision, the two bodies
momentum, A and B travel with velocities v1 and v2 along direction
m1u1 + m2u2 = m1v + m2v making angles 1 and 2 with the incident direction.
m1 u1  m 2 u 2 Collision is perfectly elastic, so according to
or v= (m1  m 2 ) ……(1) conservation of K.E.

1 1 1 1
Now as the KE of the system before collision is m1u12  m 2 u 22  m1 v12  m 2 v 22
2 2 2 2
1 1
K1 = m1u12  m 2 u 22
2 2 2 v2

1
And after collision is K1 = (m1  m 2 )v 2 1 2
2 2
m1 m2
x' u1 u2 1 x

Loss in KE during collision :


1 v1
1 1 2 1
K1 – KF =  m1u1  m 2 u 2  – (m1 + m2)v2 …(2)
2
Before collision after collision
 2 2  2
Substituting the value of v from equation (1), Momentum of the two bodies separately conserved
along x-axis
1 (m1u1  m 2 u 2 ) 2  so m1u1 + m2u2 = m1v1 cos 1 + m2v2 cos 2
K = 2  m1u 2  m 2 u 2  
2 2

 (m1  m 2 )  Applying the conservation of linear momentum along
y-axis
0 + 0 = m1v1 sin  + (–m2v2 sin 2)
1  m1m 2  u1  u 2  2u1u 2  
2 2


K = 2  
or (m1  m 2 )  or m1v1 sin 1 = m2v2 sin 2

Note :
1 m1m 2 1 (i) Total energy remains conserved in all types of
 K = 2 (m  m ) (u1 – u2)2 = µv 2rel collisions.
1 2 2
Note: (ii) Only conservative forces work in elastic collisions.
(i) If the target is initially at rest (iii) Line of collision (L.O.C.)
u2 = 0 and u1 = u Line along which normal contact force acts is called
m1m 2 L.O.C. L.O.C. is independent of direction of velocity
K = 2(m  m ) u2 of colliding bodies.
1 2

14
Motion of System of Particles and Rigid Body

(iv) Collision is an event in which strong force acts Sol. Let v1 and v2 be the velocity of the body just before
between two or more bodies for a short time. and after the collision
(v) In a collision we consider situation before and after 1
K1 = mv12 = mgh1 ……(1)
the collision. These terms refer to conditions when 2
interactive force between particles effectively and
becomes zero. The duration of collision is negligible
as compared to the time for which we observe the 1
K2 = mv22 = mgh2 ……(2)
event. 2

(vi) In a collision effect of external non impulsive (e.g. Dividing we get


gravity, friction) forces can be neglected because v12 h1 10 v1
  =4
they are very small compared to impulsive forces & v 22 h 2 2.5 v2 = 2
time of collision is very small.
(vii) When two bodies collide, they exert force on each 
other through point of contact, perpendicular to T
the plane of contact. The direction of force of
M
interaction is line of collision.
mg(h1  h 2 )
(viii) After collision, only the components of velocity percentage loss in K.E. = × 100
mgh1
along line of collision changes, the  components
of velocity remain unaffected. 10  2.5
= × 100 = 75%
10

SOLVED EXAMPLE Example-21


A body strikes obliquely with another identical
Example-19 stationary rest body elastically. Prove that they will
A simple pendulum of length 1 m has a wooden bob of move perpendicular to each other after collision.
mass 1kg. It is struck by a bullet of mass 10–2 kg moving Sol. Conservation of linear momentum in X-drection gives
with a speed of 2 × 102 m/s. The bullet gets embeded mu1 = mv1 cos  + mv2 cos 2
into the bob. Obtain the height to which the bob rises  u1 = v1 cos 1 + v2 cos 2 ……(1)
before swinging back. conservation of linear momentum in Y-direction
Sol. Applying principle of conservation of linear momentum gives 0 = mv1 sin 1 – mv2 sin 2
mu = (M + m)v  10–2 × (2 × 102) = (1 + 0.01)v  0 = v1 sin 1 – v2 sin 2 ……(2)
2 Conservation of kinetic energy gives
 v=
1.01 2 v2
K.E. of the block with bullet in it, is converted into P.E.
as it rises through a height h 1 2
1
x' u1 2 x
1 m m
(M + m)v2 = (M + m)gh  v2 = 2gh
2
1 v1
v2  2 
2
1 Before collision after collision
h= h=    = 0.2 m
2g  1 . 01  2  9. 8 1 1 1
mu12  mv12  mv22
2 2 2
Example-20
 u12  v12  v 22 ……(3)
A body falling on the ground from a height of 10 m,
Equeation (1)2 + (2)2 gives
rebounds to a height 2.5 m calculate
 u12 + 0 = v12 cos2 1 + v 22 cos2 2 + 2v1v2 cos 1 cos
(i) The percentage loss in K.E.
(ii) Ratio of the velocities of the body just before and 2 + v12 sin2 1 + v 22 sin2 2 – 2v1v2 sin 1 sin 2
just after the collision.
15
Motion of System of Particles and Rigid Body

(iii) Time taken in nth rebound :


 u12 = v12 (cos2 1 + sin2 1) + v 22 (cos2 2 + sin2 2)
h1 = e2h
+ 2v1v2 (cos 1 cos 2 – sin2 1 + sin2 2)
 u12 = v12 + v 22 + 2v1v2 cos 1 +2) 1 2 1
gt1  e 2 gt 2
2 2
{ u12 = v12 + v 22 }
 cos (1 + 2) = 0  1 + 2 = 90º t12  e 2 t 2
t1 = et
Example-22
A ball is dropped freely from height 'h'. This ball is 2h
t1 = e ……(1)
continuously rebounding, then find out : g
(i) velocity of ball after 'n' rebound h2 = e4h
(ii) height attained by the ball after 'n' rebound
1 2 1 
(iii) time taken by the ball in nth rebound gt 2  e 4  gt 2 
(iv) total distance convered by the ball before it stops 2  2 
rebounding t 22  e 4 t 2 ,
Sol. (i) Velocity after 'n' rebound :
t2 = e2 t

2h
t2 = e2 ……(2)
g
h h1 h2
2h
Similarly t n  e
n
v1 v2 v3 v4 tn = en t
v1 v2 v3 up to  g

No. of collision 1 2 4 5 Special Case : Total time in 'n' rebound (T)


T = t + 2t1 + 2t2 + ……
   
By Newton formula  v 2  v1   e  u1  u 2  T = t + 2et + 2e2t + 2e3t + ……
  T = t + 2t (e + e2 + e3 + ……)
v= 2gh here v 2 = 0, u 2 = 0 (surface at rest)
v1 = ev (opposite direction)  e  1 e  2h  1  e 
T = t + 2t  =  =  
v1 = u1 1 e   1  e  g 1 e 
v1 = ev ……(1)
v2 = ev1 ……(2)
v2 = e(ev) ……(3) [(1) & (2)]
(iv) Distance convered by the ball before it stops :
Similarly v3 = e3v, v4 = e4v ……
vn = en v S = h + 2h1 + 2h2 + …… + 
vn  e n
2gh
= h + 2e2h + 2e4h + 2e6h + ……

(ii) Height attained by the ball after 'n' rebound : = h + 2e2h (1 + e2 + e4 + e6 + …… )


v1 = ev
2gh1  e 2gh  1   2e 2 
S = h + 2e h  2 =h
2 1  2 ,
h1 = e2h  1 e   1 e 
v2 = e2v
2gh 2  e 2 2gh
 1  e2 
h2 = e4h S = h  1  e2 
Similarlyhn = e2n h  

16
Motion of System of Particles and Rigid Body

ROTATIONAL MOTION
RIGID BODY  For every pair of particles in a rigid body, there is
Rigid body is defined as a system of particles in which no velocity of separation or approach between
distance between each pair of particles remains the particles. i.e. any relative motion of a point B
constant (with respect to time). Remember, rigid body on a rigid body with respect to another point A on
is a mathematical concept and any system which the rigid body will be perpendicular to line joining
satisfies the above condition is said to be rigid as long A to B, hence with respect to any particle A of a
as it satisfies it. rigid body the motion of any other particle B of
that rigid body is circular motion.

Let velocities of A and B with respect ground be VA

and VB respectively in the figure below..

A & B are beads which move on a circular fixed ring

If the above body is rigid VA cos1 = VB cos2 (velocity


of approach/separation is zero)
VBA = relative velocity of B with respect to A.
VBA = VA sin 1 + VB sin 2 (which is perpendicular to
line AB)
B will appear to move in a circle to an observer fixed at
A.
W.r.t. any point of the rigid body the angular velocity
of all other points of the that rigid body is same.
Suppose A, B, C is a rigid system hence during any
motion sides AB, BC and CA must rotate through the
same angle. Hence all the sides rotate by the same rate.
 If a system is rigid, since there is no change in the
distance between any pair of particles of the
system, shape and size of system remains
constant. Hence we intuitively feel that while a
stone or cricket ball are rigid bodies, a balloon or
elastic string is non rigid.
But any of the above system is rigid as long as
relative distance does not change, whether it is a
cricket ball or a balloon. But at the moment when
the bat hits the cricket ball or if the balloon is
squeezed, relative distance changes and now the From figure (i) angular velocity of A and B w.r.t. C is ,
system behaves like a non-rigid system. From figure (ii) angular velocity of Aand C w.r.t. B is ,

17
Motion of System of Particles and Rigid Body

TYPES OF MOTION OF RIGID BODY

Types of Motion of rigid body

Pure Translational Pure Rotational Combined Translational and


Motion Motion Rotational Motion

Pure Translational Motion


A body is said to be in pure translational motion, if the
displacement of each particle of the system is same
during any time interval. During such a motion, all the
 
particles have same displacement (s) , velocity (v) and
We know that each particle has same angular velocity

acceleration (a) at an instant. (since the body is rigid.)
so, v1 = r1, v2 = r2, v3 = r3 ...... vn = rn
Consider a system of n particle of mass m1, m2, m3, ......
mn under going pure translation. 1 1
Total Kinetic Energy = m1v12 + m2v22 + ...
then from above definition of translational motion 2 2
1
= [m r 2 + m2r22 + ..........................] 2
2 11
1 2
=  Where  = m1r12 + m2r22 + .............
2
(is called moment of inertia)

      = angular speed of body.


a1  a 2  a 3  ......a n = a (say)
   
and v1  v 2  v3  ......vn = v (say) Combined Translational and Rotational Motion
A body is said to be in combined translation and
From Newton's laws for a system. rotational motion if all point in the body rotates about
 =  + m  an axis of rotation and the axis of rotation moves with
Fext m1a1 2a 2 + m 3a 3 + ......................
respect to the ground. Any general motion of a rigid
 =M a body can be viewed as a combined translational and
Fext rotational motion.
Where M = Total mass of the body
    MOMENT OF INERTIA
P = m1v1 + m 2 v2 + m3 v3 + ...................... Like the centre of mass, the moment of inertia is a
   property of an object that is related to its mass
P = MvCom = Mv
distribution. The moment of inertia (denoted by I) is an
Total Kinetic Energy of body =
important quantity in the study of system of particles
1 1 1 that are rotating. The role of the moment of inertia in
m v 2 + m v 2 + ................. = Mv2 the study of rotational motion is analogous to that of
2 1 1 2 2 2 2
mass in the study of linear motion. Moment of inertia
Pure Rotational Motion gives a measurement of the resistance of a body to a
Figure shows a rigid body of arbitrary shape in rotation change in its rotaional motion.
about a fixed axis, called the axis of rotation. Every If a body is at rest, the larger the moment of inertia of a
point of the body moves in a circle whose center lies body the more difficuilt it is to put that body into
on the axis of rotation, and every point moves through rotational motion. Similarly, the larger the moment of
the same angle during a particular time interval. Such a inertia of a body, the more difficult to stop its rotational
motion is called pure rotation. motion. The moment of inertia is calculated about some
axis (usually the rotational axis).

18
Motion of System of Particles and Rigid Body

Moment of inertia depends on :


SOLVED EXAMPLE
(i) density of the material of body

(ii) shape & size of body Example-23


Two heavy particles having masses m1 & m2 are situated
(iii) axis of rotation in a plane perpendicular to line AB at a distance of r1
and r2 respectively.
In totality we can say that it depends upon distribution
of mass relative to axis of rotation.

Note
Moment of inertia does not change if the mass :
(i) is shifted parallel to the axis of the rotation
(ii) is rotated with constant radius about axis of
rotation
(i) What is the moment of inertia of the system about
axis AB?
Moment of Inertia of a Single Particle
(ii) What is the moment of inertia of the system about
For a very simple case the moment of inertia of a an axis passing through m1 and perpendicular to
the line joining m1 and m2 ?
single particle about an axis is given by, (iii) What is the moment of inertia of the system about
an axis passing through m1 and m2?
Sol. (i) Moment of inertia of particle on left is I1 = m1r12.
Moment of Inertia of particle on right is I2 = m2r22.
Moment of Inertia of the system about AB is
I = I1+ I2 = m1r12 + m2r22
(ii) Moment of inertia of particle on left is I1 = 0
Moment of Inertia of the system about CD is
I = I1 + I2 = 0 + m2(r1 + r2)2
I = mr2 ...(i) (iii) Moment of inertia of particle on left is I1 = 0
Here, m is the mass of the particle and r its distance Moment of inertia of particle on right is I2 = 0
Moment of Inertia of the system about EF is
from the axis under consideration.
I = I1 + I2 = 0 + 0

Example-24
Moment of Inertia of a System of Particles
Three light rods, each of length 2, are joined together
The moment of inertia of a system of particles about an to form a triangle. Three particles A, B, C of masses m,
2m, 3m are fixed to the vertices of the triangle. Find the
axis is given by,
moment of inertia of the resulting body about
I= m r
i
i i
2
...(ii)
(a) an axis through A perpendicular to the plane ABC,
(b) an axis passing through A and the midpoint of BC.

Sol.

where ri is the perpendicular distance from the axis to


the ith particle, which has a mass mi.
(a) B is at a distant 2 from the axis XY so the moment
of inertia of B (IB) about XY is 2 m (2)2

19
Motion of System of Particles and Rigid Body

The moment of inertia of the body about X' Y' is (A) Uniform rod about a perpendicular bisector
m(0)2 + 2m()2 + 3m()2 = 5 m2 Consider a uniform rod of mass M and length l figure
and suppose the moment of inertia is to be calculated
about the bisector AB. Take the origin at the middle
point O of the rod. Consider the element of the rod
between a distance x and x + dx from the origin. As the
rod is uniform,
Mass per unit length of the rod = M/l
so that the mass of the element = (M/l)dx.

The perpendicular distance of the element from the line


Example-25 AB is x. The moment of inertia of this element about AB
Four particles each of mass m are kept at the four corners is
of a square of edge a. Find the moment of inertia of the
M
system about a line perpendicular to the plane of the dI  dx x 2 .
l
square and passing through the centre of the square.
Sol. The perpendicular distance of every particle from the When x = – l/2, the element is at the left end of the rod.
As x is changed from – l/2 to l/2, the elements cover the
given line is a / 2 . The moment of inertia of one
whole rod.
2 1 2 Thus, the moment of inertia of the entire rod about AB
particle is, therefore, m(a / 2) = ma . The moment is
2
of inertia of the system is, l /2
l /2
M 2  M x3  M l2
1
I  l x dx  l 3
  –l /2

12
therefore, 4  ma = 2 ma2.
2
 l /2
2
(B) Moment of inertia of a rectangular plate
about a line parallel to an edge and passing
through the centre
The situation is shown in figure. Draw a line parallel to
AB at a distance x from it and another at a distance x +
dx. We can take the strip enclosed between the two
lines as the small element.
MOMENT OF INERTIA OF RIGID BODIES
For a continuous mass distribution such as found in a
rigid body, we replace the summation of I   mi ri2 by
i

an integral. If the system is divided into infinitesimal


element of mass dm and if r is the distance from a mass
element to the axis of rotation, the moment of inertia is,
It is “small” because the perpendiculars from different
I =  r 2 dm points of the strip to AB differ by not more than dx. As
the plate is uniform,
M
its mass per unit area =
bl

M M
Mass of the strip = b dx  dx .
bl l
The perpendicular distance of the strip from AB = x.

20
Motion of System of Particles and Rigid Body

The moment of inertia of the strip about The moment of inertia of the plate about OX is
R
M 2M 3 MR 2
AB = dI = dx x 2 . The moment of inertia of the given I x dx  .
l R2 2
0
plate is, therefore,
l/2
M 2 M l2 (E) Moment of inertia of a hollow cylinder about
I 
l / 2
l
x dx 
12 its axis
Suppose the radius of the cylinder is R and its mass is
The moment of inertia of the plate about the line parallel
M. As every element of this cylinder is at the same
to the other edge and passing through the centre may
be obtained from the above formula by replacing l by b perpendicular distance R from the axis, the moment of
and thus, inertia of the hollow cylinder about its axis is

Mb 2 I   r 2 dm  R 2  dm  MR 2
I .
12
(F) Moment of inertia of a uniform solid cylinder
(C) Moment of inertia of a circular ring about about its axis
its axis (the line perpendicular to the plane of Let the mass of the cylinder be M and its radius R.
the ring through its centre) Draw two cylindrical surface of radii x and x + dx coaxial
Suppose the radius of the ring is R and its mass is M. with the given cylinder. Consider the part of the cylinder
As all the elements of the ring are at the same in between the two surface. This part of the cylinder
perpendicular distance R from the axis, the moment of may be considered to be a hollow cylinder of radius x.
inertia of the ring is The area of crosssection of the wall of this hollow
cylinder is 2 x dx. If the length of the cylinder is l, the
I   r 2 dm   R 2 dm  R 2  dm  MR 2 . volume of the material of this elementary hollow cylinder
is 2 x dxl.
(D) Moment of inertia of a uniform circular plate The volume of the solid cylinder is  R2 l and it is uniform,
about its axis hence its mass per unit volume is
Let the mass of the plate be M and its radius R. The
centre is at O and the axis OX is perpendicular to the
plane of the plate.

Draw two concentric circles of radii x and x + dx, both


M
centred at O and consider the area of the plate in 
between the two circles.  R2 l
This part of the plate may be considered to be a circular The mass of the hollow cylinder considered is
ring of radius x. As the periphery of the ring is 2 x and
M 2M
its width is dx, the area of this elementary ring is 2xdx. 2 x dx l  2 x dx .
 R2 l R
The area of the plate is  R2. As the plate is uniform,
As its radius is x, its moment of inertia about the given
M axis is
Its mass per unit area =
 R2
 2M 
dI   2 x dx  x 2 .
M 2M x dx R 
Mass of the ring  2 x dx 
R 2
R2 The moment of inertia of the solid cylinder is, therefore,
Using the result obtained above for a circular ring, the R
2M 3 MR 2
moment of inertia of the elementary ring about OX is I 2
x dx  .
0
R 2
 2Mx dx  2
dI    x .
Note that the formula does not depend on the length of
 R
2
the cylinder.
21
Motion of System of Particles and Rigid Body

(G) Moment of inertia of a uniform hollow two spheres of radii x and x + dx concentric with the
sphere about a diameter given solid sphere. The thin spherical shell trapped
Let M and R be the mass and the radius of the sphere, between these spheres may be treated as a hollow
O its centre and OX the given axis (figure). The mass is sphere of radius x.
spread over the surface of the sphere and the inside is The mass per unit volume of the solid sphere
hollow. M 3M
Let us consider a radius OA of the sphere at an angle  = 
4 3 4R 3
with the axis OX and rotate this radius about OX. The R
3
point A traces a circle on the sphere. Now change  to
 + d and get another circle of somewhat larger radius The thin hollow sphere considered above has a surface
on the sphere. The part of the sphere between these area 4x2 and thickness dx. Its volume is 4 x2 dx and
hence its mass is
two circles, shown in the figure, forms a ring of radius
R sin. The width of this ring is Rd and its periphery  3M  3M 2
=  4  R 3  (4 x dx) = R 3 x dx
2

is 2R sin. Hence,  


the area of the ring = (2R sin) (Rd). Its moment of inertia about the diameter OX is, therefore,
M 2  3M 2  2M 4
Mass per unit area of the sphere  . dl =  3 x dx  x2 = 3 x dx
4 R 2 3R  R
M M If x = 0, the shell is formed at the centre of the solid
The mass of the ring  (2R sin )(Rd)  sin  d.
4R 2 2 sphere. As x increases from 0 to R, the shells cover the
whole solid sphere.
The moment of inertia of the solid sphere about OX is,
therefore,
R
2M 2
I= R
0
3
x 4 dx =
5
MR 2 .

RADIUS OF GYRATION (K)


The radius of gyaration of a body is the distance from
The moment of inertia of this elemental ring about OX is axis of rotation, the square of this distance when
multiplied by the mass of body then it gives the moment
M  M
d I   sin  d. (R sin ) 2  R 2 sin 3  d of inertia of the body (I = MK2) about same axis of
 2  2 rotaion
As  increases from 0 to , the elemental rings cover
the whole spherical surface. The moment of inertia of
the hollow sphere is, therefore,
MR 2  
 
M 2 3
I R sin  d    (1  cos ) sin  d 
2

0 2 2 0 
MR  
2 
   (1  cos ) d (cos ) 
2

2   0 

MR 2  cos3   2
  cos     MR
2

2  3 0 3
I = MK2

(H) Moment of inertia of a uniform solid sphere  mr


I= 2

about a diameter MK =  mr
2 2

Let M and R be the mass and radius of the given solid


m1r12  m 2 r2 2  ......  m n rn 2
K2 =
M
m1r12  m 2 r2 2  .......  m n rn 2
K=
m1  m 2  ......m n
if m1 = m2 = m3 = m then, M = mn
r12  r2 2  ........  rn2
K=
n
n = Total number of particles
sphere. Let O be centre and OX the given axis. Draw

22
Motion of System of Particles and Rigid Body

I Unsymmetrical attachment :
Radius of gyaration K 
M I  mr 2
K
Mm

Radius of gyration depends on


(i) axis of rotation
(ii) distribution of mass of body
Radius of gyration does not depend on
(i) Mass of the body
(ii) Angular quantities (angular displacement, angular THEOREMS OF MOMENT OF INERTIA
velocity etc.) Theorem of perpendicular axis
Symmetrical separation (applicable only for two dimensional bodies or plane
M = Mass of disc laminas)
R = Radius of disc The moment of inertia of a plane lamina about an axis
perpendicular to its plane is equal to the sum of the
moments of inertia of the lamina about any two mutually
perpendicular axis in its own plane intersecting each
other at the point through which the perpendicular axis
passes.
M
if part is separated.
4
3 Iz = Ix + Iy
Remaining Mass = M = M'
4
3
M.I. of remaining part I' = I where
4
Radius of gyration of remaining part Ix = MI of the body about X-axis
Iy = MI of the body about Y-axis
3I Iz = MI of the body about Z-axis
K'  4 = I K applicable only for two dimensional bodies and can
3M M not be used for three dimensional bodies.
4 Theorem of parallel axis (for all type of bodies)
 K remains unchanged
Symmetrical attachment :

I
K
M

Moment of inertia of a body about any axis is equal to


the moment of inertia about a parallel axis passing
through the centre of mass plus product of mass of the
body and the square of distance between these two
2I
K'  K parallel axis.
2M I = ICM + Md2
Unsymmetrical separation : ICM = Moment of inertia about the axis passing through
centre of mass applicable for bodies of any type and
I  mr 2 shape.
K
Mm
MOMENT OF INERTIA OF SOME REGULAR BODIES
Moment of inertia of ring
(i) About an axis passing through the centre of ring
and perpendicular to its plane
mass of ring = M and radius of ring = R
23
Motion of System of Particles and Rigid Body

z
SOLVED EXAMPLE
O R
Example-26
dm Find the moment of inertia about axis AB.
z' Sol. IAB = Idia + MR2 A
The ring is assumed to be made up of small elements.
MR 2 R
Consider one such element of mass dm = + MR2
Moment of inertia of this element about the axis 2
ZOZ' = dI
3
dI = dm . R2 = MR2
2 B
Moment of inertia of the whole ring about axis ZOZ ' is R
I   dI  R 2
 dm  MR
2
Example-27
2
I = MR Calculate moment of inertia w.r.t. rotational axis XX' in
(ii) About the diameter of the ring following figures.
Let moment of inertia of the ring about each
massless rode X
1kg 4kg
Z y
X'
x' 20cm 70cm 100cm
x
Id fig (a)
y'
Id
Z' Y
4kg (0,3)
diameter = Id (i.e. XX' and YY').
X' X
Both the diameters are perpendicular to the axis ZZ'
which is passing through the centre of ring and 2kg (0,–2)
perpendicular to its plane, by theorem of perpendicular fig (b)
3kg (0,–4)
axes. I xx '  I yy'  I zz '
1 Sol. (a) Ixx' = 4 × (0.3)2 + 1 x (0.8)2 = 1 kg-m2
or Id + Id = IZ  2Id = MR2  Id  MR 2
2 (b) Ixx' = 4 × (3)2 + 2 × (2)2 + 3 × (4)2 = 92 kg-m2
(iii) About an axis tangential and parallel to the
diameter of the ring Example-28
Let moment of inertia of the ring about the tangent AB Four bodies of masses 5 kg , 2 kg, 3 kg, and 4 kg are
parallel to the diameter YY' of the ring = IT respectively placed at position (0,0,0), (2,0,0), (0, 3, 0)
Applying theorem of parallel axis
and (-2, –2, 0). Calculate the moment of inertia
IT = moment of inertia of ring about diameter YY' + MR2
about x- axis, y-axis and z-axis.
1 3
IT  MR 2  MR 2  I T  MR 2
2 2
(iv) About the tangent parallel to the axis passing Y
3kg (0,3,0)
through the centre of ring and perpendicular to
its plane.
Let M.I. of the ring about the tangent (AB) parallel to
an axis passing through the centre of the ring and (0,0,0) 5kg 2kg
X' X
perpendicular to its plane = I'T (2,0,0)
z 4kg
I I'T (–2,2,0)

Sol. Ix = 3 × (3)2 + 4 × (2)2 = 43 unit


Iy = 2 × (2)2 + 4 × (2)2 = 24 unit
R
z'
Applying theorem of parallel axis Iz = 2 × (2)2 + 3 × (3)2 + 4 × (2 2) 2
I'T = moment of inertia about ZZ' + MR2 = MR2 + MR2 = 8 + 27 + 32 = 67 unit
I'T = 2MR2 (Iz = Ix + Iy = 43 + 24 = 67)
24
Motion of System of Particles and Rigid Body

MOMENTOFINERTIAOFSOMEREGULARBODIES
Shape of Position of the axis Figure Moment of Radius of
the body of rotation Inertia (I) gyration (K)
z
(1) Circular (a) About an axis
Ring perpendicular to the CM d
plane and passes MR2 R
through the centre B

Mass = M
Radius = R
y Iz
x'

1 R
(b) About the diametric axis MR 2
x 2 2
y'

I1
(c) About an axis tengential
CM R
to the rim and 2MR2 2R
perpendicular to the plane M
of the ring

(d) About an axis tangential


O
3 3
to the rim and lying in the MR 2 R
2 2
R
plane of ring

(2) Circular Disc (a) About an axis passing

R 1 R
through the centre and MR 2
2 2
perpendicular to the plane
of disc
Mass = M
Radius = R
y
x'
MR 2 R
(b) About a diametric axis
4 2

x y'

25
Motion of System of Particles and Rigid Body

Shape of Position of the axis Figure Moment of Radius of


the body of rotation Inertia (I) gyration (K)

(c) About an axis tangential


CM 5 5
to the rim and lying in the MR 2 R
R O ICM 4 2
plane of the disc

(d) About an axis tangential Iz


3 3
to the rim & perpendicular CM MR 2 R
2 2
to the plane of disc R
M
R

(3) Annular disc (a) About an axis passing


M 2 R 12  R 22
R2  R1  R 22 
2 
R2 R1 through the centre and R1
2
perpendicular to the plane M
of disc
M = Mass
R1 = Internal
Radius
M 2 R 12  R 22
 R1  R 22 
4 
R2 = Outer (b) About a diameteric axis R2 Radius
R1 2
(R2 > R1) M

I
(4) Solid Sphere (a) About its diametric axis
2 2
which passes through its MR 2 R
5 5
centre of mass M R

M = Mass
R = Radius

I
(b) About a tangent to the
7 7
Sphere MR 2 R
5 5
M R

26
Motion of System of Particles and Rigid Body

Shape of Position of the axis Figure Moment of Radius of


the body of rotation Inertia (I) gyration (K)

I
(5) Hollow (a) About diametric axis M
2 2
Sphere passing through centre of MR 2 R
3 3
(Thin spherical mass R
Shell)

table tennis ball

(b) About a tangent to 5 5


I MR 2 R
the surface M 3 3
M = Mass
R = Radius R
Thickness
negligible
table tennis ball

(6) Hollow About diametric axis passes


I 2 
R 5  r5  
2 R r
5 5


M 3 3
sphere with cavity through centre of mass M
5 R r  
5 R 3  r3 
R

lawn tennis ball


r=Internal radius
R=Outer radius
M=Mass

(7) Hollow (a) About its geometrical


Cylinder axis which is parallel to its MR2 R
length

M = Mass
R = Radius (b) About an axis which is
 R 2 L2  R 2 L2
L = Length perpendicular to its length M   
 2 12  2 12
and passes through its
centre of mass
L

27
Motion of System of Particles and Rigid Body

Shape of Position of the axis Figure Moment of Radius of


the body of rotation Inertia (I) gyration (K)

(c) About an axis


 R 2 L2  R 2 L2
perpendicular to its length M   
 2 3 2 3
and passing through one
end of the cylinder
L

(8) Solid (a) About its geometrical I


MR 2 R
Cylinder axis, which is parallel to R M
2 2
M = Mass its length
R = Radius
L = Length

I
(b) About an axis tangential
R M 3 3
to the cylinderical surface MR 2 R
2 2
and parallel to its
geometrical axis

(c) About an axis passing


 L2 R 2  L2 R 2
through the centre of M   
12 4  12 4
mass and perpendicular
to its length L

(9) Thin Rod (a) About an axis passing


ML2 L
through centre of mass and
12 2 3
perpendicular to its length L

Thickness is
negligible (b) About an axis passing
ML2 L
w.r.t. length through one end and
3 3
perpendicular to length L
of the rod

28
Motion of System of Particles and Rigid Body

Shape of Position of the axis Figure Moment of Radius of


the body of rotation Inertia (I) gyration (K)

(10) Rectangular (a) About an axis passing


Mb 2 b
Plate through centre of mass and
b 12 2 3
perpendicular to side b in a
its plane

M = Mass (b) About an axis passing


Ma 2 a
a = Length through centre of mass and
12 2 3
b = Breadth perpendicular to side a in b
its plane. a

(c) About an axis passing


M 2 a 2  b2
throught centre of mass (a  b 2 )
b 12 12
and perpendicular to plane a

(11) Triangular About an axis passing


Ma 2 a
Prism through centre of mass
6 6
perpendicular to its plane

a a

each side of
base = a

(12) Cube About an axis passes


Ma 2 a
through centre of mass
6 6
and perpendicular to face

Side a

29
Motion of System of Particles and Rigid Body

Therefore moment of inertia of the system about


Note BC = MI of sphere at A about BC + MI of sphere at B
It has dimension [ML2] and SI unit kgm2. about BC + MI of sphere at C about BC + MI of sphere
Moment of inertia for a given axis depends on mass, at D about BC
shape and size of body. If same mass is casted in 2  2 2 2 
=  Ma 2  Mb2   Ma 2  Ma 2   Ma 2  Mb 2 
different shapes with same axis, moment of inertia 5  5 5 5 
will be different.
8 2
It is not a vector as direction CW or ACW is not to = Ma 2  2Mb 2 = M(4a 2  5b 2 )
be specfied and also not a scalar as it has different 5 5
values in different directions (i.e. about different
axes). It is a tensor quantity. Example-31
A uniform wire of length  and mass M bented in the
shape of semicircle of radius r as shown in figure.
SOLVED EXAMPLE Calculate moment of inertia about XX'

Example-29 r
The moment of inertia of sphere is 40 kg-m2 about the X X'
diameter. Determine the moment of inertia about any

tangent. Sol. Length of the wire = r  r =

Mr 2 M 2

 Ixx' = =
2 2 2
R C
Example-32
Sol.
Calculate moment of inertia of an annular disc about an
axis which lying in the plane of disc and tangent to the
I Ig (i) inner circle and (ii) outer circle.
Mass of the disc is M and its inner radius is R1 and
2 outer is R2 .
Given that MR 2  40 or MR2 = 100
5
by theorem of parallel axes
2 7 7
I = Ig + MR2 = MR 2  MR 2 = MR 2 =  100 Sol.
5 5 5
= 140 kg-m2
(i) MI about tangent to the inner circle is
Example-30 M
Four sphere each of diameter 2a and mass M are placed IAB = (R12 + R22) + MR12
4
with their centres on the four corners of a square of (ii) MI about tangent to the outer circle is
side b. Calculate the M.I. of the system about one side
of the square taken as the axis. M
ICD= (R12 + R22) + MR22
4

Example-33
Four spheres each of mass M and diameter 2a are placed
at the corners of square of side b as shown below.
Calculate the moment of inertia w.r.t. given axis (a) Axis
BB' (b) Axis AA'

Sol. ABCD is square of side b and four sphere each of mass


Sol.
M and radius a are placed at the four corners. Suppose
we have to calculate moment of inertia of the system
about side BC as Axis.
30
Motion of System of Particles and Rigid Body

(a) Axis BB' passes through the centre and


perpendicular to the plane Example-36
2
Calculate M.I. of given particle system about axis a - b
 b  and c - d
IBB' = 4 × ICM + 4M  
 2 a c
1kg 2kg 3kg 4kg
1m
2 b2  2 2 b2  2m
=4× Ma2 + 4M × = 4M  a   3m
5 2 5 2 4m d
b
(b) Axis AA' passes through the centre and spheres 1
Sol. I1 – 2 = 1 × (1)2 + 2 × (2)2 + 3 × (3)2 + 4 × (4)2 = 100 kg - m2
and 3
I3 – 4 = 1 × (2)2 + 2 × (1)2 + 3 × (0)2 + 4 × (1)2 = 10 kg - m2
2
 b  2 b2 8
IAA' = 4ICM + 2M   = 4 × Ma2 + 2M = Ma2 Example-37
 2 5 2 5
Adjoining diagram has three disc, in which each has
+ Mb2 mass M and radius R. Find the moment of inertia of this
system about axis xx'
Example-34
Four rods are placed in the form of a square. Calculate
the moment of inertia about an axis passes through the
centre and perpendicular to the plane (Assume mass
M and length L of each rod)

M
L Sol. Isystem = 2 × Iupper + Iinner
5  1 11
= 2  MR 2   MR 2 = MR 2
4  4 4

 L
2
 ML2  ML2 4 Example-38
Sol. I = 4ICM + 4M   = 4   +4 = ML2
2  12  4 3 Diameter of each sphereical shell is R and mass M,
they are joined by a light and massless rod. Calculate
Example-35 the moment of inertia xx' axis.
Three rods are placed in the form of equilateral triangle.
Calculate the M.I. about axis passing through the centre
and perpendicular to the plane (Assume mass and
length of each rod is M and L respectively). number of
sides n = 3
2 2 
Sol. Isystem  MR 2   MR 2  M(2R) 2 
Iz 3 3 
M,L M,L 4 16
 MR 2  4MR 2  MR 2
3 3
x
M,L Example-39
The moment of inertia of a sphere about its diameter is
2 I. Four such spheres are arranged as shown in figure.
3ML2  L 
+ 3M 
 2 3 
Sol. I = 3ICM + 3M x2 = Find the moment of inertia of the system about the axis
12 XX'.
L L L
X= = =
 2 tan 60 2 3
2 tan
n

ML2 ML2 ML2


I= + = 2 5
4 4 2 Sol. I M(2R) 2  M(2R) 2  I
5 2
31
Motion of System of Particles and Rigid Body

 Isystem  I  2[I  M(2R) 2 ]  I Sol. As the mass is uniformly distributed on the disc, so
9M
5 mass density (per unit area) =
= I  2I  2  I  I  9I R 2
2
9M  R 
2

 M
R 2  3 
Example-40 Mass of removed portion =
Two rings have their moments of inertia in the ratio 4 :
So the moment of inertia of the removed portion about
1 and their diameters are in the ratio 4 : 1 Find the ratio
of their masses. the stated axis by theorem of parallel axis is :
2 2
I1 M1  R 1 
2 M R   2R 
I1    M
  3 
......(i)
Sol.   2 3
I2 M 2  R 2 
If the disc would not have been reomoved, then the
2
M1 I1  R 2 
2
4 1 1 moment of inertia of complete disc about the stated
       
M 2 I 2  R1  1 4 4 R
2

axis is I2 then I 2  9M ......(ii)


2
Example-41 So the moment of inertia of the idsc shown fig. in I2 – I1.
Four holes of radius R are cut from a thin square plate i.e., I2 – I1 = 4MR2
of side 4R and mass M. Determine inertia of the
remaining portion about z-axis. TORQUE
Torque represents the capability of a force to produce
change in the rotational motion of the body.
Torque about a point :
   
Torque of force F about a point   r  F

Sol. M = Mass of square plate before cutting the holes


 M  2 M
Mass of one hole m   R 
16R 
2
16
 Moment of inertia of the remaining portion,
I = Isquare – 4Ihole
M  mR 2  
= (16R 2  16R 2 )  4   m( 2R) 2  Where F = force applied
12  2 
P = point of application of force
8  8 10 
 MR 2  10mR 2    MR 2 Q = Point about which we want to calculate the torque.
3  3 16  
r = position vector of the point of application of force
w.r.t. the point Q.
Example-42
which we want to determine the torque.
A thin uniform disc of mass 9M and of radius R. A disc 
 = r F sin = rF = rF
R
of radius is cut as shown in figure. Find the moment Where = angle between the direction of force and the
3
position vector of P wrt. Q.
of inertia of the remaining disc about an axis passing
r = r sin  = perpendicular distance of line of action of
through O and perpendicular to the plane of disc.
force from point Q ,it is also called force arm.
 
F = F sin  = component of F perpendicular to r
S unit of torque is Nm
Torque is a vector quantity and its direction is
determined using right hand thumb rule and its always
perpendicular to the plane of rotation of the body.

32
Motion of System of Particles and Rigid Body

Sol. Torque about point O,


SOLVED EXAMPLE     
  r0  F, r0  ˆi  ˆj , F  5 3 ˆi  5jˆ
Example-43 
  (iˆ  ˆj) × ( 5 3 ˆi  5jˆ ) = 5(1  3) kˆ
A particle of mass M is released in vertical plane from a
point P at x = x0 on the x-axis it falls vertically along the Torque about point A ,
y-axis. Find the torque  acting on the particle at a time     
t about origin ?   ra  F, ra  ˆj , F  5 3 ˆi  5jˆ
Sol. Torque is produced by the force of gravity. 
   ˆj × ( 5 3 ˆi  5jˆ ) = 5( 3) kˆ
 = r F sin  k̂
or  = r F  x 0 mg
Example-46

Find out torque about point A, O and B. If F  10iˆ

y
(3,5)
(0,5)
A

O B (3,0) x
x
= r mg 0 = mgx0 k̂
r
    
Sol. Torque about point A ,  A  rA  F, rA  3iˆ , F 10iˆ
Example-44

A particle having mass m is projected with a velocity v0  A  3iˆ  10iˆ  0
from a point P on a horizontal ground making an angle
 with horizontal. Find out the torque about the point     
ˆ F 10iˆ
Torque about point B ,  B  rB  F, rB  5j,
of projection acting on the particle when it is at its

maximum height ?  B  5jˆ  10iˆ  –50 kˆ
Torque about point O ,
V0
    
 O  rO  F, rO  3iˆ  5j,
ˆ F 10iˆ


P Q  O  (3iˆ  5j)
ˆ  10iˆ  – 50 kˆ

R v 2 sin 2
Sol.  = rFsin = mg = 0 mg
2 2g BODY IS IN EQUILIBRIUM
We can say rigid body is in equillibrium when it is in
mv 0 2 sin 2
= (a) Translational equilibrium
2

i.e. Fnet  0
Example-45
 

Find the torque about point O and A. F  5 3 ˆi  5jˆ F 
net x  
 0 and Fnet y
 0 and

(b) Rotational equillibrium


y 
 net  0
(1,1) 30°
i.e., torque about any point is zero
60°
Note

(i) If net force on the body is zero then net torque of


x the forces may or may not be zero.
O A B

33
Motion of System of Particles and Rigid Body

Sol. Since the body is in equillibrium so we conclude


SOLVED EXAMPLE 
F net  0 and torque about any point is zero i.e.,

Example-47  net  0
A pair of forces each of same magnitude and acting in
opposite direction on the rod.

Let us assume that we apply F force downward at A


Sol. Net force on the body is zero but torque of the forces angle  from the horizontal, at x distance from B

about From F net  0
B  B  F + F  Fnet x = 0 which gives
F2 = 8 N
 B  2F
From Fnet y = 0  5 + 6 = F1 + 3
 about C  C  2F  F1 = 8 N
If body is in equillibrium then torque about point B is
Example-48 zero.
Determine the point of application of third force for  3 × 5 + F1. x – 5 × 10 = 0
which body is in equillibrium when forces of 20 N & 30  15 + 8x – 50 = 0
N are acting on the rod as shown in figure 35
x=  x = 4.375 cm
9

Example-50
A uniform rod length , mass m is hung from two strings
of equal length from a ceiling as shown in figure.
Determine the tensions in the strings ?
Sol. Let the magnitude of third force is F, is applied in upward
direction then the body is in the equilibrium when

(i) Fnet  0 (Translational Equillibrium)
 20 + F = 30  F = 10 N
So the body is in translational equilibrium when 10 N
force act on it in upward direction.
(ii) Let us assume that this 10 N force act.
Then keep the body in rotational equilibrium Sol. Let us assume that tension in left and right string is TA
So Torque about C = 0 and TB respectively. Then
i.e. c = 0  
Rod is in equilibrium then Fnet  0 & net  0

From Fnet  0
mg = TA + TB ...(1)
 30 × 20 = 10 x From net = 0 about A
x = 60 cm
 3
so 10 N force is applied at 70 cm from point A to keep mg  TB  0
the body in equilibrium. 2 4

2mg
Example-49  TB =
Determine the point of application of force, when forces 3
are acting on the rod as shown in figure. 2mg
From eq. (1) TA  = mg
3

mg
 TA =
3

34
Motion of System of Particles and Rigid Body

So Fnet & net on the plank is zero


Example-51 (i) from Fnet = 0
A stationary uniform rod of mass ‘m’, length ‘’ leans  Fnet x = 0
against a smooth vertical wall making an angle  with N1 = 0
rough horizontal floor. Find the normal force & frictional
Now Fnet  0
force that is exerted by the floor on the rod? y

 N2 + T = mg ...(i)
from net = 0
 net about point A is zero
So N2 .  = mg . /2
mg
 N2 
2

Example-53
Sol. As the rod is stationary so the linear acceleration and A square plate is hinged as shown in figure and it is
angular acceleration of rod is zero. held stationary by means of a light thread as shown in
i.e., acm = 0 ;  = 0. figure. Then find out force exerted by the hinge.
N2 = f
 acm=0
N 1 = mg
Torque about any point of the rod should also be zero

Sol. F.B.D.

=0

A = 0  mg cos  + f  sin  = N1 cos . 
2
mgcos 
N1 cos  = sin  f +
2
mgcos  mgcot   Body is in equilibrium and T and mg force passing
f= =
2 sin  2 through one line so
from net = 0, N = 0
Example-52
A thin plank of mass m and length  is pivoted at one ROTATION ABOUT A FIXED AXIS
end and it is held stationary in horizontal position by If Hinge = moment of inertia about the axis of rotation
means of a light thread as shown in the figure then find (since this axis passes through the hinge,
out the force on the pivot. hence the name Hinge ).

ext = resultant external torque acting on the body
about axis of rotation
 = angular acceleration of the body.

Sol. Free body diagram of the plank is shown in figure.  ext Hinge =  
 Plank is in equilibrium condition
Hinge

35
Motion of System of Particles and Rigid Body


1 ˆ  (v ˆi  v ˆj  v k)
L  m (xiˆ  yjˆ  zk) ˆ 
Rotational Kinetic Energy = .  . 2  x y z 
2
   
P  M v CM ; Fexternal  M a CM
ˆi ˆj kˆ

Lm x y z
Net external force acting on the body has two vx vy vz
component tangential and centripetal.

2 i.e., L = m[ î (yvz – zvy) – j (xVZ – zVx) + k (xvy – yvx)]
v
 FC = maC = m = m2 rCM • As the magnitude of the angular momentum
rCM
L = mvr sin
 Ft = mat = m rCM  
(a) For  = 0° or 180° i.e. r and v are parallel or anti -
parallel sin = 0
ANGULAR MOMENTUM (MOMENT OF LINEAR
L will be minimum. Lmin = 0
MOMENTUM )
If the axis of rotation is on the line of motion of moving
Angular momentum of a body about a given axis is the
particle thenangular momentumis minimumand zero.
product of its linear momentum and perpendicular
(b) For  = 90°, i.e., angular momentum is maximum
distance of line of action of linear momentum vector  
from the axis of rotation when r and v are orthogonal sin  = 1
L will be maximum. i.e Lmax = mvr.
In case of circular motion i.e., in case of circular motion
of a particle angular momentum is maximum and is mvr.

• As | L| = mvr sin so if the point is not on the line of
motioni.e.,   0° or 180°, i.e., a particle in translatory
motion always have an angular momentum unless the
point is on the line of motion
• In case of circular motion of a particle
Angular momentum = Linear momentum ×     
L  r  p  m(r  v)
Perpendicular distance of line of action of momentum
  
from the axis of rotation [ p = mv ]  L = mvr sin90 n [  = 90°]
  
L = mv × rsin or L = r  p [Where n is a unit vector perpendicular to the plane of
 motion in accordance with right hand Screw rule]
Here L is the angular momentum of a moving particle 
 or L = mr2 n [ v = r]
about point O, p is the linear momentum of the particle  
 or L =I
and r is the position vector of the particle regarding 
the point. [ mr2 = I and  n =  ]
UNIT : S I J-sec In case of circular motion, angular momentum is equal
to the product of moment of inertia with angular
 
or kg–m2/sec
Dimensions : [M L2 T–1] velocity.This result is rotational analogue of p = mv
• Angular momentum is an axial vector. • In case of rotational motion
L = I
2
1 1 L  L2
and ER = I2  ER = I   =
2 2 I 2I
This result is rotational analogue of
p2
  ET =
• As torque (r  F ) is defined as the 'moment of force', 2m
 
Angular momentum is also define as moment of linear • In case of circular motion for a particle L = I 
momentum.  
dL d
• In cartisian Coordinates angular momentum :  =I [ I is const.]
       
dt

dt
L = ( r × p ) = m ( r × v ) [ p = mv ]
dL  dL 
= I   =
dt dt
36
Motion of System of Particles and Rigid Body


d   
[ =  and  = I  ] SOLVED EXAMPLE
dt
The rate of change of angular momentum is equal to
the net torque acting on the particle. This expression Example-54
 A particle of mass m is projected at t = 0 from the point
dp  P on the ground with speed v0 at an angle of 45° to the
is rotational analogue of = and so also referred
dt F horizontal. Find the magnitude and direction of the
as newtons II law for rotational motion. v0
angular momentum of the particle at time t 
g
DIRECTION OF ANGULAR MOMENTUM DUE TO LINEAR
MOTION Sol. Only force acting on projectile (particle) is its weight
Where you want to determine the angular momentum, mg
imagine your position at that point. Spread palm of v0
so at time t torque of mg about P is = mgd = mg t
right hand along the positon vector and curve the 2
fingers in direction of velocity of particle then thumb
of right hand shows direction of angular momentum.

 change in angular momentum = angular impulse


t t
mgv0 mgv 0 t 2 1 . mv03
L =   dt   t dt  
0 2 0 2 2 2 2 g
initial angular momentum about P is zero (line of action

L = mvb(-k)  of linear momentum is passing through P)
b = r sin v0 1 . mv30
    so angular momentum at time t  is
L = mvrsin (-k)  L = m( r × v ) g 2 2 g
Aparticle is moving parallel to X-axis, or Y-axis or along and its direction is perpendicular to the plane of page
a straight line with constant velocity then its angular inwards.
momentum about origin will remain constant.
Example-55
The diameter of a solid disc is 0.5 m and its mass is
16kg. What torque will increase its angular velocity
from zero to 120 rotations/minute in 8 seconds ?
Sol. Moment of inertia of solid disk
2
1 1  0 .5  1
I MR 2   16    2
2 2  2 
The angular momentum of a system of particles is angular velocity   120  2   4  and change in
equal to the vector sum of angular momentum of each 60
particle angular momentumL = I – 0 = I
     angular impulse ×t = L
L  L1  L 2  L3        Ln
L 1 1 
 =    4  Nm
ANGULAR IMPULSE t 8 2 4
If a large torque acts on a body for a small time then
Angular impulse =  dt Example-56
 A particle of mass 0.01 kg having position vector
Angular impulse =  av.. t =  L = change in angular 
r  (10 ˆi  6 ˆj) metre and moving with a velocity 5 ˆi m/s,
momentum
 calcualte its angular momentum about origin.
 L  
[   ] Sol. p  mv  0.01  5 ˆi  0.0 5 ˆi
t
   ˆ
Action of angular impulse is to change the angular  L  r  p  (10 ˆi  6 ˆj)  0.05 ˆi = 0.3(–k)
momentum. it has same unit, dimensions and directions
as angular momentum. = –0.3kˆ J/sec
37
Motion of System of Particles and Rigid Body

Example-57 Example-60
A solid cylinder of mass 20 kg rotates about its axis A string is wrapped around the rim of a wheel of moment
with angular speed 100 rad. s–1, the radius of the cylinder of inertia 0.20 kg-m2 and radius 20 cm. The wheel is free
is 0.25m. What is the magnitude of the angular to rotate about its axis. Initially, the wheel is at rest. The
momentum of the cylinder about its axis.
string is now pulled by a force of 20 N. Find the angular
Sol. M = 20 kg,
velocity of the wheel after 5 second.
 = 100 rad s–1
R = 0.25m Sol. Angular impulse = change in angular momentum
Moment of inertia of the cylinder about its axis t = I – 0

1 1
MR 2   20   0 .2 5  = 0.625 kg - m2
2
I=
2 2
Angular momentum L = I = 0.625 × 100 = 62.5 joule-
second
Rt 2 0  0.2
F =  10 0 rad/sec.
Example-58 I 0.2
A particle performs uniform circular motion with an
CONSERVATION OF ANGULAR MOMENTUM
angular momentum L. If the frequency of particle's
motion is double and its kinetic energy is halved then Condition of conservation of angular momentum

determine value of angular momentum.
 L Lf  Li I  Ii I(f  i )
Sol. L = mvr  =  = = f =
t  t t t
but V = rand  = 2f
If the resultant external torque acting on a system is
 v  KE
L = mv    zero then the total angular momentum of the system
 2 f  f remains constant.

 1  L
 KE  mv2  If  = 0 then  0  L = constant
 2  t
if the frequency is doubled and KE is halved then  Lf = Li or I11 = I22
If a system is isolated from its surrounding
 KE 
KE '  2  L
New angular momentum L'   
f ' (2 f) 4

Example-59
A particle having mass 5kg is moving on a straight line
y = 2x + 4 with velocity 3 5 m/s. Find its angular
momentum about origin.
Sol. m = tan = 2 and C = 4
( y = mx + C) i.e. any internal intraction between part of the system
can not alter its total angular momentum.
• If a person skating on ice folds his arms then his M.I.
decreases and '' increases.
• A diver jumping from a height folds his arms and legs (I
decrease) in order to increase no. of rotation in air by
increasing ''.

d 2 8 Ii
 sin    d i
4 5 5 f

8 If
L = mvd = 5 × 3 5 × = 120 kg-m2/sec
5
Ii > If
f > i
38
Motion of System of Particles and Rigid Body

• If a person moves towards the centre of rotating Sol. Idisc  – mvR = (Idisc + mR2) '
platform then 'I' decrease and '' increase.
Idisc   m vR
 ' =
Idisc  mR 2

Example-63
A solid cylinder of mass 'M' and radius 'R' is rotating
along its axis with angular velocity without friction.
A particle of mass 'm' moving with velocity v collide
against the cylinder and sticks to its rim.After the impact
calculate angular velocity of cylinder.
Note Sol. Intial angular momentum of cylinder = I
• The angular velocity of a planet around the sun
increases when it comes near the sun.
• The speed of the innerlayers of the whirlwind in a
tornado is alarmingly high.
• If external torque on the system is zero, then the
angular momentum is conserved. However the
rotatinoal kinetic energy is not conserved.
1 2 2 1 2 2 Intial angular momentum of particle = mvR
I12 = I22  I1 1  I2 2 Before sticking total angular momentum
2 2
L1 = I + mvR
1 1
 I1  I1 1  I2  I2 2  I1 K R1 = I2 KR 2 After sticking total angular momentum
2 2

2 2 L2 = (I + mR2) '
If I1 > I2 then K R1 < K R2 L1 = L2  (I + mR2)' = I+ mvR
So if the moment of inertia decreases, the rotational I mvR
New angular velocity ' =
kinetic energy increases and vice versa. I  mR 2
1 2 1
SOLVED EXAMPLE Initial kinetic energy of system = I + mv2
2 2
1
Example-61 Final kinetic energy of system = (I + mR2)'2
2
Two wheels Aand B are placed co-axially. Wheel A has
moment of inertia IA and angular velocity A while B is Example-64
stationary. On joining them with a clutch they move
Keeping the mass of earth constant. If its radius is
jointly by angular velocity '' then M. I. of 'B' will be.
halved then the duration of the day will be.
2 2
2
2 2 R
Sol. I1 1 = I2 2  MR2 × T = M  × T
5 1 5 2  2

T1
 T2 =  T1 = 24 hr
4
T2 = 6 hr
IA  A  IA 
Sol. IA A = (IA + IB) or IB = Example-65

Explain with reason why if ice melts at pole then M.I. of
Example-62 earth increases, angular velocity  decrease and day
A cockroach of mass 'm' is moving with velocity v in will be longer.
anticlockwise direction on the rim of a disc of radius R. Sol. If ice of the pole is melts then it will come towards the
The M.I. of the disc about the axis is 'I' and it is rotating equator and moment of inertia of earth will increase
in clockwise direction with an angular velocity ''. If the because mass particles at equator are at more distant from
cockroach stops then angular velocity of disc will be: rotational axis as compare to pole.
m R 2
v Time period (T) = so if I increasesthen decreases.
 
 T increases
m R Due to increament of time period the duration of day
' and night will increase.
39
Motion of System of Particles and Rigid Body

d d
Example-66  d  (I )d  I d  I d   Id 
dt dt
A rotating table has angular velocity '' and moment of
since  is the net torque, the integral in equation (ii) is
inertia I1. A person of mass 'm' stands on centre of the total work done on the rotating rigid body.
rotating table. If the person moves a distance r along
2
its radius then what will be the final angular velocity of 1 1 2
Wtotal   I d  2 I  I1
2
2
rotating table. 1
2
Sol. Initial angular momentum = Final angular momentum
When a torque does work on a rotating rigid body, the
I1 1 kinetic energy changes by an amount equal to the work
I11 = (I1 + m r2) 2 2= done.
I1  m r 2
• Work energy theorem in rotational motion
The work done by torque = Change in kinetic energy
WORK AND POWER IN ROTATIONAL MOTION
 of rotation.
Suppose a tangential force Ftan acts at the rim of a 1 2 1 2
pivoted disk [for example, a man running while pushing W I2  I1
2 2
on a playground merry-go-round]. The change in the rotational kinetic energy of a rigid
body equals the work done by torques exerted from
outside the body. This equation is analogous to
eqaution to work-energy theorem for a particle.
• Rotational power
The power associated with the work done by a torque
acting on a rotating body :
divide both sides of equation dW = dt by the time
intervl dt during which the angular displacement occurs.
The disk rotates through an infinitesimal angle d about dW d

a fixed axis during an infinitesimal time interval dt. dt dt
 instantaneous rotational power Pr = 
The work dW done by the force Ftan while a point on
the rim moves a distance ds is dW = Ftan ds. SOLVED EXAMPLE
If d is angular displacement, then ds = R d and
 dW = Ftan R d
 Example-67
the torque due to the force Ftan is  = Ftan R A point mass is tied to one end of a cord whose other
 dW =  d ....(i) end passes through a vertical hollow tube, caught in
during an angular displacement from 1 to 2 the total one hand. The point mass is being rotated in a horizontal
work W done by the torque : circle of radius 2m with speed of 4m/s. The cord is then
2 pulled down so that the radius of the circle reduces to
W 1
 d
1m. Compute the new linear and angular velocities of
(work done by a torque) ....(ii) the point mass and compute the ratio of kinetic energies
If the torque is constant while the angle changes by a under the initial and final states.
finite amount  = 2 – 1 .
Sol. The force on the point mass due to cord is radial and
W =  (2 – 1 ) =  
hence the torque about the centre of rotation is zero.
(work done by a constant torque) ....(iii)
Therefore, the angular momentum must remain constant
The work done by a constant torque is the product of
torque and the angular displacement. as the chord is shortened.
Let  represent the net torque on the body so taht from Let mass of the particle is m and in the circle of radius r1
equation  = I. linear velocity v1 and angular velocity 1. Further let in
Assuming that the body is rigid so that the moment of a circle of radius r2 the linear velocity v2 and angular
inertia I is constant. velocity 2
 Initial angular momentum = Final angular momentum
ds
d
R R Ftan

Overhead view
of merry-go-round

40
Motion of System of Particles and Rigid Body

v1 v
 I11 = I22  mr1
2
 mr22 2 Example-70
r1 r2 If the angular momentum of a body is increased by 200 %
 r1v1 = r2v2 then find increase in its rotational kinetic energy.
Sol. L1 = L
 r1 2
v2  v1   4 = 8 m/s
r2 1 2 00
L2  L   3L
100
v1 8
and 2   = 8 rad/s Percentage increase in rotational energy
r2 1
E 2  E1  E2 
 10 0 =  E  1   10 0
1
I 2 =
FinalK.E. 2 2 2 E1  1 

InitialK.E. 1
I1 12
2  L2   9 L2 
=  22  1   1 00   2  1   1 00  8 00 %
 L1   L 
2
v 
mr22   2  2
 r2   v2  (8 )  4
2
Example-71
=
v 
2
v12 (4 )2 The power output of an automobile engine is
mr12   1  advertised to be 200 hp at 6000 rpm. What is the
 r1  corresponding torque.
Example-68
746
A thin meter scale is kept vertical by placing its one end on Sol. P  200hp  200hp   1.49  10 5 W
1 hp
floor, keeping the end in contact stationary, it is allowed to
fall. Calculate the velocity of its upper end when it hit 2
the floor . z = 6000 rev/mim = 60 00  = 628 rad/s
60
Sol. Loss in PE = gain in rotational KE
P 1.49  10 5
   237.5 N.m
 628

ROLLING
Rolling Motion
When a body performs translatory motion as well as
rotatory motion then it is known as rolling.
mg  1 2 1 m 
2
v2 The velocity of centre of mass represents linear motion
= I = × 2
2 2 2 3  while angular velocity represents rotatory motion.
For rolling without slipping (or pure rolling)
 v = 3 g v = R
Total energy in rolling = Translatory kinetic energy +
Example-69 Rotatory kinetic energy.
If the kinetic energy of a body is increased by 300%
then determine percentage increase in its angular
momentum

Sol. Percentage increase in momentum = L 2  L1  1 0 0


L1
L E
 E1 = E Total Energy
300 1 1 1 1  v2 
E2  E  E mv2  I2 = mv  mK
2 2
E r E = 4E  2
10 0 2 2 2 2 2 R 
E2  E1 1  K2 
Increase in momentum =  100 E mv2 1  
E1 2  R2 
4E  E K2 K2
=  100  100% E translation : E rotation : E total  1 : : 1 
E R2 R2
41
Motion of System of Particles and Rigid Body

K2
K2 E trans 1 E trans 1 E rotation R2
Body  2  
R2 E rotation K E total K2 E total K2
1 2 1 2
R2 R R

1 1
Ring 1 1
2 2

1 2 1
Disc 2
2 3 3

2 5 5 2
Solid sphere
5 2 7 7

2 3 3 2
Spherical shell
3 2 5 5

1 2 1
Solid cylinder 2
2 3 3

1 1
Cylinderical shell 1 1
2 2
PURE ROLLING
• If the velocity of point of contact with respect to the
surface is zero then it is known as pure rolling.
• If a body is performing rolling then the velocity of any
point of the body with respect to the surface is given
by

vA = 2vCM

vE = 2 vCM

vF = 2 vCM
   
v  v CM    R vP = 0

SOLVED EXAMPLE

Example-72
A body of radius R and mass m is placed on horizontal
rough surface with linear velocity v0 , after some time it
comes in the condition of pure rolling then determine :
vAS = vCM + R = vCM + vCM = 2vCM (i) Time t at which body starts pure rolling.

R vCM 3
vBS = vCM + = vCM + = vCM
2 2 2
vCS = vCM + 0 = vCM
R vCM 3
vDS = vCM – = vCM  = vCM
4 4 4 (ii) Linear velocity of body at time t.
vES = vCM – R = vCM – vCM = 0 (iii) Work done by frictional force in in this time t.
42
Motion of System of Particles and Rigid Body

Sol. For translatory motion v = u + at


v0
initial velocity u= v0 v  v0  g
 R2 
Let after time t pure rolling starts and at this time t final g 1  2 
velocity = v and  K 

 R2 
v0 1  2   v0
v  v0 
v0
  K 
2
R R2
1 2 1 2
K K

R2
v0  v0  v0
acceleration = a v  K2
 R2
from FBD : 1 2
Normal Reaction N = mg K
Velocity of body when pure rolling starts
Friction force f = N = mg r ma = mg v0
[ f = ma] v
K2
Retardation 1 2
a = g R
v = v0 – at Work done in sliding by frictional force :
(–ve sign for retardation)
Work done = Initial kinetic energy – Final kinetic energy
v = v0 – gt ......(i)
For rotatory motion
Work done by friction
 = 0 + t
(Initial angular velocity 0 = 0) 1 1  K2 
Wf  Mv 02  Mv 2 1  2 
 = t ......(ii) 2 2  R 
 fR mgR
   I    I  = Wf 
1
Mv 02 
1 Mv 02

Mv 02
mK 2 mK 2
2 2 K  2
 R2 
 1  2 
2 1  2 
gR  R   K 
  ......(iii)
K2
from eqn. (ii) and eqn. (iii) Example-73
A thin hollow cylinder
gR
 t ......(iv) (a) Slides without rotating with a speed v.
K2
(b) Rolls with the same speed without slipping.
 for pure rolling Find ratio of kinetic energy in the two cases.
v
v = R r   ......(v) K2
R Sol. For thin hollow cylinder 1 [same as ring]
R2
From equation (iv) and (v)
1
v gR (a) Etrans. = Mv2
 2 t or 2
R K
1  K2  1
gR t 2 (b) Erolling = Mv2 1  2  = Mv2 (1 + 1) = Mv2
v .......(vi)
2  R  2
K2 1
Mv2
substitute v from eqn. (vi) into eqn.(i) E trans. 2 1
= 2 = =1:2
E rolling Mv 2
gR 2 t v0
 v0  gt  t 
K2  R2 
g 1  2  ROLLING MOTION ON AN INCLINED PLANE
 K  A body of mass M and radius R rolling down a plane
Putting the value of t in equation (i) inclined at an angle  with the horizontal. The body
rolls without slipping.

43
Motion of System of Particles and Rigid Body

n2
s  tan 
1  n2
for cylinder
1
n2 =
2
2 1
so a = g sin  and s  tan 
3 3
The centre of mass of the body moves in a straight line. 2
for solid sphere n2 =
External forces acting on the body are : 5
Weight Mg of the body vertically downwards thorugh 5
so a= g sin 
the center of mass of the body. 7
The normal reaction N of the inclined plane. 2
and  s  tan 
The frictional force f acting upwards and parallel to the 7
inclined plane. Alternative Method
Net force acting downwards and parallel to the inclined By conservation of energy
plane = Mg sin – f
1 1 1 1  v2 
Let linear acceleration of body = a mgh = mv2  I2 = mv2  mK2  2
2 2 2 2 R 
and moment of inertia of body about horizontal axis
passing through centre of mass = I 1 2  K2 
 Ma = Mg sin – f mgh = 2 mv 1  2 
torque acting on the body  = I = R × f  R 
and h = s sin
I  Ia
 f  2 gs sin 
R R2 so v2 
(a = R) K2
so we can write 1
R2
Ia Ia
Ma  Mg sin   2  a  g sin   By using 3rd equation of motion v2 = 0 + 2as
R MR 2 Acceleration when body rolls down
g sin  g sin  g sin 
or a  a a
I K2 K2
1 1  1 2
MR 2 R2 R
g sin 
 a Time taken by the rolling body to reach the bottom
1  n2
K2 1 2 2s h
[I = MK2 and let = n2] s at  t  , s
R2 2 a sin 
frictional force f acting on the body : Time of descend when body rolls down
g sin 
f  Mg sin   M 2s  K2  2h  K2 
1 2
K2 tR = g sin  1  2  = 1  
 R  g sin 2   R2 
R
 K2 
  1 2h  K2 
2  n2  = sin  g 1  
 Mg sin   R 2   Mg sin   2   R2 
1  K  1  n 
 R 2  When body slide
Clearly the force of friction is less than the weight of K2
the body. 0
R2
The condition for rolling without slipping of a body on
an inclined plane is : 2s
Time of descend when body slides down tS =
f < s N  f   s Mg cos  g sin 
2h 1 2h
 n2  = =
 Mg sin   2 
  s Mg cos  g sin 2  sin  g
1  n  so tR > tS
44
Motion of System of Particles and Rigid Body

In next figure if v2 > v1


SOLVED EXAMPLE Then change in kinetic energy due to rolling

Example-74 1 2  K2  1  K2 
= 2 mv2 1  2   2 mv1 1  2 
2
When a sphere of moment of inertia 'I' rolls down on an  R   R 
inclined plane then percentage of rotational kinetic
energy in total energy will be:
1  K2  2
=2 
m 1   (v2  v1 )
2

1 2  R2 
I
Er 2
Sol.  100   100
Et 1 1 2
Mv  I
2

2 2 Rolling/Sliding
1 v2
MR 2  2 2 1
 5 R  100
 5  100  28%
vR
K2
2  K2  2 vS = 1 2 ,
Mv 1  2  1 r
 R  5

When a body rolls on a plane, then its rotational kinetic aR 1


energy is less than its linear kinetic energy but for a aS = K2
,
ring or hollow cylinderical shell both energies are equal. 1 2
r
If different body are rolled down on an inclined plane
then body which has
tR K2
= 1
tS r2
1

vR < vS , aR < aS , tR > tS

K2
1. Least, will reach first • For pure rolling there should be fricti on on surface
R2
because it provides torque to the body.
K2 • On smooth surface rolling does not take place.
2. Maximum, will reach last
R2 • The displacement of point of contact with respect to
K2 the surface is equal to zero in pure rolling so work
3. equal, will reach together
R2 done is equal to zero.
4. From figure  • Work is done against friction force in sliding and it is
a3 > a2 > a1 negative.
t3 < t2 < t1 • Angular momentum is conserved about point of
v1 = v2 = v3 contact.
For inclined plane :
(i) Velocity of falling and sliding bodies from inclined
plane are equal and is more than rolling body.
(ii) Acceleration is maximum in case of falling and
minimum in case of rolling.
(iii) Falling body reaches the bottom first while rolling
body is in the last.

45
Motion of System of Particles and Rigid Body

Comparision betweenformula of translatory motionand rotatory For example consider a wheel which rolls without
motion slipping. In this case the point of contact with the
Translatory Motion Rotatory Motion ground has zero velocity. Hence, this point represents

 dp  dL the ICR for the wheel. If it is imagined that the wheel is
F 
dt dt momentarily pinned at this point, the velocity of any
    point on the wheel can be found using v = r. Here r is
F  ma   I
  the distance of the point from ICR. Similarly, the kinetic
• Linear momentum (P ) Angular momentum (L) energy of the body can be assumed to be pure rotational
   
P  mv J  I about IAOR or,
• Linear kinetic energy Rotational kinetic energy
1 1 2
KE  mv2 E I
2 2
  
• Work done W  F.S Work done W  .
(constant force) (constant torque)
• Variable force Variable torque
    1
W   F.ds W   .d K I IAOR  2
2
• Power in linear motion Power in rotational motion Rotation + Translation
     Pure rotation about IAOR passing through IC
dW F.ds dW .d
P  P  1 1
KE = mv COM  ICOM
dt dt dt dt 2 2
   2 2
P  F.V P  .
1
 KE  I IAOR
2
• Work energy theorem Work energy theorem
2
in T. M. in R. M.
1 1 1 2 1 2 Location of the ICR
W mv22  mv12 W I2  I1
2 2 2 2 If the location of the ICR is unknown, it may be
• Linear impulse Angular impulse determined by using the fact that the relative position
It is product of large It is product of large vector extending from the ICR to a point is always
force for small time torque for small time

 p
perpendicular to the velocity of the point. Following
 L
F  three possibilities exist.
t t
    (i) Given the velocity of a point (normally the centre of
p  Ft = Impulse L  t mass) on the body and the angular velocity of the body
= Angular impulse If v and  are known, the ICR is located along the line
Impulse momentum Angular Impulse 
drawn perpendicular to v at P, such that the distance
theorem momentum theorem
v
from P to IC is, r  . Note that ICR lie on that side of
INSTANTANEOUS AXIS OF ROTATION 
The combined effects of translation of the centre of mass P which causes rotation about the ICR, which is
and rotation about an axis through the centre of mass 
consistent with the direction of motion caused by 
are equivalent to a pure rotation with the same angular 
and v .
speed about an axis passing through a point of zero
velocity. Such an axis is called the instantaneous axis of
rotation. (IAOR). This axis is always perpendicular to
the plane used to represent the motion and the
intersection of the axis with this plane defines the location
of instantaneous centre of zero velocity (IC).

46
Motion of System of Particles and Rigid Body

EXERCISE-I

CENTRE OF MASS Q.7 Three identical metal balls each of radius r are placed
CALCULATION OF COM touching each other on a horizontal surface such that
Q.1 The centre of mass of a body : an equilateral triangle is formed, when centres of three
(1) Lies always at the geometrical centre balls are joined. The centre of the mass of system is
(2) Lies always inside the body located at
(1) Horizontal surface
(3) Lies always outside the body
(2) Centre of one of the balls
(4) Lies within or outside the body
(3) Line joining centres of any two balls
(4) Point of intersection of the medians
Q.2 A body has its centre of mass at the origin. The x-
coordinates of the particles Q.8 Centre of mass is a point
(1) may be all positive (1) Which is geometric centre of a body
(2) may be all negative (2) From which distance of particles are same
(3) must be all non-negative (3) Where the whole mass of the body is supposed to
(4) may be positive for some particles and negative concentrated
in other particles (4) Which is the origin of reference frame

Q.3 All the particles of a body are situated at a distance Q.9 Choose the correct statement about the centre of mass
R from the origin. The distance of the centre of mass (CM) of a system of two particles
of the body from the origin is (1) The CM lies on the line joining the two particles
(1) = R (2)  R (3) > R (4)  R midway between them
(2) The CM lies on the line joining them at a point
Q.4 Where will be the centre of mass on combining two whose distance form each particle is inversely
masses m and M(M > m) : proportional to the mass of that particle
(1) towards m (2) towards M (3) The CM lies on the line joining them at a point
(3) between m and M (4) anywhere whose distance from each particle is proportional
to the square of the mass of that particle
Q.5 A uniform solid cone of height 40 cm is shown in (4) The CM is on the line joining them at a point whose
figure. The distance of centre of mass of the cone distance from each particle is proportional to the
from point B (centre of the base) is : mass of that particle

Q.10 A cricket bat is cut at the location of its centre of mass


as shown. Then

(1) The two pieces will have the same mass


(2) The bottom piece will have larger mass
(3) The handle piece will have larger mass
(4) Mass of handle piece is double the mass of bottom
piece

Q.11 The centre of mass of triangle shown in figure has


coordinates
y

(1) 20 cm (2) 10/3 cm


(3) 20/3 cm (4) 10 cm
h
Q.6 In the HCl molecule, the separation between the nuclei
of the two atoms is about 1.27 Å (1 Å = 10–10 m). The x
approximate location of the centre of mass of the b
molecule, distance from hydrogen atom assuming the h b b h
(1) x = ,y= (2) x = ,y=
chlorine atom to be about 35.5 times massive as 2 2 2 2
hydrogen is
b h h b
(1) 1Å (2) 2.5 Å (3) 1.24 Å (4) 1.5 Å (3) x = , y= (4) x = ,y=
3 3 3 3
47
Motion of System of Particles and Rigid Body

Q.12 Three identical spheres, each of mass 1 kg are kept as Q.18 If the external forces acting on a system have zero
shown in figure, touching each other, with their centres resultant, the centre of mass
on a straight line. If their centres are marked P, Q, R (1) must not move (2) must accelerate
respectively, the distance of centre of mass of the (3) may move (4) may accelerate
system from P (origin) is
Q.19 Two balls are thrown in air. The acceleration of the
centre of mass of the two balls while in air (neglect
air resistance)
(1) depends on the direction of the motion of the
balls
(2) depends on the masses of the two balls
(3) depends on the speeds of the two balls
PQ  PR  QR PQ  PR
(1) (2) (4) is equal to g
3 3
PQ  QR PR  QR Q.20 The motion of the centre of mass of a system of two
(3) (4) particles is unaffected by their internal forces :
3 3
(1) irrespective of the actual directions of the internal
Q.13 A uniform square plate ABCD has a mass of 10 kg. If forces
two point masses of 3 kg each are placed at the corners (2) only if they are along the line joining the particles
C and D as shown in the adjoining figure, then the (3) only if they are at right angles to the line joining the
centre of mass shifts to the point which is lie on - particles
y' (4) only if they are obliquely inclined to the line joining
the particles.
x' x
O Q.21 Two objects of masses 200 gm and 500 gm posses
velocities 10iˆ m/s and 3iˆ  5jˆ m/s respectively. The
D y C
(1) OC (2) OD (3)OY (4) OX velocity of their centre of mass in m/s is :

DISPLACEMENT, VELOCITY, ACCELERATION OF COM (1) 5iˆ  25jˆ (2) i  25jˆ
7
Q.14 If the momentum of a body is increased n times, its
25 5
kinetic energy increases (3) 5iˆ  ˆj (4) 25iˆ  ˆj
(1) n times (2) 2n times 7 7

(3) n times (4) n 2 times


Q.22 2 bodies of different masses of 2kg and 4kg are moving
with velocities 20m/s and 10m/s towards each other
Q.15 A bomb travelling in a parabolic path under the effect
due to mutual gravitational attraction. What is the
of gravity, explodes in mid air. The centre of mass of
velocity of their centre of mass ?
fragments will:
(1) 5 m/s (2) 6 m/s (3) 8 m/s (4) zero
(1) Move vertically upwards and then downwards
(2) Move vertically downwards
Q.23 Two bodies of masses 2 kg and 4 kg are moving with
(3) Move in irregular path
velocities 2 m/s and 10m/s respectively along same
(4) Move in the parabolic path which the unexploded
direction. Then the velocity of their centre of mass will
bomb would have travelled.
be
Q.16 A body at rest may have (1) 8.1 m/s (2) 7.3 m/s (3) 6.4 m/s (4) 5.3 m/s
(1) Energy (2) Momentum
(3) Speed (4) Velocity Q.24 Two particle of masses m1 and m2 initially at rest start
moving towards each other under their mutual force of
Q.17 Internal forces can change : attraction. The speed of the centre of mass at any time
(1) the linear momentum but not the kinetic energy t, when they are at a distance r apart, is
of the system.  m1m 2 1 
(2) the kinetic energy but not the linear momentum (1) Zero (2)  G r 2 . m  t
of the system.  1 

(3) linear momentum as well as kinetic energy of the


 m1m 2 1   mm 1 
system. (3)  G 2 . t (4)  G 1 2 2 . t
(4) neither the linear momentum nor the kinetic  r m2   r m1  m 2 
energy of the system.
48
Motion of System of Particles and Rigid Body

Q.25 A body of mass 20 kg is moving with a velocity of 2 Q.32 A bomb explodes in air in two equal fragments. If
and another body of mass 10 kg is moving with velocity one of the fragments is moving vertically upwards
V along same direction . The velocity of their centre of with velocity v0, then the other fragment is moving-
mass is (1) Vertically up with velocity v0
(1) 5/3 (2) 2/3 (3)  (4) Zero (2) Vertically downwards with velocity v0
(3) In any arbitrary direction
Q.26 The two particles X and Y, initially at rest, start moving (4) None of these
towards each other under mutual attraction. If at any
instant the velocity of X is V and that of Y is 2V, the Q.33 Two identical blocks A and B, each of mass ‘m’ resting
velocity of their centre of mass will be on smooth floor are connected by a light spring of
(1) Zero (2)V (3)2V (4) V/2 natural length L and spring constant K, with the spring
at its natural length. A third identical block ‘C’
Q.27 Two particles whose masses are 10 kg and 30 kg and (mass m) moving with a speed v along the line joining
their position vectors are ˆi  ˆj  kˆ and ˆi  ˆj  kˆ A and B collides with A. the maximum compression in
the spring is
respectively would have the centre of mass at -
m v mv mv
(iˆ  ˆj  k)
ˆ (iˆ  ˆj  k)
ˆ (1) v (2) m (3) (4)
(1)  (2) 2k 2k k 2k
2 2

(iˆ  ˆj  k)
ˆ (iˆ  ˆj  k)
ˆ Q.34 A ball of mass 3 kg collides with a wall with velocity 10
(3)  (4) m/sec at an angle of 30° and after collision reflects at
4 4
the same angle with the same speed. The change in
Q.28 Two balls A and B of masses 100 gm and 250 gm momentum of ball in MKS unit is-
respectively are connected by a stretched spring of (1) 20 (2) 30 (3) 15 (4) 45
negligible mass and placed on a smooth table. When
the balls are released simultaneously the initial Q.35 An object of mass 3m splits into three equal fragments.
acceleration of B is 10 cm/sec2 west ward. What is the Two fragments have velocities vjˆ and viˆ . The velocity
magnitude and direction of initial acceleration of the of the third fragment is
ball A -
(1) 25 cm/sec2 East ward (1) v(ˆj  ˆi) (2) v(iˆ  ˆj)
(2) 25 cm/sec2 North ward
(3) 25 cm/sec2 West ward v(iˆ  ˆj)
(3)  v(iˆ  ˆj) (4)
(4) 25 cm/sec2 South ward 2

LAW OF CONSERVATION OF MOMENTUM Q.36 A bomb at rest has mass 60 kg. It explodes and a
Q.29 Two bodies of masses m1 and m2 have equal kinetic fragment of 40 kg has kinetic energy 96 joule. Then
energies. If p1 and p2 are their respective momentum, kinetic energy of other fragment is-
then ratio p1 : p2 is equal to (1) 180 J (2) 190 J (3) 182 J (4) 192 J
(1) m1 : m2 (2) m2 : m1
Q.37 When a U238 nucleus originally at rest, decays emitting
2 2
(3) m1 : m 2 (4) m : m
1 2 an alpha particle having a speed 'u', the recoil speed
of the residual nucleus is-
Q.30 A particle of mass 4m which is at rest explodes into 4u 4u 4u 4u
three fragments. Two of the fragments, each of mass m (1) (2) – (3) (4) –
234 238 238 234
are found to move with speed v each, in mutually
perpendicular directions. The total energy released in
IDEA OF IMPULSIVE / NON IMPULSIVE FORCE
the process of explosion is-
Q.38 A force of 50 dynes is acted on a body of mass
(1) 3mv2/2 (2) mv2
5gm which is at rest for an interval of 3 sec, then
(3) 4mv2 (4) 2mv2
impulse is-
Q.31 A bullet of mass m is being fired from a stationary (1) 0.16 × 10–3 N-S (2) 0.98 × 10–3 N-S
–3
gun of mass M. If the velocity of the bullet is v, (3) 1.5 × 10 N-S (4) 2.5 × 10–3 N-S
the velocity of the gun is-
Q.39 The area of F-t curve is A, where 'F' is the force on
Mv mv one mass due to the other. If one of the colliding
(1) m  M (2) bodies of mass M is at rest initially, its speed just
M
after the collision is :
(M  m)v Mm
(3) (4) 2A
M Mv (1) A/M (2) M/A (3) AM (4)
M
49
Motion of System of Particles and Rigid Body

Q.40 A body of mass 0.5 kg is projected under gravity with Q.48 A lead ball strikes a wall and falls down, a tennis ball
a speed of 98 m/s at an angle of 60° with the vertical. having the same mass and velocity strikes the wall
The change in momentum [in magnitude] of the body and bounces back. Check the correct statement
(1) The momentum of the lead ball is greater than that
when it returns on ground is-
of the tennis ball
(1) 24.5 N-s (2) 49.0 N-s
(2) The lead ball suffers a greater change in momentum
(3) 98.0 N-s (4) 49 3 N-s compared with the tennis ball
(3) The tennis ball suffers a greater change in
Q.41 If two balls, each of mass 0.06 kg, moving in opposite momentum as compared with the lead ball
(4) Both suffer an equal change in momentum
directions with speed of 4m/s, collide and rebound with
the same speed, then the impulse imparted to each ball
Q.49 In the figure shown the block A collides head on
due to other (in kg-m/s) is : with another block B at rest. Mass of B is twice the
(1) 0.48 (2) 0.53 (3) 0.81 (4) 0.92 mass of A. The block A stops after collision. The
co-efficient of restitution is :
COLLISIONS (OBLIQUE & HEAD ON)
Q.42 A block moving in air explodes in two parts then
just after explosion
(1) the total momentum must be conserved (1) 0.5 (2) 1
(2) the total kinetic energy of two parts must be (3) 0.25 (4) it is not possible
same as that of block before explosion.
(3) the total momentum must change Q.50 A body of mass m1 collides head on elastically
with a stationary body of mass m2. If velocities
(4) the total kinetic energy must not be increased
of m1 before and after the collision are v and
–v/3 respectively then the value of m1/m2 is-
Q.43 In head on elastic collision of two bodies of equal
(1) 1 (2) 2 (3) 0.5 (4) 4
masses, it is not possible :
(1) the velocities are interchanged Q.51 Two identical smooth spheres A and B are moving
(2) the speeds are interchanged with same velocity and collides with similar spheres
(3) the momenta are interchanged C and D, then after collision- (Consider one dimensional
(4) the faster body speeds up and the slower body collision)
slows down

Q.44 A ball of mass 'm', moving with uniform speed, collides


elastically with another stationary ball. The incident
ball will lose maximum kinetic energy when the mass
of the stationary ball is
(1) m (2) 2m (3) 4m (4) infinity (1) D will move with greater speed
(2) C and D will move with same velocity v
Q.45 The coefficient of restitution e for a perfectly elastic (3) C will stop and D will move with velocity v
(4) All spheres A, B, C & D will move with velocity
collision is
v/2
(1) 1 (2) 0 (3)  (4) – 1
Q.52 When two bodies collide elastically, then
Q.46 A shell initially at rest explodes into two pieces of equal (1) Kinetic energy of the system alone is conserved
mass, then the two pieces will (2) Only momentum is conserved
(1) Be at rest (3) Both energy and momentum are conserved
(2) Move with different velocities in different directions (4) Neither energy nor momentum is conserved
(3) Move with the same velocity in opposite directions
(4) Move with the same velocity in same direction Q.53 A body of mass m having an initial velocity v, makes
head on collision with a stationary body of mass M.
Q.47 Two solid rubber balls A and B having masses 200 and After the collision, the body of mass m comes to rest
and only the body having mass M moves. This will
400 gm respectively are moving in opposite directions
happen only when
with velocity of A equal to 0.3 m/s. After collision the
(1) m >> M (2) m << M
two balls come to rest, then the velocity of B is
(1) 0.15 m/sec (2) 1.5 m/sec 1
(3) m = M (4) m = M
(3) – 0.15 m/sec (4) None of the above 2
50
Motion of System of Particles and Rigid Body

Q.54 A particle of mass m moving with horizontal speed Q.61 A bullet of mass m = 50 gm strikes a sand bag of
6 m/sec as shown in figure. If m <<M then for one mass M = 5 kg hanging from a fixed point, with a
dimensional elastic collision, the speed of lighter 
horizontal velocity v p . If bullet sticks to the sand
particle after collision will be
u1 = 6 m/s
bag then the ratio of final & initial kinetic energy of
u2 = 4 m/s
m M the bullet is (approximately) :
(1) 10–2 (2) 10–3 (3) 10–6 (4) 10–4
(1) 2m/sec in original direction
(2) 2 m/sec opposite to the original direction Q.62 There are hundred identical sliders equally spaced
(3) 4 m/sec opposite to the original direction on a frictionless track as shown in the figure. Initially
(4) 4 m/sec in original direction all the sliders are at rest. Slider 1 is pushed with
velocity v towards slider 2. In a collision the sliders
Q.55 Two equal masses m1 and m2 moving along the same stick together. The final velocity of the set of
straight line with velocities + 3 m/s and – 5 m/s hundred stucked sliders will be :
respectively collide elastically. Their velocities after
the collision will be respectively
(1) + 4 m/s for both (2) – 3 m/s and +5 m/s
(3) – 4 m/s and + 4 m/s (4) – 5 m/s and + 3 m/s v v
(1) (2) (3) zero (4) v
99 100
Q.56 A body falls on a surface of coefficient of restitution
0.6 from a height of 1 m. Then the body rebounds to a Q.63 A mass of 10 gm, moving horizontally with a velocity
height of of 100 cm/sec, strikes the bob of a pendulum and sticks
to it. The mass of the bob is also 10 gm. The maximum
(1) 0.6 m (2) 0.4 m (3) 1 m (4) 0.36 m height to which the system can be raised is (g = 10 m/
sec2)
Q.57 A ball of weight 0.1 kg coming with speed 30 m/s strikes
with a bat and returns in opposite direction with speed
40 m/s, then the impulse is (Taking final velocity as
positive)
(1) 0.1  (40)  0.1  (30) (2) 0.1  (40)  0.1  (30)
(3) 0.1  (40)  0.1  (30) (4) 0.1  (40)  0.1  (20)

Q.58 A ball of mass m falls vertically to the ground from a (1) Zero (2) 5 cm (3) 2.5 cm (4) 1.25 cm
height h1 and rebound to a height h2. The change in
momentum of the ball on striking the ground is Q.64 A space craft of mass M is moving with velocity V and
suddenly explodes into two pieces. A part of it of mass
(1) mg (h1 – h2) (2) m( 2gh1  2gh 2 )
m becomes at rest, then the velocity of other part will
(3) m 2g(h1  h 2 ) (4) m 2g (h1  h 2 ) be
MV MV mV (M  m)V
(1) (2) (3) (4)
Q.59 A sphere of mass M moving with velocity u collides Mm Mm Mm m
head on elastically with a sphere of mass m at
rest. After collision their respective velocities are Q.65 Two bodies having same mass 40 kg are moving in
opposite directions, one with a velocity of 10 m/s and
V and v. The value of v is-
the other with 7 m/s. If they collide and move as one
M m body, the velocity of the combination is
(1) 2u (2) 2u (1) 10 m/s (2) 7 m/s (3) 3 m/s (4) 1.5 m/s
m M

2u 2u Q.66 A particle falls from a height h upon a fixed horizontal


(3) 1  m M (4) 1  M m plane and rebounds. If e is the coefficient of restitution,
the total distance travelled before rebounding has
Q.60 Two spheres approaching each other collides stopped is
elastically. Before collision the speed of A is 5m/s 1 e2  1e2 
and that of B is 10m/s. Their masses are 1kg and (1) 
h 
 (2) 
h 

1e 1 e
2 2
 
0.5kg. After collision velocities of A and B are
respectively- 
h 1e  2  
h 1e  2 
(1) 5 m/s –10 m/s (2) 10 m/s, –5 m/s (3) 2  2  (4) 2  2 
1  e  1e 
(3) –10 m/s, –5 m/s (4) –5 m/s, 10 m/s
51
Motion of System of Particles and Rigid Body

Q.67 The bob A of a simple pendulum is released when the Q.72 A body of mass m moving with a constant velocity v
string makes an angle of 45 o with the vertical. It hits hits another body of the same mass moving with the
same velocity v but in the opposite direction and sticks
another bob B of the same material and same mass to it. The velocity of the compound body after collision
kept at rest on the table. If the collision is elastic is
(1) v (2) 2v
O
(3) Zero (4) v/2
45o
Q.73 A ball is dropped from height 10 m. Ball is embedded in
sand 1 m and stops, then
(1) Only momentum remains conserved
A
(2) Only kinetic energy remains conserved
B (3) Both momentum and K.E. are conserved
(4) Neither K.E. nor momentum is conserved
(1) Both A and B rise to the same height
(2) Both A and B come to rest at B Q.74 A body of mass 2kg is moving with velocity 10 m/s
(3) Both A and B move with the same velocity of A towards east. Another body of same mass and same
(4) A comes to rest and B moves with the velocity of A velocity moving towards north collides with former
and coalsces and moves towards north-east. Its
Q.68 A ball of mass m moving with velocity V, makes a head velocity is
on elastic collision with a ball of the same mass moving (1) 10 m/s (2) 5 m/s (3) 2.5 m/s (4) 5 2 m / s
with velocity 2V towards it. Taking direction of V as
positive velocities of the two balls after collision are
(1) V and 2 V (2) 2 V and V Q.75 A body of mass m 1 is moving with a velocity V. It
(3) V and 2 V (4) 2 V and V collides with another stationary body of mass m 2 .
They get embedded. At the point of collision, the
Q.69 A body of mass M 1 collides elastically with another velocity of the system
(1) Increases
mass M 2 at rest. There is maximum transfer of energy (2) Decreases but does not become zero
when (3) Remains same
(1) M 1  M 2 (4) Become zero
(2) M 1  M 2
VARIABLE MASS SYSTEM
(3) M 1  M 2 Q.76 If the force on a rocket which is ejecting gases with a
(4) Same for all values of M 1 and M 2 relative velocity of 300 m/s, is 210 N. Then the rate of
combustion of the fuel will be :
Q.70 A mass ‘m’ moves with a velocity ‘v’ and collides (1) 10.7 kg/sec (2) 0.07 kg/sec
inelastically with another identical mass. After collision (3) 1.4 kg/sec (4) 0.7 kg/sec
v
the Ist mass moves with velocity in a direction Q.77 A rocket with a life-off mass 3.5 × 104 kg is blasted
3
perpendicular to the initial direction of motion. Find upwards with an initial acceleration of 10 m/s2. The
the speed of the 2nd mass after collision initial thrust of the blast is-
(1) 14.0 × 105 N (2) 1.76 × 105 N
v 5
(3) 3.5 × 10 N (4) 7.0 × 105 N
3
v At rest
m m Q.78 Fuel is consumed at the rate of 100 kg/sec. in a rocket.
before collision After collision The exhaust gases are ejected as a speed of 4.5 ×
104 m/s. What is the thrust experienced by the rocket-
2 v (1) 3 × 106 N (2) 4.5 × 106 N
(1) v (2) (3) v (4) 3v
6
(3) 6 × 10 N (4) 9 × 106 N
3 3
Q.79 A 6000 kg rocket is set for vertical firing. If the exhaust
Q.71 A completely inelastic collision is one in which the speed is 1000 m/sec. How much gas must be ejected
two colliding particles each second to supply the thrust needed to give
(1) Are separated after collision the rocket an initial upward acceleration of 20 m/sec²-
(2) Remain together after collision (consider g = 9.8 msec–2 acceleration due to gravity)
(3) Split into small fragments flying in all directions (1) 92.4 kg/sec (2) 178.8 kg/sec
(4) None of the above (3) 143.2 kg/sec (4) 47.2 kg/sec
52
Motion of System of Particles and Rigid Body

ROTATIONAL MOTION Q.88 A particle is moving with a constant angular velocity


ANGULAR DISPLACEMENT, VELOCITY & about an exterior axis. Its linear velocity will depend
ACCELERATION upon -
Q.80 The shaft of a motor rotates at a constant angular (1) perpendicular distance of the particle form the axis
velocity of 3000 rpm. The radians it has turned in 1 sec (2) the mass of particle
are (3) angular acceleration of the particle
(1) 1000  (2) 100  (3)  (4) 10  (4) the linear acceleration of particle

Q.81 The angular speed of second hand of a clock is Q.89 A chain couples and rotates two wheels in a bicycle.
(1) (1/60) rad/s (2) (/60) rad/s The radii of bigger and smaller wheels in a bicycle.
(3) (2/60) rad/s (4) (360/60) rad/s The radii of bigger and smaller wheels are 0.5m and
0.1m respectively. The bigger wheel rotates at the rate
of 200 rotations per minute, then the rate of rotation of
Q.82 A pulley one meter in diameter rotating at 600
smaller wheel will be -
revolutions a minute is brought to rest in 80 see by a
(1) 1000 rpm (2) 50/3 rpm
constant force of friction on its shaft. How many
(3) 200 rmp (4) 40 rpm
revolutions does it make before coming to rest
(1) 200 (2) 300 (3) 400 (4) 500
Q.90 A body is in pure rotation. The linear speed  of a
particle, the distance r of the particle from the axis and
Q.83 Two bodies of mass 10 kg and 5 kg moving in concentric
the angular velocity  of the body are related as
orbits of radius r1 and r2 such that their periods are
same. The ratio of centripetal accelerations is v
= . Thus-
(1) r1/r2 (2) r2/r1 (3) (r1/r2)3 (4) (r2/r1)2 r

Q.84 A wheel starts rotating from rest and attains an angular 1


(1)  (2)  r
velocity of 60 rad/sec in 5 seconds. The total angular r
displacement in radians will be- (3) = 0 (4)  is independent of r.
(1) 60 (2) 80 (3) 100 (4) 150
MOMENT OF INERTIA
Q.85 A body rotates at 300 rotations per minute. The value Q.91 Four masses are fixed on a massless rod as shown in
(in radian) of the angle described in 1 sec is- fig. The moment of inertia about the axis P is about
(1) 5 (2) 5 (3) 10 (4) 10
P
Q.86 Figure shows a small wheel fixed coaxially on a bigger
2 kg 5 kg 5 kg 2 kg
one of double the radius. The system rotates about
the common axis. The strings supporting A and B do
0.2 m 0.2 m 0.2 m 0.2 m
not slip on the wheels. If x and y be the distances
travelled by A and B in the same time interval, then-
(1) 2 kg m2 (2) 1 kg m2 (3) 0.5 kg m2(4) 0.3 kg m2

Q.92 By the theorem of perpendicular axes, if a body be in


X-Z-plane then :-
(1) Ix – Iy = Iz (2) Ix + Iz = Iy
(3) Ix + Iy = Iz (4) Iy + Iz = Ix
A
B Q.93 The theorem of perpendicular axes is not applicable
for determination of moment of inertia along the
(1) x = 2y (2) x = y diameter, for which of the following body :
(3) y = 2x (4) None of these (1) sphere (2) disc (3) ring (4) blade

Q.87 The linear and angular acceleration of a particle are Q.94 The axis X and Z in the plane of a disc are mutually
10 m/sec2 and 5 rad/sec2 respectively. It will be at a perpendicular and Y-axis is perpendicular to the plane
distance from the axis of rotation. of the disc. If the moment of inertia of the body about
X and Y axes is respectively 30 kg m2 and 40 kg m2
1
(1) 50 m (2) m (3) 1 m (4) 2 m then M.I. about Z-axis in kg m2 will be :-
2
(1) 70 (2) 50 (3) 10 (4) Zero
53
Motion of System of Particles and Rigid Body

Q.95 The moment of inertia of a ring of mass m and radius r Q.103 A stone of mass 4 kg is whirled in a horizontal circle of
about an axis, passing through the centre and radius 1m and makes 2 rev/sec. The moment of inertia
perpendicular to the plane of the ring is of the stone about the axis of rotation is-
(1) (1/2) mr2 (2) mr2 (1) 64 kg × m2 (2) 4 kg × m2
(3) (1/4) mr2
(4) (3/4) mr2 (3) 16 kg × m2 (4) 1 kg × m2

Q.96 Two rods each of mass m and length  are joined at Q.104 A solid cylinder of mass 20kg has length 1 m and
the centre to form a cross. The moment of inertia of radius 0.2 m. Then its moment of inertia (in kg-m2)
this cross about an axis passing through the common about its geometrical axis is :–
centre of the rods and perpendicular to the plane (1) 0.8 kg-m2 (2) 0.4 kg-m2
formed by them, is : (3) 0.2 kg-m2 (4) 20.2 kg-m2
m 2 m 2 m 2 m 2
(1) (2) (3) (4) Q.105 In an arrangement four particles, each of mass 2 gram
12 6 3 2 are situated at the coordinate points (3, 2, 0),
(1, –1, 0), (0, 0, 0) and (–1, 1, 0). The moment of inertia
Q.97 The moment of inertia of a body about a given axis of this arrangement about the Z-axis will be-
of rotation depends upon :- (1) 8 units (2) 16 units (3) 43 units (4) 34 units
(1) the distribution of mass
(2) distance of particle of body from the axis of Q.106 The moment of inertia of a circular ring (radius R, mass
rotation M) about an axis which passes through tangentially
(3) shape of the body and perpendicular to its plane will be:–
(4) all of the above
MR 2 3
(1) (2) MR2 (3) MR2 (4) 2MR2
Q.98 For the same total mass which of the following will 2 2
have the largest moment of inertia about an axis
passing through the centre of mass and perpendicular Q.107 A solid sphere and a hollow sphere of the same mass
to the plane of the body have the same M.I. about their respective diameters.
(1) A disc of radius a The ratio of their radii will be :
(2) A ring of radius a
(1) 1 : 2 (2) 3: 5
(3) A square lamina of side 2a
(4) Four rods forming a square of side 2a (3) 5: 3 (4) 5 : 4

Q.99 The moment of inertia of a thin uniform circular disc Q.108 Three solid spheres of mass M and radius R are
about one of the diameters is I. Its moment of inertia shown in the figure. The moment of inertia of the
about an axis perpendicular to the .plane of disc and system about XX' axis will be :-
passing through its centre is

 2I
X
(1) (2) 2I (3) I/2 (4) I / 2

Q.100 The moment of inertia of a uniformsemicircular wire of


mass M and radius r about a line perpendicular to the
plane of the wire through the centre is X'

1 2 1 2 2 2 14
(1) Mr2 (2) Mr (3) Mr (4) Mr 7
2 4 5 (1) MR2 (2) MR2
2 5

Q.101 The density of a rod AB increases linearly from A to B. 16 21


(3) MR2 (4) MR2
Its midpoint is O and its centre of mass is at C. Four 5 5
axes pass through A, B, Oand C, all perpendicular to
the length of the rod. The moments of inertia of the Q.109 The moment of inertia of a square lamina about the
rod about these axes are IA, IB, IO and IC respectively. perpendicular axis through its centre of mass is 20 kg-
(1) IA > IB (2) IA < IB (3) IO > IC (4) IO < IC m2. Then, its moment of inertia about an axis touching
its side and in the plane of the lamina will be :-
Q.102 Moment of momentum is called (1) 10 kg-m2 (2) 30 kg-m2
(1) Inertia (2) Momentumof inertia (3) 40 kg-m 2
(4) 25 kg-m2
(3) Torque (4)Angular momentum
54
Motion of System of Particles and Rigid Body

Q.110 Three rings, each of mass P and radius Q are arranged as Q.116 The moment of inertia of a ring of mass M and radius
shown in the figure. The moment of inertia of the R about PQ axis will be :
arrangement about YY’ axis will be-
D P
Y M
1 2
Q Q R
P P O O'

Q
D' Q

3 P
Y' MR 2 3
(1) MR2 (2) (3) MR2 (4) 2MR
7 2 2 2
(1) PQ2 (2) PQ2
2 7
Q.117 Four point masses (each of mass m) are arranged it
2 5
(3) PQ2 (4) PQ2 the X-Y plane the moment of inertia of this array of
5 2 masses about Y-axis is

Q.111 The moment of inertia of a rod of mass M and length Y


a (a,a)
L about an axis passing through one edge and
a (2a,0)
perpendicular to its length will be :– O a X
(0,0) a
ML2 ML2 ML2
(1) (2) (3) (4) ML2 (a,–a)
12 6 3
(1) ma2 (2) 2ma2 (3) 4ma2 (4) 6ma2
Q.112 Three thin uniform rods each of mass M and length L
and placed along the three axis of a Cartesian Q.118 An equilateral traingular wire frame is made from 3 rods
coordinate system with one end of each rod at the of equal mass and length  each. The frame is rotated
origin. The M.I. of the system about z-axis is- about an axis perpendicular to the plane of the frame
and passing through its end. What is the radius of
ML2 2ML2 gyration of the frame ?
(1) (2)
3 3

2 (1) (2)
ML 2
(3) (4) ML2
6
 
(3) (4)
Q.113 Moment of inertia : 2 2 3
(1) is a vector quantity (2) is a scalar quantity
(3) is a tensor quantity (4) can not be calculate FIXED AXIS ROTATION + TOPPLING
Q.119 For a system to be in equilibrium, the torques acting
Q.114 Four particles each of mass m are placed at the corners on it must balance. This is true only if the torques are
of a square of side length .The radius of gyration of taken about
the system about an axis perpendicular to the square (1) the centre of the system
and passing through centre is :– (2) the centre of mass of the system
  (3) any point on the system
(1) (2) (3)  (4)  2 (4) any point on the system or outside it
2 2
Q.120 A rectangular block has a square base measuring a × a,
Q.115 The M.I. of a thin rod of length  about the and its height is h. It moves on a horizontal surface in
perpendicular axis through its centre is I. The M.I. of a direction perpendicular to one of the edges h being
the square structure made by four such rods about vertical. The coefficient of friction is µ. It will topple if
a perpendicular axis to the plane and through the
centre will be :- h a 2a a
(1)   (2)   (3)   (4)  
(1) 4I (2) 8I (3) 12I (4) 16I a h h 2h

55
Motion of System of Particles and Rigid Body

Q.121 A wheel has moment of inertia 5 × 10–3 kg m2 and is 


Q.130 Let F be a force acting on a particle having position
making 20 rev/sec. The torque needed to stop it in 10  
sec is .......... × 10–2 N-m : vector r . Let  be the torque of this force about the
(1) 2 (2) 2.5 (3) 4 (4) 4.5 origin, then-
   
(1) r .  = 0 and F .  = 0
Q.122 A force of (2î  4 ĵ  2k̂ ) Newton acts at a point    
(2) r .  = 0 and F .   0
(3î  2 ĵ  4k̂ ) metre from the origin. The magnitude    
(3) r .   0 and F .   0
of torque is -    
(1) zero (2) 24.4 N-m (4) r .   0 and F .   0
(3) 0.244 N-m (4) 2.444 N-m
ENERGY ANALYSIS
Q.123 Rate of change of angular momentum with respect to Q.131 A ring of radius r and mass m rotates about an axis
time is proportional to : passing through its centre and perpendicular to its
(1) angular velocity (2) angular acceleration plane with angular velocity . Its kinetic energy is
(3) moment of inertia (4) torque
1 1 2 2
(1) mr (2) mr2 (3) mr22 (4) mr 
Q.124 When constant torque is acting on a body then :– 2 2
(1) body maintain its state or moves in straight line
with same velocity Q.132 A rod of length L is hinged at one end. It is brought to
(2) acquire linear acceleration a horizontal position and released. The angular velocity
(3) acquire angular acceleration of the rod when it is in vertical position is
(4) rotates with a constant angular velocity
(1) 2g / L (2) 3g / L (3) g / 2L (4) g / L

Q.125 The angular velocity of a body is   2î  3 ĵ  4k̂
Q.133 Two bodies A and B having same angular momentum

and a torque   î  2 ĵ  3k̂ acts on it. The and IA > IB, then the relation between (K.E.)A and (K.E.)B
will be :–
rotational power will be-
(1) 20 watt (2) 15 watt (1) (K.E.)A > (K.E.)B (2) (K.E.)A = (K.E.)B
(3) (K.E.)A < (K.E.)B (4) (K.E.)A  (K.E.)B
(3) 17 watt (4) 14 watt
ANGULAR MOMENTUM
Q.126 If I = 50 kg-m2, then how much torque will be applied to Q.134 If a running boy jumps on a rotating table which of the
stop it in 10 sec. Its initial angular speed is 20 rad/sec.: following is conserved
(1) 100 N-m (2) 150 N-m (1) linear momentum (2) kinetic energy
(3) 200 N-m (4) 250 N-m (3) Angular momentum (4) neither of the above

Q.127 A torque of 2 newton-m produces an angular Q.135 A thin circular ring of mass M and radius r is rotating
acceleration of 2 rad/sec2 a body. If its radius of about its axis with a constant angular velocity . Two
gyration is 2m, its mass will be - objects, each of mass m are attached gently to the
(1) 2 kg (2) 4 kg (3) 1/2 kg (4) 1/4 kg opposite ends of a diameter of the ring. The wheel
now rotates with an angular velocity
Q.128 A particle is at a distance r from the axis of rotation. A
given torque  produces some angular acceleration in M   M  2m 
(1)  M  m 
it. If the mass of the particle is doubled and its distance (2)
 M  2m 
from the axis is halved, the value of torque to produce
the same angular acceleration is - M   M  2m 
(1) /2 (2)  (3) 2 (4) 4 (3)  M  2m  (4)
M
Q.129 For rotational motion, the Newton's second law of
motion is indicated by :- Q.136 A rotating table completes one rotation is 10 sec. and

its moment of ineratia is 100 kg-m2. A person of 50 kg.
 mass stands at the centre of the rotating table. If the
dp
(1)   
 
(2) F 
dt person moves 2m. from the centre, the angular velocity
of the rotating table in rad/sec. will be:


dJ   2 20  2
(3)   (4) F12  F21 (1) (2) (3) (4) 2
dt 30 30 3
56
Motion of System of Particles and Rigid Body

Q.137 A circular hoop of mass m, and radius R rests flat on a Q.143 Two wheels P and Q are mounted on the same axle.
horizontal frictionless surface. A bullet, also of mass The moment of inertia of P is 6 kg-m2 and it is rotating
m, and moving with a velocity v, strikes the hoop and at 600 rotations per minute and Q is at rest. If the two
gets embedded in it. The thickness of the hoop is much are joined by means of a clutch then they combined
smaller than R. The angular velocity with which the and rotate at 400 rotations per minute. The moment of
system rotates after the bullet strikes the hoop is inertia of Q will be -
m (1) 3 kg-m2 (2) 4 kg-m2
2
(3) 5 kg-m (4) 8 kg-m2
R
Q.144 The graph between the angular momentum J and
angular velocity  will be :-
m v
V V 2V 3V J J
(1) (2) (3) (4)
4R 3R 3R 4R (1) (2)
 
Q.138 A body of mass m is moving with a constant velocity
along a line parallel to the x-axis, away from the origin.
Its angular momentumwith respect to the origin
J J
(1) is zero (2) remains constant
(3) (4)
(3) goes on increasing (4) goes on decreasing
 
Q.139 In the fig. (a) half of the meter scale is made of wood
while the other half of steel. The wooden part is COMBINED ROTATION AND TRANSLATION
pivoted at O. A force F is applied at the end of steel Q.145 If a spherical ball rolls on a table without slipping, the
part. In figure (b) the steel part is pivoted at O' and fraction of its total kinetic energy associated with
the same force is applied at the wooden end:- rotation is
wood steel steel wood (1) 3/5 (2) 2/7 (3) 2/5 (4) 3/7

O P O' P' Q.146 The speed of a homogeneous solid sphere after rolling
down an inclined plane of vertical height h, from rest
(a) F (b) F without sliding is
(1) more angular acceleration will be produced in (a) (1) gh (2)  g / 5 gh
(2) more angular acceleration will be produced in (b)
(3) same angular acceleration will be produced in both (3)  4 / 3 gh (4) (10 / 7) gh
conditions
(4) information is incomplete
Q.147 The rotational kinetic energy of a body is E. In the
absence of external torque, if mass of the body is
Q.140 A uniform heavy disc is rotating at constant angular
halved and radius of gyration doubled, then its
velocity () about a vertical axis through its centre O.
rotational kinetic energy will be :-
Some wax W is dropped gently on the disc. The angular
(1) 0.5E (2) 0.25E (3) E (4) 2E
velocity of the disc-
(1) does not change (2) increases
Q.148 A ring is rolling without slipping. Its energy of
(3) decreases (4) becomes zero
translation is E. Its total kinetic energy will be :-
(1) E (2) 2E (3) 3E (4) 4E
Q.141 If torque on a body is zero, then which is conserved:
(1) force (2) linear momentum
Q.149 One hollow and one solid cylinder of the same outer
(3) angular momentum (4) angular impulse
radius rolls down on a rough inclined plane. The foot
of the inclined plane is reached by
Q.142 A girl sits near the edge of a rotating circular platform.
(1) solid cylinder earlier
If the girl moves from circumference towards the centre
(2) hollow cylinder earlier
of the platform then the angular velocity of the platform
(3) simultaneously
will-
(4) the heavier earlier irrespective of being solid or
(1) decrease (2) increase
hollow
(3) remain same (4) becomes zero
57
Motion of System of Particles and Rigid Body

Q.150 If a solid sphere, disc and cylinder are allowed to roll Q.157 A ring takes time t1 and t2 for sliding down and rolling
down an inclined plane fromthe same height down an inclined plane of length L respectively for
(1) cylinder will reach the bottom first reaching the bottom. The ratio of t1 and t2 is :-
(2) disc will reach the bottom first (1) 2 : 1 (2) 1 : 2 (3) 1 : 2 (4) 2 : 1
(3) sphere will reach the bottom first
(4) all will reach the bottom at the same time Q.158 A person is standing on the edge of a circular platform,
which is moving with constant angular speed about an
Q.151 A uniform thin ring of mass 0.4 kg rolls without slipping axis passing through its centre and perpendicular to the
on a horizontal surface with a linear velocity of 10 cm/ plane of platform. If person is moving along any radius
s. The kinetic energy of the ring is towards axis of rotation then the angular velocity will :
–3
(1) 4 × 10 joules
–2
(2) 4 × 10 joules (1) decrease (2) remain unchanged
–3
(3) 2 × 10 joules
–2
(4) 2 × 10 joules (3) increase (4) data is insufficient

Q.152 A solid sphere rolls down without slipping two EXERCISE-II


different inclined planes of the same height but of
different inclinations CENTRE OF MASS
(1) in both cases the speeds and time of descend will Q.1 A thin uniform wire is bent to form the two equal sides
be same AB and AC of triangle ABC, where AB = AC = 5 cm.
(2) the speeds will be same but time of descend will be The third side BC, of length 6cm, is made from uniform
different wire of twice the density of the first. The distance of
(3) the speeds will be different but time of descend will centre of mass from A is :
be same 34 11 34 11
(4) speeds and time of descend both will be different (1) cm (2) cm (3) cm (4) cm
11 34 9 45

Q.153 A spherical shell and a solid cylinder of same radius Q.2 The centre of mass of a system of particles is at the
rolls down an inclined plane. The ratio of their origin. From this we conclude that
accelerations will be:- (1) the number of particles on positive x-axis is equal to
(1) 15:14 (2) 9:10 (3) 2:3 (4) 3:5 the number of particles on negative x-axis
(2) the total mass of the particles on positive x-axis is
Q.154 A solid homogeneous sphere is moving on a rough same as the total mass on negative x-axis
horizontal surface, partly rolling and partly sliding. (3) the number of particles on X-axis may be equal to
During this kind of motion of the sphere the number of particles on Y-axis.
(1) total kinetic energy is conserved (4) if there is a particle on the positive X-axis, there
must be at least one particle on the negative X-axis.
(2) angular momentum of the sphere about the point of
contact with the plane is conserved
Q.3 The centre of mass of two particles lies
(3) only the rotational kinetic energy about the centre (1) on the line perpendicular to the line joining the
of mass is conserved particles
(4) angular momentum about centre of mass is (2) on a point outside the line joining the particles
conversed (3) on the line joining the particles.
(4) none of the above .
Q.155 A ring of mass M is kept on a horizontal rough surface.
A force F is applied tangentially at its rim as shown. Q.4 A body of mass 1 kg moving in the x-direction,
The coefficient of friction between the ring and the 1
surface is . Then suddenly explodes into two fragments of mass kg
8
(1) friction will act in the forward direction
7
(2) friction will act in the backward direction and kg. An instant later, the smaller fragment is 0.14
8
(3) frictional force will not act
m above the x-axis. The position of the heavier fragment
(4) frictional force will be Mg.
is -
Q.156 A disc is rolling on an inclined plane without slipping 1 1
(1) m above x-axis (2) m below x-axis
then what fraction of its total energy will be in form 50 50
of rotational kinetic energy :
7 7
(1) 1 : 3 (2) 1 : 2 (3) 2 : 7 (4) 2 : 5 (3) m below x-axis (4) m above x-axis
50 50
58
Motion of System of Particles and Rigid Body

Q.5 A man of mass M stands at one end of a plank of length Q.11 Internal forces in a system can change
L which lies at rest on a frictionless surface. The man (1) linear momentum only
walks to other end of the plank. If the mass of the plank (2) kinetic energy only
M (3) both kinetic energy and linear momentum
is , then the distance that the man moves relative to
3 (4) neither the linear momentum nor the kinetic energy
ground is : of the system.
3L L 4L L
(1) (2) (3) (4)
4 4 5 3 Q.12 A man of mass 'm' climbs on a rope of length L
suspended below a balloon of mass M. The balloon is
Q.6 Two balls A and B of masses 100gm and 250 gm stationary with respect to ground. If the man begins to
respectively are connected by a stretched spring of climb up the rope at a speed vrel (relative to rope). In
negligible mass and placed on a smooth table. When what direction and with what speed (relative to ground)
the balls are released simultaneously the initial
will the balloon move?
acceleration of B is 10 cm/sec2 west ward. What is the
magnitude and direction of initial acceleration of the mv rel Mv rel
(1) downwards, (2) upwards,
ball A- mM mM
(1) 25 cm/sec2 Eastward mv rel (M  m)v rel
(2) 25 cm/sec2 North ward (3) downwards, (4) downwards,
M M
(3) 25 cm/sec2 West ward
(4) 25 cm/sec2 South ward
Q.13 There are some passengers inside a stationary railway
Q.7 A particle of mass 3m is projected from the ground at compartment. The track is frictionless. The centre of
some angle with horizontal. The horizontal range is R. mass of the compartment itself (without the passengers)
At the highest point of its path it breaks into two pieces is C1, while the centre of mass of the compartment plus
m and 2m. The smaller mass comes to rest and larger passengers system is C2. If the passengers move about
mass finally falls at a distance x from the point of inside the compartment along the track.
projection where x is equal to
(1) both C1 and C2 will move with respect to the ground
3R 3R 5R (2) neither C1 nor C2 will move with respect to the ground
(1) (2) (3) (4) 3R
4 2 4 (3) C1 will move but C2 will be stationary with respect to
the ground
Q.8 Two particles of mass 1 kg and 0.5 kg are moving in the (4) C2 will move but C1 will be stationary with respect to
same direction with speed of 2m/s and 6m/s respectively the ground
on a smooth horizontal surface. The speed of centre of
mass of the system is :
Q.14 A 500 kg boat has an initial speed of 10 ms–1 as it passes
10 10 11 12
(1) m/s (2) m/s (3) m/s (4) m/s under a bridge. At that instant a 50 kg man jumps
3 7 2 3
straight down into the boat from the bridge. The speed
of the boat after the man and boat attain a common
Q.9 Two particles of equal mass have initial velocities 2 î
speed is
ms–1 and 2 ĵ ms–1 . First particle has a constant
100 –1 10
acceleration ( î  ĵ) ms–2 while the acceleration of the (1) ms (2) ms–1
11 11
second particle is always zero. The centre of mass of
the two particles moves in 50 –1 5
(3) ms (4) ms–1
(1) Circle (2) Parabola 11 11
(3) Ellipse (4) Straight line
Q.15 A shell is fired from a canon with a velocity V at an
Q.10 Two particles having mass ratio n : 1 are interconnected angle  with the horizontal direction. At the highest
by a light inextensible string that passes over a smooth point in its path, it explodes into two pieces of equal
pulley. If the system is released, then the acceleration
masses. One of the pieces come to rest. The speed of
of the centre of mass of the system is :
the other piece immediately after the explosion is
 n 1
2

(1) (n – 1)2 g (2)   g


 n 1  (1) 3V cos (2) 2V cos

 n 1   n 1  3
2

(3)  (4)  g (3) V cos (4) V cos


 g
 n 1  n 1  2

59
Motion of System of Particles and Rigid Body

Q.16 A small sphere is moving at a constant speed in a vertical Q.23 A super-ball is to bounce elastically back and forth
circle. Below is a list of quantities that could be used to between two rigid walls at a distance d from each other.
describe some aspect of the motion of the sphere Neglecting gravity and assuming the velocity of super-
I - kinetic energy ball to be v0 horizontally, the average force being exerted
II - gravitational potential energy by the super-ball on each wall is :
III - momentum 1 mv 20 mv 20 2mv 20 4mv 20
Which of these quantities will change as this sphere (1) (2) (3) (4)
2 d d d d
moves around the circle ?
(1) I and II only (2) I and III only Q.24 A ball of mass 50 gm is dropped from a height h = 10 m.
(3) III only (4) II and III only It rebounds losing 75 percent of its kinetic energy. If it
remains in contact with the ground for t = 0.01 sec.,
Q.17 A bomb at rest explodes into two parts of masses m1 the impulse of the impact force is : (take g = 10 m/s2)
and m2 . If the momentums of the two parts be p1 and (1) 1.3 N–s (2) 1.06 N-s
p2, then their kinetic energies will be in the ratio of- (3) 1300 N–s (4) 105 N–s
(1) m1 / m2 (2) m2 / m1
(3) p1 / p2 (4) p2 / p1 Q.25 A force exerts an impulse I on a particle changing its
speed from u to 2u. The applied force and the initial
velocity are oppositely directed along the same line.
Q.18 Conservation of linear momentum is equivalent to-
The work done by the force is
(1) Newton's second law of motion
(2) Newton's first law of motion 3 1
(1) Iu (2) Iu (3) Iu (4) 2 Iu
(3) Newton's third law of motion 2 2
(4) Conservation of angular momentum.
Q.26 A particle of mass 4m which is at rest explodes into
Q.19 A body of mass m collides against a wall with the three fragments. Two of the fragments each of mass m
are found to move with a speed 'v' each in mutually
velocity  and rebounds with the same speed. Its
perpendicular directions. The minimum energy released
change of momentum is-
in the process of explosion is :
(1) 2 m (2) m (3) – m (4) 0 (1) (2/3) mv2 (2) (3/2) mv2
2
(3) (4/3) mv (4) (3/4) mv2
Q.20 A bomb initially at rest explodes by it self into three
equal mass fragments. The velocities of two fragments Q.27 A bullet of mass m moving vertically upwards

are (3 i +2 j ) m/s and (– i – 4 j ) m/s. The velocity of


 instantaneously with a velocity 'u' hits the hanging
block of mass 'm' and gets embedded in it. As shown in
the third fragment is (in m/s) - the figure the height through which block rises after
(1) 2 i + 2 j (2) 2 i – 2 j
the collision, assume sufficient space above block :

(3) – 2 i + 2 j (4) –2 i – 2 j

Q.21 A stone of mass m1 moving with a uniform speed v


suddenly explodes on its own into two fragments. If
the fragment of mass m2 is at rest, the speed of the
other fragment is- (1) u2/2g (2) u2/g (3) u2/8g (4) u2/4g
m1v m2 v
(1) (m  m ) (2) (m  m ) Q.28 A ball strikes a smooth horizontal ground at an angle of
1 2 1 2 45° with the vertical. What cannot be the possible angle
of its velocity with the vertical after the collision.
m1v m1v (Assume e  1).
(3) (m  m ) (4) m
1 2 2 (1) 45° (2) 30° (3) 53° (4) 60°

Q.29 In an inelastic collision-


Q.22 A nucleus of mass number A originally at rest emits -
(1) momentum is conserved but kinetic energy is not
particle with speed v. The recoil speed of daughter (2) momentum is not conserved but kinetic energy is
nucleus is : conserved
4v 4v v v (3) neighter momentum nor kinetic energy is conserved
(1) (2) (3) (4) (4) both the momentum and kinetic energy are conserved
A4 A4 A4 A4
60
Motion of System of Particles and Rigid Body

Q.30 A bullet of mass m = 50 gm strikes (t  0)a sand bag Q.35 Two ballsA and B having masses 1 kg and 2 kg, moving
of mass M = 5 kg hanging from a fixed point, with a with speeds 21 m/s and 4 m/s respectively in opposite

horizontal velocity v p . If bullet sticks to the sand bag direction, collide head on. After collision A moves with
then the ratio of final & initial kinetic energy of the a speed of 1 m/s in the same direction, then the
bullet is : coefficient of restitution is
(1) 10–2 (2) 10–3 (3) 10–6 (4) 10–4 (1) 0.1 (2) 0.2 (3) 0.4 (4) None

Q.31 In the arrangement shown, the pendulum on the left is Q.36 A particle of mass m moves with velocity v0 = 20 m/sec
pulled aside. It is then released and allowed to collide towards a large wall that is moving with velocity v = 5 m/
with other pendulum which is at rest.Aperfectly inelastic sec. towards a particle as shown. If the particle collides
collision occurs and the system rises to a height h/4 . with the wall elastically, the speed of the particle just
The ratio of the masses (m1 / m2) of the pendulum is : after the collision is :

(1) 1 (2) 2 (3) 3 (4) 4


(1) 30 m/s (2) 20 m/s (3) 25 m/s (4) 22 m/s
Q.32 Two perfectly elastic balls of same mass m are moving
with velocities u1 and u2. They collide elastically n Q.37 A massive ball moving with speed v collides head-on
times. The kinetic energy of the system finally is : with a tiny ball at rest having a mass very less than the
1m 2 1m 2 mass of the first ball. If the collision is elastic, then
(1) u1 (2) (u1  u 22 ) immediately after the impact, the second ball will move
2u 2u
with a speed approximately equal to:
1 1 (4) .
(3) m(u12  u 22 ) (4) mn(u12  u 22 ) (1) v (2) 2v (3) v/2
2 2
Q.38 A sphere of mass m moving with a constant velocity
Q.33 A ball hits the floor and rebounds after an inelastic hits another stationary sphere of the same mass. If e is
collision . In this case-
the coefficient of restitution, then ratio of speed of the
(1) the momentum of the ball just after the collision is
first sphere to the speed of the second sphere after
the same as that just before the collision
collision will be :
(2) the mechanical energy of the ball remains the same
in the collision  1 e  1 e   e 1  e 1 
(1)   (2)  1  e  (3)  e  1  (4)  e  1 
(3) the total momentum of the ball and the earth is 1 e     
conserved.
(4) the total energy of the ball and the earth is Q.39 A truck moving on horizontal road east with velocity
conserved 20ms–1 collides elastically with a light ball moving with
velocity 25 ms–1 along west. The velocity of the ball
Q.34 Six steel balls of identical size are lined up along a
just after collision
straight frictionless groove. Two similar balls moving
with a speed V along the groove collide with this row (1) 65 ms–1 towards east (2) 25 ms–1 towards west
on the extreme left hand then- (3) 65 ms–1 towards west (4) 20 ms–1 towards east

Q.40 Three blocks are initially placed as shown in the figure.


Block A has mass m and initial velocity v to the right.
Block B with mass m and block C with mass 4 m are both
(1) all the balls will start moving to the right with initially at rest. Neglect friction. All collisions are elastic.
speed 1/8 each
The final velocity of block A is
(2) all the six balls initially at rest will move on with
speed V/6 each and two identical balls will come V
to rest A B C
(3) two balls from the extreme right end will move on m m 4m
with speed V each and the remaining balls will
remain at rest (1) 0.60 v to the left (2) 1.4 v to the left
(4) one ball from the right end will move on with speed (3) v to the left (4) 0.4 v to the left
2V, the remaining balls will be at rest.
61
Motion of System of Particles and Rigid Body

Q.41 Two billiard balls undergo a head-on collision. Ball 1 is ROTATIONAL MOTION
twice as heavy as ball 2. Initially, ball 1 moves with a Q.47 A fan is running at 3000 rpm. It is switched off. It comes
speed v towards ball 2 which is at rest. Immediately to rest by uniformly decreasing its angular speed in 10
after the collision, ball 1 travels at a speed of v/3 in the seconds. The total number of revolutions in this
same direction. What type of collision has occured ? period.
(1) inelastic (1) 150 (2) 250 (3) 350 (4) 300
(2) elastic
(3) completely inelastic Q.48 A block hangs from a string wrapped on a disc of radius
(4) Cannot be determined from the information given 20 cm free to rotate about its axis which is fixed in a
horizontal position. If the angular speed of the disc is
Q.42 A neutron travelling with a velocity v and K.E. E. 10 rad/s at some instant, with what speed is the block
collides perfectly elastically head on with the nucleus going down at that instant ?
of an atom of mass number A at rest. The fraction of (1) 4 m/s (2) 3 m/s (3) 2 m/s (4) 5 m/s
total energy retained by neutron is-
Q.49 The moment of inertia of a non-uniform semicircular
FG A  1IJ 2
F A  1IJ
(2) G
2
wire having mass m and radius r about a line
(1)
H A  1K H A  1K perpendicular to the plane of the wire through the
centre is

(3) G
F A  1IJ 2
F A  1IJ
(4) G
2 1 2 1 2 2 2
H AK H AK (1) mr2 (2)
2
mr (3)
4
mr (4)
5
mr

Q.50 Let A and B be the moments of inertia of two solid


Q.43 An object of mass 5 kg and speed 10 ms-1 explodes cylinders of identical geometrical shape and size about
into two pieces of equal mass. One piece comes to their axes, the first made of aluminium and the second
rest. The kinetic energy added to the system during of iron.
the explosion is- (1) A < B
(1) Zero. (2) 50 J. (3) 250 J. (4) 500 J. (2) A = B
(3) A > B
Q.44 Two particles of same mass m moving with velocities (4) relation between A and B depends on the actual
u1 and u2 collide perfectly inelastically. The loss of shapes of the bodies.
energy would be-
(1) 1/2m (u1 – u2)2 (2) 1/4m (u1 – u2 )2 Q.51 Let I1 and I2 be moments of inertia of a body about two
2
(3) m (u1 – u2 ) . (4) 2m (u1 – u1)2. axes 1 and 2 respectively, The axis 1 passes through
the centre of mass of the body but axis 2 does not.
Q.45 A ball of 0.1kg strikes a wall at right angle with a speed (1) I1 < I2
of 6 m/s and rebounds along its original path at 4 m/s. (2) If I1 < I2, the axes are parallel.
The change in momentum in Newton- sec is- (3) If the axes are parallel, I1 < I2
(1) 103 (2) 102 (3) 10 (4) 1 (4) If the axes are not parallel, I1  I2.

Q.46 A block of mass m starts from rest and slides down a Q.52 A unifrom thin rod of length L and mass M is bent at
frictionless semi-circular track from a height h as shown. the middle point O as shown in figure. Consider an
axis passing through its middle point O and
When it reaches the lowest point of the track, it collides
perpendicular to the plane of the bent rod. Then
with a stationary piece of putty also having mass m. If
moment of inertia about this axis is :
the block and the putty stick together and continue to
slide, the maximum height that the block-putty system
could reach is

2 1
h (1) mL2 (2) mL2
3 3
1
(1) h/4 (2) h/2 (3) mL2 (4) dependent on 
12
(3) h (4) independent of h

62
Motion of System of Particles and Rigid Body

Q.53 The moment of inertia of a uniform circular disc about Q.61 A wheel starting with angular velocity of 10 radian/
its diameter is 200 gm cm2. Then its moment of inertia sec acquires angular velocity of 100 radian/sec in 15
about an axis passing through its center and
seconds. If moment of inertia is 10kg-m2, then applied
perpendicular to its circular face is
(1) 100 gm cm2 (2) 200 gm cm2 torque (in newton-metre) is
(3) 400 gm cm 2
(4) 1000 gm cm2 (1) 900 (2) 100 (3) 90 (4) 60

Q.54 Two spheres of same mass and radius are in contact Q.62 A torque of 2 newton-m produces an angular
with each other. If the moment of inertia of a sphere acceleration of 2 rad/sec2 a body. If its radius of
about its diameter is I, then the moment of inertia of
gyration is 2m, its mass will be:
both the spheres about the tangent at their common
point would be - (1) 2kg (2) 4 kg (3) 1/2 kg (4) 1/4 kg
(1) 3I (2) 7I (3) 4I (4) 5I
Q.63 A particle is at a distance r from the axis of rotation. A
Q.55 The M.I. of a disc about its diameter is 2 units. Its given torque  produces some angular acceleration
M.I. about axis through a point on its rim and in the
in it. If the mass of the particle is doubled and its
plane of the disc is
(1) 4 units (2) 6 units distance from the axis is halved, the value of torque
(3) 8 units (4) 10 units to produce the same angular acceleration is
(1) /2 (2)  (3) 2 (4) 4
Q.56 A solid sphere and a hollow sphere of the same mass
have the same moments of inertia about their Q.64 A uniform circular disc A of radius r is made from a
respective diameters, the ratio of their radii is
metal plate of thickness t and another uniform circular
(1) (5)1/2 : (3)1/2 (2) (3)1/2 : (5)1/2
(3) 3 : 2 (4) 2 : 3 disc B of radius 4r is made from the same metal plate of
thickness t/4. If equal torques act on the discs A and
Q.57 Moment of inertia of a thin semicircular disc (mass = B, initially both being at rest. At a later instant, the
M & radius = R) about an axis through point O and angular speeds of a point on the rim of A and another
perpendicular to plane of disc, is given by :
point on the rim of B are A and B respectively. We
O have
R (1) A > B
(2) A = B
(3) A < B
(4) the relation depends on the actual magnitude of
1 2 1 2 1 2
(1) MR (2) MR (3) MR (4) MR 2
the torques.
4 2 8


A force F = 4 i  10 j acts on a body at a point
Q.58 A rigid body can be hinged about any point on the x-
Q.65
axis. When it is hinged such that the hinge is at x, the

having position vector  5 i  3 j relative to origin


moment of inertia is given by I = 2x2 – 12x + 27 The x-
coordinate of centre of mass is
(1) x = 2 (2) x = 0 (3) x = 1 (4) x = 3 of co-ordinates on the axis of rotation . The torque
acting on the body about the origin is :
Q.59 A disc of radius 2m and mass 200kg is acted upon by
a torque 100N-m. Its angular acceleration would be (1) 38 k (2)  25 k
(1) 1 rad/sec2 (2) 0.25 rad/sec2
2
(3) 0.5 rad/sec . (4) 2 rad/sec2. (3) 62 k (4) none of these

Q.60 On applying a constant torque on a body-


Q.66 In case of torque of a couple if the axis is changed by
(1) linear velocity may be increases
(2) angular velocity may be increases displacing it parallel to itself, torque will:
(3) it will rotate with constant angular velocity (1) increase (2) decrease
(4) it will move with constant velocity (3) remain constant (4) None of these

63
Motion of System of Particles and Rigid Body

Q.67 Four equal and parallel forces are acting on a rod (as Q.72 A weightless rod is acted on by upward parallel forces
shown in figure) at distances of 20 cm, 40 cm, 60 cm of 2N and 4N ends A and B respectively. The total
and 80 cm respectively from one end of the rod. Under length of the rod AB = 3m. To keep the rod in
the influence of these forces the rod : equilibrium a force of 6N should act in the following
manner :
(1) Downwards at any point between A and B.
(2) Downwards at mid point of AB.
(3) Downwards at a point C such that AC = 1m.
(4) Downwards at a point D such that BD = 1m.

Q.73 A Cubical bloc of mass M and edge a slides down a


rough inclined plane of inclination  with a uniform
(1) is at rest
velocity. The torque of the normal force on the block
(2) experiences a torque
about its centre has a magnitude.
(3) experiences a linear motion
(1) zero (2) Mga
(4) experiences a torque and also a linear motion
1
(3) Mga sin  (4) Mga sin 
Q.68 A rod of weight w is supported by two parallel knife 2
edges A & B and is in equilibrium in a horizontal
position. The knives are at a distance d from each other. Q.74 A uniform cube of side a and mass m rests on a rough
The centre of mass of the rod is at a distance x from A. horizontal table. A horizontal force 'F' is applied normal
The normal reactions at A and B will be : to one of the faces at a point that is directly above the
(1) NA = 2w (1  x/d), NB = wx/d ]
3a
(2) NA = w (1  x/d), NB = wx/d ] centre of the face, at a height above the base. The
4
(3) NA = 2w (1  x/d), NB = 2wx/d ]
(4) NA = w (2  x/d), NB = wx/d ] minimum value of 'F' for which the cube begins to tilt
about the edge is (assume that the cube does not slide).
Q.69 A body is rotating with constant angular velocity 2 4 5 1
(1) mg (2) mg (3) mg (4) mg
about a vertical axis fixed in an inertial frame. The net 3 3 4 2
force on a particle of the body not on the axis is
(1) horizontal and skew with the axis Q.75 The moment of inertia and rotational kinetic energy
(2) vertical of a fly wheel are 20kg-m2 and 1000 joule respectively.
(3) horizontal and intersecting the axis Its angular frequency per minute would be -
(4) none of these.
600 25 5 300
(1) (2) (3) (4)
Q.70 The uniform rod of mass 20 kg and length 1.6 m is  2  
pivoted at its end and swings freely in the vertical

The angular velocity of a body is  = 2 i + 3 j + 4 k


plane. Angular acceleration of rod just after the rod is 
released from rest in the horizontal position as shown Q.76
in figure is
and a torque  = i + 2 j + 3 k acts on it. The

rotational power will be


(1) 20 watt (2) 15 watt
(3) 17 watt (4) 14 watt
15g 17g 16g g
(1) (2) (3) (4)
16 16 15 15
Q.77 A circular ring of wire of mass M and radius R is
making n revolutions/sec about an axis passing
Q.71 Two men support a uniform horizontal rod at its two through a point on its rim and perpendicular to its
ends. If one of them suddenly lets go, the force exerted plane. The kinetic energy of rotation of the ring is
by the rod on the other man will: given by-
(1) remain unaffected (1) 42MR2n2 (2) 22MR2n2
(2) increase
(3) decrease 1 2
(3)  MR2n2 (4) 82MR2n2
(4) become unequal to the force exerted by him on the 2
beam.
64
Motion of System of Particles and Rigid Body

Q.78 Rotational kinetic energy of a disc of constant moment Q.87 The centre of a disc rolling without slipping on a plane
of inertia is - surface moves with speed u. A particle on the rim of
(1) directly proportional to angular velocity the wheel at the same level as the centre will be moving
(2) inversely proportional to angular velocity at speed
(3) inversely proportional to square of angular (1) zero (2) u (3) u (4) 2u
velocity
(4) directly proportional to square of angular velocity Q.88 A thin string is wrapped several times around a cylinder
kept on a rough horizontal surface. A boy standing at
Q.79 A circular disc has a mass of 1kg and radius 40 cm. It a distance  from the cylinder draws the string towards
is rotating about an axis passing through its centre him as shown in figure. The cylinder rolls without
and perpendicular to its plane with a speed of slipping. The length of the string passed through the
10rev/s. The work done in joules in stopping it would hand of the boy while the cylinder reaches his hand is
be-
(1) 4 (2) 47.5 (3) 79 (4) 158

Q.80 A constant torque acting on a uniform circular wheel


changes its angular momentum from A0 to 4A0 in 4 sec.
the magnitude of this torque is : (1)  (2) 2 (3) 3 (4) 4
(1) 4A0 (2) A0 (3) 3A0 /4 (4) 12A0
Q.89 A solid sphere, a hollow sphere and a ring, all having
Q.81 A particle moves with a constant velocity parallel to equal mass and radius, are placed at the top of an
the Y-axis. Its angular momentum about the origin. incline and released. The friction coefficients between
(1) is zero (2) remains constant the objects and the incline are equal and but not
(3) goes on increasing (4) goes on decreasing. sufficient to allow pure rolling. The greastest kinetic
energy at the bottom of the incline will be achieved by
Q.82 A particle performs uniform circular motion with an (1) the solid sphere
angular momentum L. If the frequency of particle's (2) the hollow sphere
motion is doubled and its kinetic energy is halved, the (3) the ring
angular momentum becomes: (4) all will achieve same kinetic energy.
(1) 2L (2) 4L (3) L/2 (4) L/4
Q.90 A hollow sphere and a solid sphere having equal mass
and equal radii are rolled down without slipping on a
Q.83 A boy sitting firmly over a rotating stool has his arms
rough inclined plane.
folded. If he stretches his arms, his angular momentum
(1) The two spheres reach the bottom simultaneously
about the axis of rotation
(2) The hollow sphere reaches the bottom with lesser
(1) increases (2) decreases
speed.
(3) remains unchanged (4) doubles
(3) The hollow sphere reaches the bottom with greater
kinetic energy
Q.84 The rotational kinetic energy of a rigid body of
(4) The two spheres will reach the bottom with same
moment of inertia 5 kg-m2 is 10 joules. The angular
linear momentum
momentum about the axis of rotation would be -
(1) 100 joule-sec (2) 50 joule-sec
Q.91 A body is given translational velocity and kept on a
(3) 10 joule-sec (4) 2 joule -sec surface that has sufficient friction. Then:
(1) body will move forward before pure rolling
Q.85 The angular velocity of a body changes from one (2) body will move backward before pure rolling
revolution per 9 second to 1 revolution per second (3) body will start pure rolling immediately
without applying any torque. The ratio of its radius (4) none of these
of gyration in the two cases is
(1) 1 : 9 (2) 3 : 1 (3) 9 : 1 (4) 1 : 3 Q.92 A disk and a ring of the same mass are rolling to
have the same kinetic energy. What is ratio of their
Q.86 A disc rolls on a table. The ratio of its K.E. of rotation velocities of centre of mass
to the total K.E. is - (1) (4:3)1/2 (2) (3 : 4)1/2
(1) 2/5 (2) 1/3 (3) 5/6 (4) 2/3 1/2
(3) (2) : (3) 1/2
(4) (3)1/2 : (2)1/2
65
Motion of System of Particles and Rigid Body

Q.93 A body kept on a smooth horizontal surface is pulled


by a constant horizontal force applied at the top point EXERCISE-III
of the body. If the body rolls purely on the surface, its
shape can be :
KVPY
(1) thin pipe (2) uniform cylinder PREVIOUS YEAR’S
(3) uniform sphere (4) thin spherical shell
CENTRE OF MASS
Q.1 A soldier with a machine gun, falling from an airplane
Q.94 A solid sphere with a velocity (of centre of mass) v
gets detached from his parachute. He is able to resist
and angular velocity  is gently placed on a rough
horizontal surface. The frictional force on the sphere : the downward acceleration if the shoots 40 bullets a
(1) must be forward (in direction of v) second at the speed of 500 m/s. If the weight of a bullet
(2) must be backward (opposite to v) is 49 gm, what is the weight of the man with the gun ?
(3) cannot be zero Ignore resistance due to air and assume the acceleration
(4) none of the above due to gravity g = 9.8 ms–2. [KVPY (SA)-2010]
(A) 50 kg (B) 75 kg
Q.95 A cylinder is pure rolling up an incline plane. It stops (C) 100 kg (D) 125 kg
momentarily and then rolls back. The force of friction.
(1) on the cylinder is zero throughout the journey Q.2 Two masses m1 and m2 are connected by a massless
(2) is directed opposite to the velocity of the centre of spring of spring constant k and unstreched length  .
mass throughout the journey The masses are placed on a frictionless straight channel
(3) is directed up the plane throughout the journey which we consider our x-axis. They are initially at rest
(4) is directed down the plane throughout the journey
at x = 0 and x =  , respectively. At t = 0, a velocity of v0
is suddenly imparted to the first particle. At a later time
Q.96 A sphere is released on a smooth inclined plane from t0, the centre of mass of the two masses is at-
the top. When it moves down its angular momentum [KVPY (SB/SX)-2010]
is:
m 2
(1) conserved about every point (A) x 
(2) conserved about the point of contact only m1  m2
(3) conserved about the centre of the sphere only
(4) conserved about any point on a fixed line parallel m1 m 2 v0 t
(B) x  
to the inclined plane and passing through the m1  m 2 m1  m 2
centre of the ball.
m2 m 2 v0 t
(C) x  
Q.97 A circular wooden loop of mass m and radius R rests m1  m 2 m1  m 2
flat on a horizontal frictionless surface. A bullet, also
of mass m, and moving with a velocity V, strikes the m 2 m1v0 t
(D) x  
loop and gets embedded in it. The thickness of the m1  m 2 m1  m 2
loop is much smaller than R. The angular velocity with
which the system rotates just after the bullet strikes
the loop is Q.3 On a horizontal frictionless frozen lake, a girl (36 kg)
and a box (9 kg) are connected to each other by means
of a rope. Initially they are 20 m apart. The girl exerts a
horizontal force on the box, pulling it towards her. How
far has the girl travelled when she meets the box
[KVPY (SA)-2011]
(A) 10 m
V V 2V 3V
(1) (2) 3R (3) 3R (4) 4R (B) Since there is no friction, the girl will not move
4R
(C) 16 m
(D) 4 m
66
Motion of System of Particles and Rigid Body

Q.4 A narrow but tall cabin is falling freely near the earth’s Q.8 A small boy is throwing a ball towards a wall 6 cm in
surface. Inside the cabin, two small stones A and B are
front of him. He releases the ball at a height of 1.4 m
released from rest (relative to the cabin). Initially A is
much above the centre of mass B much below the centre from the ground. The ball bounces from the wall at a
of mass of the cabin. A close observation of the motion height of 3 m, rebounds from the ground and reaches
of A and B will reveal that - [KVPY (SB/SX)-2011] the boy’s hand exactly at the point of release. Assuming
(A) Both A and B continue to be exactly at rest relative
the two bounces (one from the wall and the other from
to the cabin
(B) A moves slowly upward and B moves slowly the ground) to be perfectly elastic, how far ahead of
downward relative the cabin the boy did the ball bounce from the ground?
(C) Both A and B fall to the bottom of the cabin with [KVPY (SB/SX)-2013]
constant acceleration due to gravity
(A) 1.5 m (B) 2.5 m
(D) A and B move slightly towards each other vertically
(C) 3.5 m (D) 4.5 m
Q.5 A clay ball of mass m and speed v strikes another metal
ball. of same mass m, which is at rest. They stick together Q.9 Two uniform plates of the same thickness and area but
after collision. The kinetic energy of the system after
collision is - [KVPY (SA)-2012] of different materials, one shaped like an isosceles
(A) mv2/2 (B) mv2/4 triangle and the other shaped like a rectangle are joined
(C) 2 mv2 (D) mv2 together to form a composite body as shown in the
figure. If the centre of mass of the composite body is
Q.6 A ball of mass m suspended from a rigid support by an
inextensible massless string is released from a height h located at the midpoint of their common side, the ratio
above its lowest point. At its lowest point it collides between masses of the triangle to that of the rectangle
elastically with a block of mass 2m at rest on a is :- [KVPY (SB/SX)-2015]
frictionless surface. Neglect the dimensions of the ball
and the block. After the collision the ball rises to a
maximum height of – [KVPY (SB/SX)-2012]

(A) 1 : 1 (B) 4 : 3
m (C) 3 : 4 (D) 2 : 1
h
2m
Q.10 A smaller cube with side b (depicted by dashed lines)
(A) h/3 (B) h/2 is excised from a bigger uniform cube with side a as
(C) h/8 (D) h/9 shown below such that both cubes have a common
vertex P. Let X = a/b. If the centre of mass of the
Q.7 A man inside a freely falling box throws a heavy ball
remaining solid is at the vertex O of smaller cube then X
towards a side wall. The ball keeps on bouncing
between the opposite walls of the box. We neglect air satisfies. [KVPY (SB/SX)-2016]
resistance and fricton. Which of the following figures
depicts the motion of the centre of mass of the entire
system (man, the ball and the box)?
[KVPY (SA)-2013]

(A) (B)

(A) X3 – X2 – X – 1 = 0
(B) X2 – X – 1 = 0
(C) (D) (C) X3 + X2 – X – 1 = 0
(D) X3 – X2 – X +1 = 0

67
Motion of System of Particles and Rigid Body

ROTATIONAL MOTION Q.14 A bullet of mass m is fired horizontally into a large


Q.11 Seven identical coins are rigidly arranged on a flat table sphere of mass M and radius R resting on a smooth
in the pattern shown below so that each coin touches horizontal table. [KVPY (SB/SX)-2013]
its neighbors. Each coin is a thin disc of mass m and M
radius r. Note that the moment of inertia of an individual
coin about an axis passing through centre and
perpendicular to the plane of the coin is mr2/2. h 2R

The bullet hits the sphere at a height h from the table


P and sticks to its surface. If the sphere starts rolling
without slipping immediately on impact, then
The moment of inertia of the system of seven coins h 4m  3M h m  3M
(A) R  2 m  M 
about on axis that passes through the point P (the centre   (B)
R m  2M
of the coin positioned directly to the right of the central
coin) and perpendicular to the plane of the coins is h 10m  7M h 4m  3M
(C)  (D) 
[KVPY (SB/SX)-2012] R 5(m  M) R mM
55 2 127 2
(A) mr (B) mr
2 2 Q.15 A solid sphere rolls without slipping, first horizontal
111 2 and then up to a point X at height h on an inclined
(C) mr (D) 55 mr2 plane before rolling down, as shown.
2
[KVPY (SB/SX)-2013]
Q.12 Two identical uniform rectangular blocks (with longest x
side L) and a solid sphere of radius R are to be balanced
at the edge of a heavy table such that the centre of the h
sphere remains at the maximum possible horizontal
distance from the vertical edge of the table without
toppling as indicated in the figure. [KVPY (SA)-2013] The initial horizontal speed of the sphere is
(A) 10gh / 7 (B) 7gh / 5
(C) 5gh / 7 (D) 2gh

Table
R
Q.16 A uniform thin rod of length 2L and mass m lies on a
L horizontal table. A horizontal impulse J is given to the
x rod at one red. There is no friction. The total kinetic
If the mass of each block is M and of the sphere is energy of the rod just after the impulse will be
M/2, then the maximum distance x that can be achieved [KVPY (SB/SX)-2014]
is
(A) 8L/15 (B) 5L/6 J2 J2
(A) (B)
(C) (3L/4+R) (D) (7L/15+R) 2m m
Q.13 A wheel of radius R with an axle of radius R/2 is shown 2J 2 6J 2
in the figure and is free to rotate about a frictionless (C) (D)
m m
axis through its centre and perpendicular to the page.
Three forces (F, F, 2F) are exerted tangentially to the
respective rims as shown in the figure. Q.17 A solid cylinder P rolls without slipping from rest down
[KVPY (SB/SX)-2013] an inclined plane attaining a speed vP at the bottom.
Another smooth solid cylinder Q of same mass and
45° dimensions slides without friction from rest down the
F
F inclined plane attaining a speed vQ at the bottom. The
R/2
V 
ratio of the speeds  Q  is –
R  VP 
2F
[KVPY (SB/SX)-2014]
The magnitude of the net torque acting on the system
(A) 3/ 4 (B) 3 / 2
is nearly
(A) 3.5 FR (B) 3.2 FR (C) 2/3 (D) 4/3
(C) 2.5 FR (D) 1.5 FR
68
Motion of System of Particles and Rigid Body

Q.18 A solid sphere spinning about a horizontal axis with Q.22 The moments of inertia of a non-uniform circular disc
an angular velocity  is placed on a horizontal surface. (of mass M and radius R) about four mutually
Subsequently it rolls without slipping with an angular perpendicular tangents AB, BC, CD, DAare I1, I2, I3 and
velocity of - [KVPY (SB/SX)-2014]
I4, respectively (the square ABCD circumscribes the
(A) 2/5 (B) 7/5
(C) 2/7 (D)  circle). The distance of the center of mass of the disc
from its geometrical center is given by -
Q.19 One mole of ideal gas undergoes a linear process as [KVPY (SB/SX)-2015]
shown in figure below. Its temperature expressed as
1
function of volume V is - [KVPY (SA)-2015] (A) (I1  I3 )2  (I2  I4 ) 2
4MR
P0
1
(B) (I1  I3 )2  (I 2  I 4 )2
12MR
(0,0) V0
1
P0 V0 P0 V (C) (I1  I 2 ) 2  (I3  I4 ) 2
(A) (B) 3MR
R R
1
P0 V0   V
2 (I1  I3 )2  (I2  I 4 ) 2
P0 V  V   (D)
(C) 1   (D) 1    2MR
R  V0  R   V0  
 
Q.23 A uniform metal plate shaped like a triangle ABC has a
Q.20 Two spherical objects each of radii R and masses m1 mass of 540 gm. the length of the sides AB, BC and CA
and m2 are suspended using two strings of equal length are 3cm, 5 cm and 4cm, respectively. The plate is pivoted
L as shown in the figure (R < < L). The angle,  which freely about A. What mass must be added to a vertex,
mass m2 makes with the vertical is approximately- so that the plate can hang with the long edge horizontal
[KVPY (SB/SX)-2015]
[KVPY (SA)-2016]
(A) 140 gm at C (B) 540 gm at C
(C) 140 gm at B (D) 540 gm at B

Q.24 A "V" shaped rigid body has two identical uniform


m1 m2
arms. What must be the angle between the two arms so
m1R 2m1R that when the body is hung from one end, the other
(A) (m  m )L (B) (m  m )L arm is horizontal ? [KVPY (SA)-2016]
1 2 1 2
(A) cos–1 (1/3) (B) cos–1(1/2)
2m 2 R m2R
(C) (m  m )L (D) (m  m )L (C) cos–1 (1/4) (D) cos–1 (1/6)
1 2 1 2

Q.25 A uniform metal plate shaped like a triangle ABC has a


Q.21 A horizontal disk of moment of inertia 4.25 kg-m2 with mass of 540 gm. the length of the sides AB, BC and CA
respect to its axis of symmetry is spinning counter are 3cm, 5 cm and 4cm, respectively. The plate is pivoted
clockwise at 15 revolutions per second about its axis,
freely about A. What mass must be added to a vertex,
as viewed from above. A second disk of moment of
so that the palte can hang with the long edge horizontal
inertia 1.80 kg-m2 with respect to its axis of symmetry is
[KVPY (SA)-2016]
spinning clockwise at 25 revolutions per second as
(A) 140 gm at C (B) 540 gm at C
viewed from above about the same axis and is dropped
on top of the first disk. The two disks sticks together (C) 140 gm at B (D) 540 gm at B
and rotate as one about their axis of symmetry. The
new angular velocity of the system as viewed from Q.26 Uniform ring of radius R is moving on a horizontal
above is close to - [KVPY (SB/SX)-2015] surface with speed v and then climbs up a ramp of
(A) 18 revolutions/ second and clockwise inclination 30º to a height h. There is no slipping in the
(B) 18 revolutions/second and counter clockwise entire motion. Then h is- [KVPY (SB/SX)-2016]
(C) 3 revolutions/second and clockwise (A) v2/2g (B) v2/g
(D) 3 revolutions/second and counter clockwise (C) 3v2/2g (D) 2v2/g
69
Motion of System of Particles and Rigid Body

Q.27 A solid cube of wood of side 2a and mass M is resting NEET / AIPMT MEDICAL
on a horizontal surface as shown in the figure. The PREVIOUS YEAR'S
cube is free to rotate about a fixed axis AB. A bullet of CENTRE OF MASS
mass m (<<M) and speed v is shot horizontally at the Q.1 Two particles which are initially at rest, move towards
face opposite to ABCD at a height 4a/3 from the surface each other under the action of their internal attraction.
If their speeds are v and 2v at any instant, then the
to impart the cube an angular speed . It strike the face
speed of centre of mass of the system will be :
and embeds in the cube. Then c is close to (note : the [AIPMT-2010]
moment of inertia of the cube about an axis (1) 2v (2) 0
perpendicular to the face and passing through the (3) 1.5 v (4) v
center of mass is 2Ma2/3) [KVPY (SB/SX)-2017] Q.2 A ball moving with velocity 2 m/s collides head on with
another stationary ball of double the mass. If the
C coefficient of restitution is 0.5, then their velocities (in
D m/s) after collision will be : [AIPMT-2010]
(1) 0, 1 (2) 1, 1
(3) 1, 0.5 (4) 0, 2
B Q.3 A mass m moving horizontally (along the x-axis) with
A velocity v collides and sticks to a mass of 3m moving
vertically upward (along the y-axis) with velocity 2v.
(A) Mv/ma (B) Mv/2ma The final velocity of the combination is :
[AIPMT Mains-2011]
(C) mv/Ma (D) mv/2Ma
3 ˆ 1 ˆ 1 ˆ 3 ˆ
(1) vi  vj (2) vi  vj
2 4 4 2
Q.28 A table has a heavy circular top of radius 1 m and mass
1 ˆ 3 ˆ 2 ˆ 1 ˆ
20 kg, placed on four light (considered massless) legs (3) vi  vj (4) vi  vj
placed symmetrically on its circumference. The 3 2 3 3
maximum mass that can be kept anywhere on the table Q.4 Three masses are placed on the x-axis : 300 g at origin,
without toppling it is close to [KVPY (SB/SX)-2018] 500 g at x = 40 cm and 400 g at x = 70 cm. The distance
(A) 20 kg (B) 34 kg of the centre of mass from the origin is :
[AIPMT Mains-2012]
(C) 47 kg (D) 59 kg
(1) 40 cm (2) 45 cm
(3) 50 cm (4) 30 cm

Q.29 A plank is moving in a horizontal direction with a Q.5 Two persons of masses 55 kg and 65 kg respectively,
are at the opposite ends of a boat. The length of the
constant acceleration aiˆ . Auniform rough cubical block boat is 3.0 m and weighs 100 kg. The 55 kg man walks
up to the 65 kg man and sits with him. If the boat is in
of side  rests on the plank, and is at rest relative to the still water the centre of mass of the system shifts by :
plank. [KVPY (SB/SX)-2018] [AIPMT-2012]
(1) 3.0 m (2) 2.3 m
y (3) Zero (4) 0.75 m
Block Q.6 Two spheres A and B of masses m1 and m2 respectively
x a collide. Ais at rest initially and B is moving with velocity
Plank
v
v along x-axis. After collision B has a velocity in a
2
Let the center of mass of the block be at  0,  / 2 at a direction perpendicular to the original direction. the
mass A moves after collision in the direction :
given instant. If a = g/10, then the normal reaction [AIPMT-2012]
exerted by the plank on the block at that instant acts at (1) Same as that of B
(2) Opposite to that of B
(A) (0, 0) (B)   / 20, 0
1  1 
(3)   tan   to the x-axis
(C)   /10, 0 (D)   /10, 0 2
1  1 
(4)   tan    to the x-axis.
 2
70
Motion of System of Particles and Rigid Body

Q.7 A body of mass (4m) is lying in x-y plane at rest. It Q.13 A rigid ball of mass m strikes a rigid wall at 60° and
suddenly explodes into three pieces. Two pieces, each gets reflected without loss of speed as shown in the
of mass (m) move perpendicular to each other with equal figure. The value of impulse imparted by the wall on
speeds (v). The total kinetic energy generated due to the ball will be : [NEET Phase II-2016]
explosion is : [AIPMT-2014]
m
3 V
(1) mv2 (2) mv2
2 60°
(3) 2mv2 (4) 4mv2 60°
v
Q.8 Two particles A and B, move with constant velocities
 
v1 and v2 . At the initial moment their position vectors
  (1)mV (2)2mV
are r1 and r2 respectively. The condition for particles
A and B for their collision is : [AIPMT-2015] mV mV
(3) (4)
        2 3
(1) r1  v1  r2  v 2 (2) r1  r2  v1  v 2
   
r1  r2 v 2  v1    Q.14 Which of the following statements are correct ?
(3)      (4) r.v1  r2 .v 2
| r1  r2 | | v 2  v1 | [NEET-2017]
(i) centre of mass of a body always coincides with the
centre of gravity of the body.
Q.9 A ball is thrown vertically downwards from a height of
(ii) centre of mass of a body is the point at which the
20 m with an initial velocity v0. It collides with the
total gravitational torque on the body is zero.
ground, loses 50 percent of its energy in collision and
(iii) A couple on a body produces both translational
rebounds to the same height. The initial velocity v0 is
and rotational motion in a body.
(Take g = 10 ms–2) : [AIPMT-2015]
(iv) Mechanical advantage greater than one means that
(1) 28 ms–1 (2) 10 ms–1
small effort can be used to lift a large load.
(3) 14 ms–1 (4) 20 ms–1
(1) (i) and (ii) (2) (ii) and (iii)
(3) (iii) and (iv) (4) (ii) and (iv)
Q.10 On a frictionless surface, a block of mass M moving at
speed v collides elastically with another block of same
mass M which is initially at rest. After collision the first ROTATIONAL MOTION
block moves at an angle  to its initial direction and has Q.15 (1) Centre of gravity (C.G.) of a body is the point at
a speed v/3. The second block’s speed after the which the weight of the body acts.
collision is : [AIPMT-2015] (2) Centre of mass coincides with the centre of gravity
if the earth is assumed to have infinitely large radius
3 3 (3) To evaluate the gravitational field intensity due to
(1) v (2) v
2 2 any body at an external point, the entire mass of
the body can be considered to be concentrated at
2 2 3 its C.G.
(3) v (4) v
3 4 (4) The radius of gyration of any body rotating about
an axis is the length of the perpendicular dropped
Q.11 A bullet of mass 10 g moving horizontally with a from the C.G. of the body to the axis.
velocity of 400 ms–1 strikes a wood block of mass 2 kg Which one of the following pairs of statements is
which is suspended by light inextensible string of correct ? [AIPMT-2010]
length 5 m. As a result, the centre of gravity of the (1) (4) and (1) (2) (1) and (2)
block found to rise a vertical distance of 10 cm. The (3) (2) and (3) (4) (3) and (4)
speed of the bullet after it emerges out horizontally
form the block will be : [NEET Phase II-2016] Q.16 From a circular disc of radius R and mass 9M, a small
(1) 100 ms–1 (2) 80 ms–1 disc of mass M and radius R/3 is removed
(3) 120 ms–1 (4) 160 ms–1 concentrically. The moment of inertia of the remaining
disc about an axis perpendicular to the plane of the
Q.12 Two identical balls A and B having velocities of 0.5 disc and passing through its centre is :[AIPMT-2010]
ms–1 and – 0.3 ms–1 respectively collide elastically in
40
one dimension. The velocities of B and A after the (1) MR2 (2) MR2
collision respectively will be : [NEET Phase II-2016] 9
(1) – 0.5 ms–1 and 0.3 ms–1 (2) 0.5 ms–1 and – 0.3 ms–1 4
(3) – 0.3 ms–1 and 0.5 ms–1 (4) 0.3 ms–1 and 0.5 ms–1 (3) 4MR2 (4) MR2
9
71
Motion of System of Particles and Rigid Body

Q.17 A circular disk of moment of inertia Ii is rotating in a Q.21 The moment of inertia of a thin uniform rod of mass M
horizontal plane, about its symmetry axis, with a and length L about an axis passing through its midpoint
constant angular speed i. Another disk of moment of and perpendicular to its length is I0. Its moment of inertia
inertia Ib is dropped coaxially onto the rotating disk. about an axis passing through one of its ends and
Initially the second disk has zero angular speed. perpendicular to its length is : [AIPMT-2011]
Eventually both the disks rotate with a constant angular (1) I0 + ML2/2 (2) I0 + ML2/4
(3) I0 + 2ML2 (4) I0 + ML2
speed f . The energy lost by the initially rotating disc
to friction is : [AIPMT-2010]
Q.22 The instantaneous angular position of a point on a
1 I 2b 1 Ii2 rotating wheel is given by the equation (t) = 2t3 – 6t2.
(1) i2 (2) i2 The torque on the wheel becomes zero at :
2 (Ii  Ib ) 2 (Ii  Ib )
[AIPMT-2011]
(1) t = 1s (2) t = 0.5s
I b  Ii 2 1 I b Ii
(3) (I  I ) i (4) 2 (I  I ) i
2
(3) t = 0.25s (4) t = 2s
i b i b

Q.23 ABC is an equilateral triangle with O as its centre. F1 ,
Q.18 A thin circular ring of mass M and radius R is rotating  
about its axis with constant angular velocity . Two F2 and F3 represent three forces acting along the sides
objects each of mass m are attached gently to the AB, BC and AC respectively. If the total torque about
opposite ends of a diameter of the ring. The ring now 
O is zero then the magnitude of F3 is :[AIPMT-2012]
rotates with angular velocity given by :[AIPMT-2010]
(M  2m)  2M A
(1) (2)
2m M  2m

(M  2m) M F3
(3) (4) O
M M  2m
B F2
C
Q.19 A solid cylinder and a hollow cylinder, both of the same F1
mass and same external diameter are released from the
same height at the same time on an inclined plane. Both (1) F1 + F2 (2) F1 – F2
roll down without slipping. Which one will reach the F1  F2
bottom first ? [AIPMT-2010] (3) (4) 2(F1 + F2)
2
(1) Both together only when angle of inclination of
plane is 45º Q.24 The moment of inertia of a uniform circular disc is
(2) Both together maximum about an axis perpendicular to the disc and
(3) Hollow cylinder passing through : [AIPMT Mains-2012]
(4) Solid cylinder

Q.20 A small mass attached to a string rotates on a frictionless


C
table top as shown. If the tension in the string is D
increased by pulling the string causing the radius of B A
the circular motion to decrease by a factor of 2, the
kinetic energy of the mass will: [AIPMT-2011]

(1) B (2) C
r (3) D (4) A

Q.25 When a mass is rotating in a plane about a fixed point,


its angular momentum is directed along :[AIPMT-2012]
(1) A line perpendicular to the plane of rotation
(1) Decrease by a factor of 2 (2) The line making an angle of 45º to the plane of
(2) Remain constant rotation
(3) Increase by a factor of 2 (3) The radius
(4) Increase by a factor of 4 (4) The tangent to the orbit
72
Motion of System of Particles and Rigid Body

Q.26 A circular platform is mounted on a frictionless vertical Q.31 The ratio of the accelerations for a solid sphere (mass
axle. Its radius R = 2 m and its moment of inertia about m and radius R) rolling down an incline of angle 
the axle is 200 kg m2. It is initially at rest. A 50 kg man without slipping and slipping down the incline without
stands on the edge of the platform and begins to walk rolling is : [AIPMT-2014]
along the edge at the speed of 1 ms–1 relative to the (1) 5 : 7 (2) 2 : 3
ground. Time taken by the man to complete one (3) 2 : 5 (4) 7 : 5
revolution is : [AIPMT Mains-2012]
3

(1)  s (2) s Q.32 A force F  ˆi  3jˆ  6kˆ is acting at a point
2

r  2iˆ  6ˆj  12kˆ . The value of  for which angular

(3) 2 s (4) s momentum about origin is conserved is :[AIPMT-2015]
2
(1) 0 (2) 1
(3) –1 (4) 2
Q.27 A solid cylinder of mass 3 kg is rolling on a horizontal
surface with velocity 4 ms–1. It collides with a horizontal
Q.33 Point masses m1 and m2 are placed at the opposite ends
spring of force constant 200 N m–1. The maximum
of a rigid rod of length L, and negligible mass. The rod
compression produced in the spring will be :
[AIPMT-2012] is to be set rotating about an axis perpendicular to it.
(1) 0.5 m (2) 0.6 m The position of point P on this rod through which the
(3) 0.7 m (4) 0.2 m axis should pass so that the work required to set the
rod rotating with angular velocity 0 is minimum, is
Q.28 A rod PW of mass M and length L is hinged at end P. given by : [AIPMT-2015]
The rod is kept horizontal by a massless string tied to
point Q as shown in figure. When string is cut, the
initial angular acceleration of the rod is :[NEET-2013]
m1 P m2

x (L–x)
P
Q
L
m2 m2 L
2g 2g (1) x  m L (2) x  m  m
(1) (2) 1 1 2
L 2L
m1L m1
(3)
3g
(4)
g (3) x  m  m (4) x  m L
2L L 1 2 2

Q.29 A small object of uniform density rolls up a curved Q.34 An automobile moves on a road with a speed of 54 km
surface with an initial velocity v. It reaches upto a h–1. The radius of its wheels is 0.45 m and the moment
of inertia of the wheel about its axis of rotation is 3 kg
3v 2 m2. If the vehicle is brought to rest in 15 s, the magnitude
maximum height of with respect to the initial
4g of average torque transmitted by its brakes to the wheel
position. The object is : [NEET-2013] is : [AIPMT-2015]
(1) Hollow sphere (2) Disc (1) 10.86 kg m2 s–2 (2) 2.86 kg m2 s–2
(3) Ring (4) Solid sphere (3) 6.66 kg m2 s–2 (4) 8.58 kg m2 s–2

Q.30 A solid cylinder of mass 50 kg and radius 0.5 m is free to Q.35 A uniform circular disc of radius 50 cm at rest is free to
rotate about the horizontal axis. A massless string is turn about an axis which is perpendicular to its plane
wound round the cylinder with one end attached to it and passes through its centre. It is subjected to a torque
and other hanging freely. Tension in the string required which produces a constant angular acceleration of 2.0
to produce an angular acceleration of 2 revolutions s–2 rad s–2. Its net acceleration in ms–2 at the end of 2.0 s is
is : [AIPMT-2014] approximately : [NEET Phase I-2016]
(1) 25 N (2) 50 N (1) 6.0 (2) 3.0
(3) 78.5 N (4) 157 N (3) 8.0 (4) 7.0
73
Motion of System of Particles and Rigid Body

Q.36 A disk and a sphere of same radius but different masses Q.42 Two discs of same moment of inertia rotating about
roll off on two inclined planes of the same altitude and their regular axis passing through centre and
length. Which one of the two objects gets to the bottom perpendicular to the plane of disc with angular
of the plane first ? [NEET Phase I -2016] velocities 1 and 2 . They are brought into contact
(1) Both reach at the same time face to face coinciding the axis of rotation. The
(2) Depends on their masses expression for loss of energy during this process is :
(3) Disk [NEET-2017]
(4) Sphere
1
(1) I(1  2 )2 (2) I(1  2 )2
Q.37 Two rotating bodies A and B of masses m and 2m with 4
moments of inertia IA and IB (IB > IA) have equal kinetic
1 1
energy of rotation. If LA and LB be their angular momenta (3) I(1  2 ) 2 (4) I(1  2 ) 2
8 2
respectively, then : [NEET Phase II-2016]
LB
(1) L A  (2) LA = 2LB Q.43 Three objects, A : (a solid sphere), B : (a thin circular
2 disk) and C : (a circular ring), each have the same mass
(3) LB > LA (4) LA > LB M and radius R. They all spin with the same angular
speed  about their own symmetry axes. The amounts
Q.38 A solid sphere of mass m and radius R is rotating about of work (W) required to bring them to rest, would satisfy
its diameter. Asolid cylinder of the same mass and same the relation [NEET-2018]
radius is also rotating about its geometrical axis with (1) WB > WA > WC (2) WA > WB > WC
an angular speed twice that of the sphere. The ratio of (3) WC > WB > WA (4) WA > WC > WB
their kinetic energies of rotation (Esphere / Ecylinder) will be:
[NEET Phase II-2016] Q.44 A moving block having mass m, collides with another
(1) 2 : 3 (2) 1 : 5 stationary block having mass 4m. The lighter block
(3) 1 : 4 (4) 3 : 1 comes to rest after collision. When the initial velocity
of the lighter block is v, then the value of coefficient of
Q.39 A light rod of length  has two masses m1 and m2
restitution (e) will be [NEET-2018]
attached to its two ends. The moment of inertia of the
(1) 0.8 (2) 0.25
system about an axis perpendicular to the rod and
(3) 0.5 (4) 0.4
passing through the centre of mass is :
[NEET Phase II-2016] 
Q.45 The moment of the force, F  4iˆ  5 ˆj  6kˆ at (2,0,–3),
m1m 2 2 m1  m 2 2
(1) m  m  (2) m m  about the point (2, – 2, –2), is given by [NEET-2018]
1 2 1 2
(1) 7iˆ  8 ˆj  4kˆ (2) 4iˆ  ˆj  8kˆ
(3) (m1 + m2) 2 (4) m1m 2  2
(3) 8iˆ  4 ˆj  7kˆ (4) 7iˆ  4 ˆj  8kˆ
Q.40 From a disc of radius R and mass M, a circular hole of
diameter R, whose rim passes through the centre is cut. Q.46 A solid sphere is rotating freely about its symmetry
What is the moment of inertia of the remaining part of axis in free space. The radius of the sphere is increased
the disc about a perpendicular axis, passing through keeping its mass same. Which of the following physical
the centre ? [NEET Phase I-2016] quantities would remain constant for the sphere?
[NEET-2018]
11MR 2 9 MR 2
(1) (2) (1) Rotational kinetic energy
32 32
(2) Moment of inertia
(3) Angular velocity
15 MR 2 13 MR 2
(3) (4) (4) Angular momentum
32 32
Q.47 A solid sphere is in rolling motion. In rolling motion a
Q.41 A rope wound around a hollow cylinder of mass 3 kg
body possesses translational kinetic energy (Kt) as well
and radius 40 cm. What is the angular acceleration of
as rotational kinetic energy (Kr) simultaneously. The
the cylinder if the rope is pulled with a force of 30N?
ratio Kt : (Kt + Kr ) for the sphere is [NEET-2018]
[NEET-2017]
(1) 0.25 rad s–2 (2) 25 rad s–2 (1) 10 : 7 (2) 5 : 7
(3) 5 rad s–2 (4) 25 rad s–2 (3) 7 : 10 (4) 2 : 5

74
Motion of System of Particles and Rigid Body

ANSWER KEY
EXERCISE-I
Q.1 (4) Q.2 (4) Q.3 (2) Q.4 (2) Q.5 (4) Q.6 (3) Q.7 (4) Q.8 (3) Q.9 (2) Q.10 (2)
Q.11 (3) Q.12 (2) Q.13 (3) Q.14 (4) Q.15 (4) Q.16 (1) Q.17 (2) Q.18 (3) Q.19 (4) Q.20 (1)
Q.21 (3) Q.22 (4) Q.23 (2) Q.24 (1) Q.25 (1) Q.26 (1) Q.27 (1) Q.28 (1) Q.29 (3) Q.30 (4)
Q.31 (2) Q.32 (2) Q.33 (1) Q.34 (2) Q.35 (3) Q.36 (4) Q.37 (1) Q.38 (3) Q.39 (1) Q.40 (4)
Q.41 (1) Q.42 (1) Q.43 (4) Q.44 (1) Q.45 (1) Q.46 (3) Q.47 (3) Q.48 (3) Q.49 (1) Q.50 (3)
Q.51 (2) Q.52 (3) Q.53 (3) Q.54 (1) Q.55 (4) Q.56 (4) Q.57 (2) Q.58 (2) Q.59 (3) Q.60 (4)
Q.61 (4) Q.62 (2) Q.63 (4) Q.64 (1) Q.65 (4) Q.66 (1) Q.67 (4) Q.68 (4) Q.69 (3) Q.70 (1)
Q.71 (2) Q.72 (3) Q.73 (1) Q.74 (4) Q.75 (2) Q.76 (4) Q.77 (4) Q.78 (2) Q.79 (2) Q.80 (2)
Q.81 (3) Q.82 (3) Q.83 (1) Q.84 (4) Q.85 (4) Q.86 (3) Q.87 (4) Q.88 (1) Q.89 (1) Q.90 (3)
Q.91 (2) Q.92 (2) Q.93 (1) Q.94 (3) Q.95 (2) Q.96 (2) Q.97 (4) Q.98 (4) Q.99 (2) Q.100 (1)
Q.101 (1) Q.102 (4) Q.103 (2) Q.104 (2) Q.105 (4) Q.106 (4) Q.107 (3) Q.108 (3) Q.109 (3) Q.110 (1)
Q.111 (3) Q.112 (2) Q.113 (3) Q.114 (1) Q.115 (4) Q.116 (3) Q.117 (4) Q.118 (3) Q.119 (4) Q.120 (2)
Q.121 (1) Q.122 (2) Q.123 (4) Q.124 (3) Q.125 (1) Q.126 (1) Q.127 (4) Q.128 (1) Q.129 (3) Q.130 (1)
Q.131 (4) Q.132 (2) Q.133 (3) Q.134 (3) Q.135 (3) Q.136 (1) Q.137 (2) Q.138 (2) Q.139 (2) Q.140 (3)
Q.141 (3) Q.142 (2) Q.143 (1) Q.144 (1) Q.145 (2) Q.146 (4) Q.147 (1) Q.148 (2) Q.149 (1) Q.150 (3)
Q.151 (1) Q.152 (2) Q.153 (2) Q.154 (2) Q.155 (3) Q.156 (1) Q.157 (2) Q.158 (3)

EXERCISE-II

Q.1 (1) Q.2 (3) Q.3 (3) Q.4 (2) Q.5 (2) Q.6 (1) Q.7 (3) Q.8 (1) Q.9 (4) Q.10 (3)
Q.11 (2) Q.12 (1) Q.13 (3) Q.14 (1) Q.15 (2) Q.16 (4) Q.17 (2) Q.18 (1) Q.19 (1) Q.20 (3)
Q.21 (1) Q.22 (1) Q.23 (2) Q.24 (2) Q.25 (2) Q.26 (2) Q.27 (3) Q.28 (2) Q.29 (1) Q.30 (4)
Q.31 (1) Q.32 (3) Q.33 (3) Q.34 (3) Q.35 (2) Q.36 (1) Q.37 (2) Q.38 (1) Q.39 (1) Q.40 (1)
Q.41 (2) Q.42 (1) Q.43 (3) Q.44 (2) Q.45 (4) Q.46 (1) Q.47 (2) Q.48 (3) Q.49 (1) Q.50 (1)
Q.51 (3) Q.52 (3) Q.53 (3) Q.54 (2) Q.55 (4) Q.56 (1) Q.57 (2) Q.58 (4) Q.59 (2) Q.60 (2)
Q.61 (4) Q.62 (4) Q.63 (1) Q.64 (1) Q.65 (3) Q.66 (3) Q.67 (2) Q.68 (2) Q.69 (3) Q.70 (1)
Q.71 (3) Q.72 (4) Q.73 (4) Q.74 (1) Q.75 (4) Q.76 (1) Q.77 (1) Q.78 (4) Q.79 (4) Q.80 (3)
Q.81 (2) Q.82 (4) Q.83 (3) Q.84 (3) Q.85 (2) Q.86 (2) Q.87 (3) Q.88 (2) Q.89 (2) Q.90 (2)
Q.91 (1) Q.92 (1) Q.93 (1) Q.94 (4) Q.95 (3) Q.96 (4) Q.97 (2)

EXERCISE-III
KVPY
PREVIOUS YEAR'S
Q.1 (C) Q.2 (D) Q.3 (D) Q.4 (B) Q.5 (B) Q.6 (D) Q.7 (A) Q.8 (A) Q.9 (C) Q.10 (A)
Q.11 (C) Q.12 (A) Q.13 (A) Q.14 (C) Q.15 (A) Q.16 (C) Q.17 (B) Q.18 (C) Q.19 (C) Q.20 (B)
Q.21 (D) Q.22 (A) Q.23 (A) Q.24 (A) Q.25 (A) Q.26 (B) Q.27 (D) Q.28 (C) Q.29 (B)

AIPMT / NEET
PREVIOUS YEAR'S
Q.1 (2) Q.2 (1) Q.3 (2) Q.4 (1) Q.5 (3) Q.6 (4) Q.7 (2) Q.8 (3) Q.9 (4) Q.10 (3)
Q.11 (3) Q.12 (2) Q.13 (1) Q.14 (None)Q.15 (2) Q.16 (1) Q.17 (4) Q.18 (4) Q.19 (4) Q.20 (4)
Q.21 (2) Q.22 (1) Q.23 (1) Q.24 (1) Q.25 (1) Q.26 (3) Q.27 (2) Q.28 (3) Q.29 (2) Q.30 (4)
Q.31 (1) Q.32 (3) Q.33 (2) Q.34 (3) Q.35 (3) Q.36 (4) Q.37 (3) Q.38 (2) Q.39 (1) Q.40 (4)
Q.41 (2) Q.42 (1) Q.43 (3) Q.44 (2) Q.45 (4) Q.46 (4) Q.47 (2)
75

You might also like